The question bank may take some time to load… Just enough time to stretch, blink a few times, and question your life choices — but not too long, we promise!
We recommend going Full Screen for the best experience. Have Fun !
Report a question
NEUROSCIENCE – 2019
Questions from the 2019 Module + Annual
Which macronutrient is the primary and most immediate energy source for cells, especially the brain and muscles?
1 / 128
Category:
Neurosciences – Biochemistry
In adults, 40-60% energy is provided by which of the following?
In adults , approximately 40-60% of daily energy comes from carbohydrates . Carbohydrates are the primary source of energy for most individuals and are broken down into glucose , which is used for cellular metabolism.
The recommended dietary intake suggests that carbohydrates should contribute 45-65% of total caloric intake in a balanced diet.
The brain and red blood cells (RBCs) exclusively rely on glucose as an energy source under normal conditions.
Excess carbohydrates are stored as glycogen (in the liver and muscles) or converted into fat for long-term energy storage.
Why the Other Options Are Wrong:
Lipids (Incorrect )
Lipids (fats) are an important secondary energy source , especially during fasting or prolonged exercise.
However, in a typical adult diet, fats contribute about 20-35% of energy intake , which is lower than carbohydrates.
Lipids do not provide the majority (40-60%) of energy in most diets.
Vitamins (Incorrect )
Vitamins are essential micronutrients required for metabolic reactions, but they do not provide energy .
They serve as coenzymes and help regulate metabolism but are not a direct energy source.
Minerals (Incorrect )
Minerals (e.g., calcium, iron, magnesium) are involved in various physiological functions but do not provide energy.
They help in enzyme activation, fluid balance, and nerve transmission , but they do not contribute to caloric intake.
Nucleic Acids (Incorrect )
Nucleic acids (DNA & RNA) are responsible for genetic information storage and protein synthesis .
They do not serve as a significant energy source in metabolism.
When a cranial defect results in a visible sac outside the skull, think about what normally resides inside the cranial vault. Consider what would be pushed outward if the skull bones failed to close around the developing brain.
2 / 128
Category:
Neurosciences – Pathology
A 6-month-old child suffering from encephalocele has a 6 x 6 cm protrusion on his head. Which of the following would be present in the protrusion?
Encephalocele is a neural tube defect resulting from failure of closure of the cranial portion of the neural tube during early embryonic development. It leads to a sac-like protrusion of intracranial contents through a defect in the skull , commonly in the occipital region .
The contents of the sac can vary depending on severity:
Meningocele : contains only meninges and cerebrospinal fluid
Meningoencephalocele : contains meninges and brain tissue ✅
Meningohydroencephalocele : contains meninges, brain tissue, and part of the ventricular system
In this case, the size (6×6 cm) and diagnosis of encephalocele strongly suggest the presence of both brain tissue and meninges within the protrusion.
❌ Why the Other Options Are Incorrect:
None of these : Incorrect—encephalocele always contains at least meninges and often brain tissue.
Meninges : Only correct if it were a meningocele , but the broader term encephalocele implies brain involvement .
Spinal cord : Not applicable—this is a cranial defect , not a spinal one.
Blood : While blood vessels may be present, the main pathological contents are neural tissue and meninges , not a hematoma.
Which structure in the basal ganglia degenerates in Parkinson’s disease , leading to dopamine depletion and motor dysfunction?
3 / 128
Category:
Neurosciences – Pathology
Which of the following happens in Parkinson’s disease?
Parkinson’s disease (PD) is a neurodegenerative disorder characterized by the progressive loss of dopaminergic neurons in the substantia nigra pars compacta (SNpc) . This leads to dopamine deficiency in the striatum , which disrupts the normal function of the basal ganglia, causing motor symptoms such as:
Bradykinesia (slow movements)
Rigidity (muscle stiffness)
Resting tremor (pill-rolling tremor)
Postural instability
The loss of dopaminergic neurons affects both the direct and indirect pathways of movement regulation. The resulting dopamine deficiency leads to overactivity of the inhibitory indirect pathway and underactivity of the excitatory direct pathway , causing motor impairment.
Why the Other Options Are Wrong:
D1 Receptors Are Destroyed (Incorrect )
D1 receptors are excitatory and are part of the direct pathway , which facilitates movement.
While dopamine loss affects D1 receptor stimulation , the receptors themselves are not destroyed .
D2 Receptors Are Destroyed (Incorrect )
D2 receptors are inhibitory and are part of the indirect pathway , which suppresses unwanted movement.
Dopamine loss leads to decreased activation of D2 receptors , but the receptors themselves remain intact.
D3 Receptors Are Destroyed (Incorrect )
D3 receptors are found in the limbic system and basal ganglia and play a role in reward and mood regulation .
While dopamine deficiency may alter D3 receptor function, these receptors are not specifically destroyed in Parkinson’s disease .
All of These (Incorrect )
While dopamine loss affects the function of D1, D2, and D3 receptors, the receptors themselves are not destroyed —only the dopaminergic neurons that release dopamine.
“Which term describes how we perceive things like touch, temperature, and pain from the environment?”
4 / 128
Category:
Neurosciences – Physiology
What is the sensitivity to the stimuli outside the body called?
Exteroception refers to the sensitivity to stimuli originating outside the body , such as:
Touch
Temperature
Pain
Vision
Hearing
Smell
Taste
It is mediated by sensory receptors in the skin (mechanoreceptors, thermoreceptors, nociceptors) and special senses (eyes, ears, nose, and tongue) .
Why the Other Options Are Wrong:
Kinesthesia ❌
Refers to awareness of body movement and is a component of proprioception .
It helps in coordinating voluntary movements but does not involve external stimuli.
Interoception ❌
Refers to sensation from inside the body (e.g., hunger, thirst, visceral pain, blood pressure changes).
It is mediated by visceral receptors rather than external sensory organs.
Nociception ❌
Refers to pain perception but can be both external (exteroception) or internal (interoception) .
It specifically involves nociceptors detecting harmful stimuli.
Proprioception ❌
Refers to the body’s awareness of position and movement .
It is mediated by muscle spindles, Golgi tendon organs, and joint receptors , but it does not involve external stimuli.
“Which reflex does not involve a tendon being tapped to produce a muscle contraction?”
5 / 128
Category:
Neurosciences – Physiology
Which of the following is not a deep tendon reflex?
Deep tendon reflexes (DTRs) are monosynaptic stretch reflexes that involve direct communication between a sensory neuron and a motor neuron at the spinal cord level. These reflexes are elicited by tapping a tendon, which stretches the muscle and triggers an involuntary contraction.
The conjunctival reflex , on the other hand, is a superficial reflex rather than a deep tendon reflex.
Why the Other Options Are Deep Tendon Reflexes:
Patellar reflex ✅ (Deep tendon reflex)
Also called the knee-jerk reflex .
Tests L2-L4 spinal segments .
Elicited by tapping the patellar tendon , leading to quadriceps contraction .
Biceps reflex ✅ (Deep tendon reflex)
Tests C5-C6 spinal segments .
Elicited by tapping the biceps tendon , causing biceps contraction .
Ankle reflex ✅ (Deep tendon reflex)
Also called the Achilles reflex .
Tests S1-S2 spinal segments .
Elicited by tapping the Achilles tendon , causing plantar flexion .
Brachioradialis reflex ✅ (Deep tendon reflex)
Tests C5-C6 spinal segments .
Elicited by tapping the brachioradialis tendon , leading to forearm flexion and supination .
Why the Conjunctival Reflex Is Not a Deep Tendon Reflex:
The conjunctival reflex is a superficial reflex that involves the eye mucosa .
It is mediated by cranial nerves :
Afferent limb: CN V1 (ophthalmic branch of trigeminal nerve) .
Efferent limb: CN VII (facial nerve) , causing blinking .
It does not involve a tendon or muscle stretch , making it different from deep tendon reflexes.
“Which condition involves an abnormally large posterior fossa with cerebellar malformations and hydrocephalus?”
6 / 128
Category:
Neurosciences – Embryology
Which of the following is associated with an enlargement of the posterior cranial fossa?
Dandy-Walker syndrome is a congenital brain malformation characterized by enlargement of the posterior cranial fossa , affecting the cerebellum and fourth ventricle. Key features include:
Hypoplasia or agenesis of the cerebellar vermis .
Cystic dilation of the fourth ventricle , leading to hydrocephalus.
Enlargement of the posterior fossa , often with upward displacement of the tentorium and torcula.
This condition is caused by abnormal development of the rhombencephalon (hindbrain) and is often associated with hydrocephalus, developmental delay, and ataxia .
Why the Other Options Are Wrong:
Holoprosencephaly ❌
Defect in forebrain (prosencephalon) development , leading to failure of cerebral hemispheres to separate .
Associated with midline facial abnormalities (e.g., cyclopia, cleft lip).
Does not involve the posterior fossa .
Arnold-Chiari type I ❌
Cerebellar tonsillar herniation through the foramen magnum .
Usually presents in late childhood/adulthood with headaches and syringomyelia .
No significant enlargement of the posterior fossa .
Arnold-Chiari type II ❌
More severe than Type I , associated with myelomeningocele and hydrocephalus .
Herniation of cerebellar vermis and medulla through foramen magnum .
Posterior fossa is small , not enlarged.
Agenesis of the corpus callosum ❌
Failure of corpus callosum development , leading to cognitive and motor deficits .
No effect on the posterior fossa .
“Which reflex involves a deep tendon response rather than being triggered by skin or mucosal stimulation?”
7 / 128
Category:
NeuroSciences – Anatomy
Which of the following is not a superficial reflex?
Superficial reflexes are reflexes elicited by gentle stimulation of the skin or mucous membranes. These reflexes are polysynaptic , meaning they involve both sensory and motor neurons with interneurons in between.
Deep tendon reflexes (DTRs) , on the other hand, are monosynaptic stretch reflexes , meaning they involve direct synaptic communication between sensory and motor neurons in the spinal cord without interneurons.
✅ Superficial Reflexes:
Plantar reflex → Stroking the lateral sole of the foot causes toe flexion (normal response) or Babinski sign (abnormal in adults).
Conjunctival reflex → Stimulation of the conjunctiva (eye mucosa) causes blinking.
Corneal reflex → Touching the cornea with a wisp of cotton causes blinking (CN V and VII).
Abdominal reflex → Stroking the abdomen results in contraction of the abdominal muscles.
❌ Deep Tendon Reflex (DTR) – Patellar Reflex:
The patellar reflex (knee-jerk reflex) is a monosynaptic deep tendon reflex , not a superficial reflex.
It involves the L2-L4 spinal segments and tests the function of the femoral nerve .
A tap on the patellar tendon stretches the quadriceps muscle, triggering a reflexive knee extension.
Why the Other Options Are Superficial Reflexes:
Plantar reflex ✅ (Superficial)
Elicited by stroking the lateral sole of the foot.
Tests L5-S1 spinal segments .
Conjunctival reflex ✅ (Superficial)
Touching the conjunctiva causes blinking .
Mediated by CN V (sensory) and CN VII (motor) .
Corneal reflex ✅ (Superficial)
Touching the cornea causes blinking .
Mediated by CN V1 (afferent) and CN VII (efferent) .
Abdominal reflex ✅ (Superficial)
Stroking the abdominal skin causes contraction of the underlying muscles.
Tests T7-T12 spinal segments .
“Which part of the brain acts as the body’s natural clock, synchronizing sleep-wake cycles with daylight?”
8 / 128
Category:
Neurosciences – Physiology
The suprachiasmatic nucleus is responsible for which of the following functions?
The suprachiasmatic nucleus (SCN) is a small, specialized cluster of neurons located in the anterior hypothalamus , just above the optic chiasm . It is known as the body’s master clock and is responsible for maintaining circadian rhythms —the body’s internal 24-hour biological cycle.
The SCN receives direct input from the retina via the retinohypothalamic tract , allowing it to synchronize circadian rhythms with light and darkness .
It regulates the pineal gland ’s secretion of melatonin , a hormone that promotes sleep.
Disruptions in SCN function (e.g., due to jet lag or shift work) can lead to sleep disorders, mood disturbances, and metabolic dysfunction .
Why the Other Options Are Wrong:
Release of anti-diuretic hormone (ADH) ❌
ADH (vasopressin) is released by the supraoptic nucleus and paraventricular nucleus of the hypothalamus, not the suprachiasmatic nucleus .
Thermoregulation ❌
Thermoregulation is primarily controlled by the preoptic nucleus and posterior hypothalamus , not the SCN.
Control of feeding impulses ❌
Feeding behavior is regulated by the lateral hypothalamus (hunger center) and the ventromedial hypothalamus (satiety center) .
Release of oxytocin ❌
Oxytocin is primarily produced by the paraventricular nucleus of the hypothalamus and stored in the posterior pituitary , not the SCN.
“Which two structures form the main input region of the basal ganglia and are involved in motor control and learning?”
9 / 128
Category:
NeuroSciences – Anatomy
The neostriatum consists of which of the following pairs of structures?
The neostriatum (or striatum) is a major component of the basal ganglia , which is involved in motor control, habit formation, and cognitive functions . It consists of: ✔ Caudate nucleus ✔ Putamen
Both the caudate nucleus and putamen share similar embryological origins and function as the primary input structures of the basal ganglia , receiving signals from the cerebral cortex .
Why the Other Options Are Wrong:
Putamen and globus pallidus ❌
Together, these structures form the lentiform nucleus , but the globus pallidus is not part of the neostriatum .
The globus pallidus is functionally distinct and plays a role in the output of the basal ganglia rather than receiving input.
Lentiform and globus pallidus ❌
The lentiform nucleus consists of the putamen and globus pallidus , but it is not equivalent to the neostriatum .
The globus pallidus belongs to the paleostriatum (pallidum) , not the neostriatum.
Lentiform and caudate nucleus ❌
The lentiform nucleus includes the putamen and globus pallidus , but the neostriatum only includes the putamen and caudate nucleus .
Putamen and lentiform nucleus ❌
The lentiform nucleus consists of the putamen + globus pallidus , so saying “Putamen and lentiform nucleus” would be redundant and incorrect .
“Which ventricular cavity lies between the two halves of the thalamus?”
10 / 128
Category:
Neurosciences – Embryology
Which of the following cavities is present in the diencephalon?
The third ventricle is the fluid-filled cavity located within the diencephalon . It is part of the ventricular system of the brain and plays a crucial role in cerebrospinal fluid (CSF) circulation . The third ventricle :
Lies between the two halves of the thalamus .
Connects to the lateral ventricles via the foramen of Monro .
Drains into the cerebral aqueduct , which leads to the fourth ventricle .
Since the diencephalon consists of the thalamus, hypothalamus, and epithalamus , the third ventricle is the only ventricular cavity present in this region .
Why the Other Options Are Wrong:
Lateral ventricles ❌
The lateral ventricles are located in the cerebral hemispheres (telencephalon) , not in the diencephalon.
They connect to the third ventricle via the interventricular foramen (Foramen of Monro) .
Cerebral aqueduct ❌
The cerebral aqueduct (Aqueduct of Sylvius) is a narrow channel in the midbrain that connects the third ventricle (diencephalon) to the fourth ventricle (pons & medulla).
It is not itself a cavity within the diencephalon .
Fourth ventricle ❌
The fourth ventricle is located in the hindbrain (pons and medulla) , not the diencephalon.
None of these ❌
Incorrect because the third ventricle is indeed present in the diencephalon .
“Which essential amino acid is known for its role in mood regulation and is also a precursor for melatonin?”
11 / 128
Category:
Neurosciences – Biochemistry
Which of the following amino acids is serotonin derived from?
Serotonin (5-hydroxytryptamine, 5-HT) is a monoamine neurotransmitter primarily synthesized from the essential amino acid tryptophan through the following pathway:
Tryptophan → 5-Hydroxytryptophan (5-HTP)
Enzyme: Tryptophan hydroxylase
Cofactor: Tetrahydrobiopterin (BH₄)
5-Hydroxytryptophan (5-HTP) → Serotonin (5-HT)
Enzyme: Aromatic L-amino acid decarboxylase
Cofactor: Pyridoxal phosphate (Vitamin B₆)
Serotonin is involved in mood regulation, sleep, appetite, and gastrointestinal function . It is primarily found in the brain, platelets, and enterochromaffin cells of the gut .
Why the Other Options Are Wrong:
Arginine ❌
Arginine is a precursor for nitric oxide (NO) and urea cycle intermediates , not serotonin.
Phenylalanine ❌
Phenylalanine is a precursor for tyrosine , which is further converted into dopamine, norepinephrine, and epinephrine , but not serotonin.
Tyrosine ❌
Tyrosine leads to catecholamine synthesis (dopamine, norepinephrine, and epinephrine), not serotonin.
Glutamate ❌
Glutamate is a precursor for γ-aminobutyric acid (GABA) , the brain’s major inhibitory neurotransmitter, but it does not contribute to serotonin synthesis.
“Before reaching the target organ, the sympathetic neuron first communicates with another neuron in a ganglion. What is the common neurotransmitter at this junction?”
12 / 128
Category:
Neurosciences – Physiology
Which neurotransmitter is most commonly released by pre-ganglionic sympathetic neurons?
In the autonomic nervous system (ANS) , pre-ganglionic neurons release neurotransmitters to communicate with post-ganglionic neurons . The sympathetic division follows this pattern:
Pre-ganglionic sympathetic neurons (originating in the spinal cord) release acetylcholine (ACh) .
ACh binds to nicotinic receptors (N₂) on post-ganglionic neurons.
Post-ganglionic sympathetic neurons then release norepinephrine (NE) to act on target organs, except for the sweat glands, where ACh is released instead.
Thus, the most commonly released neurotransmitter by pre-ganglionic sympathetic neurons is ACh .
Why the Other Options Are Wrong:
Serotonin (5-HT) ❌
Serotonin is mainly involved in mood regulation, sleep, and gastrointestinal motility , but it is not the primary neurotransmitter in sympathetic pathways .
Gamma-aminobutyric acid (GABA) ❌
GABA is the main inhibitory neurotransmitter in the CNS , but it is not involved in sympathetic pre-ganglionic transmission .
Epinephrine ❌
Epinephrine is released from the adrenal medulla , but not by pre-ganglionic sympathetic neurons. Instead, ACh stimulates the adrenal medulla , which then secretes epinephrine.
Norepinephrine (NE) ❌
Norepinephrine is the primary neurotransmitter of post-ganglionic sympathetic neurons, not pre-ganglionic neurons.
“Before anything else, what should be done immediately to minimize the risk of infection at the site of injury?”
13 / 128
Category:
Neurosciences – Community Medicine + Behavioural Sciences
A 30-year-old man reported to the emergency department with a complaint of a dog bite on the right arm. The casualty is conscious but the wound is bleeding. No previous medical or surgical history is reported. After history taking, what is the first step of management?
The first step in managing a dog bite is immediate wound care , which includes washing and debridement to reduce the risk of infection, including rabies and other bacterial infections (such as Pasteurella , Staphylococcus , and Streptococcus species).
Wound cleansing : The wound should be washed with soap and running water for at least 15 minutes .
Debridement : Any devitalized tissue should be removed to lower the risk of infection.
Antiseptic application : After thorough cleaning, an antiseptic (e.g., povidone-iodine) should be applied.
This step is critical because rabies virus, if present, can enter through open wounds. Proper wound cleansing is the most effective way to reduce the viral load.
Why the Other Options Are Wrong:
Testing the dog for rabies ❌
While observing the dog for signs of rabies (if available) is important, it is not the first step . Immediate wound care is prioritized before considering rabies status.
Administration of anti-rabies immunoglobulin (RIG) ❌
Rabies immunoglobulin is given only for Category III bites (deep wounds, mucosal exposure, or bites to high-risk areas like the face or hands).
Even if RIG is needed, wound cleansing comes first .
Administration of IV saline ❌
IV saline is not required unless there is severe bleeding or systemic involvement (e.g., shock). This is not a priority in a stable patient.
Administration of anti-rabies vaccine ❌
Rabies vaccination is important if indicated, but it is not the first step .
Vaccination is given after initial wound management to ensure proper immune response.
“How does the nervous system ensure that muscles contract in a coordinated way, with specific groups of fibers responding to a single command?”
14 / 128
Category:
Neurosciences – Physiology
What is a motor neuron, along with the skeletal muscle fibers innervated by that particular neuron, collectively called?
A motor unit consists of:
A single motor neuron (typically a lower motor neuron from the spinal cord or brainstem).
All the skeletal muscle fibers that it innervates.
When a motor neuron fires an action potential, all the muscle fibers in that motor unit contract simultaneously . Motor units are essential for coordinating muscle contractions and vary in size depending on the type of movement:
Small motor units (few muscle fibers per neuron) → Found in fine movements (e.g., in the fingers and eyes).
Large motor units (many muscle fibers per neuron) → Found in gross movements (e.g., in the legs and back).
Why the Other Options Are Wrong:
Primary motor area ❌
Refers to the precentral gyrus in the frontal lobe , which is responsible for voluntary motor control, not the connection between a motor neuron and its muscle fibers.
Muscle spindle ❌
A specialized sensory receptor within the muscle that detects stretch and changes in muscle length . It is not related to the motor unit itself but instead provides feedback to regulate muscle tone.
Motor complex ❌
This term is not a standard anatomical term for describing motor neurons and muscle fibers. It may refer to a general group of motor-related brain structures but is not specific to neuromuscular physiology.
Golgi tendon organ (GTO) ❌
A sensory receptor located in tendons that detects tension and force generated by muscle contractions to prevent excessive force and injury. It is not part of a motor unit.
“Think about where protein synthesis occurs in neurons. Would an axon, which depends on transport from the soma, have the necessary machinery for protein production?”
15 / 128
Category:
NeuroSciences – Anatomy
Which statement is false regarding axon structure and function?
Axons are specialized structures of neurons designed for transmitting action potentials over long distances. They originate from the axon hillock , which is a specialized region of the soma. However, axons lack Nissl substance (rough endoplasmic reticulum and ribosomes), which is why this statement is false .
Why is “Nissl substance is present” false?
Nissl substance (composed of rough ER and ribosomes) is found in the soma and dendrites but not in the axon .
The axon hillock and initial segment of the axon lack Nissl substance , making them distinct from the soma and dendrites.
Since axons do not have ribosomes or rough ER, they rely on the soma for protein synthesis , which is why they require axonal transport to receive necessary proteins.
Why the Other Options Are True:
“Transmit action potential” ✅ (True)
The primary function of the axon is to transmit action potentials from the soma to the axon terminals, where neurotransmitters are released to communicate with other neurons or muscles.
“Axons can arise from dendrites” ✅ (True)
While most axons arise from the axon hillock , in some neurons (like pseudounipolar neurons in the dorsal root ganglion), an axon may arise directly from a dendritic process.
“Axons can arise from soma” ✅ (True)
In most neurons, the axon originates directly from the soma (cell body) , specifically from a specialized region called the axon hillock .
“Axons can arise from hillock” ✅ (True)
The axon hillock is the specialized structure of the neuron where the axon begins and where action potentials are initiated .
“Think of a structure in the brainstem that contains motor and autonomic pathways. A hemorrhage here can cause paralysis and pinpoint pupils due to damage to descending tracts and sympathetic fibers.”
16 / 128
Category:
NeuroSciences – Anatomy
A 56-year-old man presents to the outpatient department with facial and limb paralysis. An examination also reveals that he has pupils reduced to the size of pinpoints. A hemorrhage is suspected. A computed tomography (CT) scan is performed. The CT image will show a hemorrhage at which of the following regions?
A hemorrhage in the pons (pontine hemorrhage) is strongly suggested by the combination of:
Facial and limb paralysis → Indicates corticospinal and corticobulbar tract involvement, which pass through the pons.
Pinpoint pupils → Caused by damage to the descending sympathetic fibers , which run in the pons and control pupil dilation. Loss of sympathetic input leads to unopposed parasympathetic activity, causing miosis (pinpoint pupils).
Pontine hemorrhages are often due to hypertensive hemorrhages , commonly affecting the paramedian branches of the basilar artery . These hemorrhages can cause coma, quadriplegia, and loss of horizontal eye movements if severe.
Why the Other Options Are Wrong:
Cerebrum:
A cerebral hemorrhage (e.g., in the basal ganglia or lobar regions) can cause contralateral weakness or sensory loss but does not typically cause pinpoint pupils . Instead, it may present with dilated or asymmetric pupils if there is herniation.
Midbrain:
A midbrain hemorrhage can cause coma and abnormal posturing due to damage to the reticular activating system .
Pupillary findings in midbrain lesions typically include blown (dilated) pupils due to involvement of the Edinger-Westphal nucleus or midposition fixed pupils (if both sympathetic and parasympathetic pathways are disrupted). Pinpoint pupils are not characteristic of midbrain damage.
Medulla:
A medullary hemorrhage can cause respiratory depression, dysphagia, and loss of gag reflex , but pinpoint pupils are not a hallmark feature.
The medulla contains the dorsal vagal nucleus and autonomic centers, but sympathetic pathways controlling pupil dilation do not primarily pass through the medulla .
Cerebellum:
A cerebellar hemorrhage typically presents with ataxia, vertigo, dysmetria, and nystagmus , but it does not cause pinpoint pupils or limb paralysis unless there is secondary brainstem compression.
“Think of a narrow passageway that serves as a bridge between two larger fluid-filled spaces in the brainstem.”
17 / 128
Category:
NeuroSciences – Anatomy
Which of the following cavities is present in the midbrain?
The cerebral aqueduct (also called the Aqueduct of Sylvius ) is a narrow channel that runs through the midbrain , connecting the third ventricle (in the diencephalon) to the fourth ventricle (in the pons and medulla). It allows the flow of cerebrospinal fluid (CSF) between these ventricles.
Since the cerebral aqueduct is the only ventricular structure passing through the midbrain , it is the correct answer.
Why the Other Options Are Wrong:
Fourth ventricle:
The fourth ventricle is located between the pons and medulla in front and the cerebellum behind. It is not present in the midbrain .
Foramen of Monro:
The Foramen of Monro (interventricular foramen) connects the lateral ventricles to the third ventricle . It is located in the diencephalon , not the midbrain.
Lateral ventricles:
The lateral ventricles are large CSF-filled cavities found in the cerebral hemispheres , not in the midbrain.
Third ventricle:
The third ventricle is a midline cavity located in the diencephalon (between the two halves of the thalamus), not in the midbrain.
“The name of the key component sounds like it comes from a lipid family and contains a molecule essential for neuron insulation.”
18 / 128
Category:
Neurosciences – Biochemistry
What is the main chemical constituent of the insulating sheaths on myelinated neurons?
The insulating sheath of myelinated neurons, known as the myelin sheath , is primarily composed of lipids and proteins , which help in rapid signal conduction along the axon. Among these, sphingomyelin , a type of sphingolipid , is a crucial component that contributes to the structural integrity and insulating properties of myelin.
Sphingomyelin is enriched in myelin membranes and plays a key role in forming the compact, multilamellar structure that allows for saltatory conduction —where electrical impulses “jump” from one node of Ranvier to another, significantly increasing conduction velocity.
Why the Other Options Are Wrong:
Galactocerebroside:
This is an important glycosphingolipid found in myelin, but it is not the primary insulating component. Instead, it plays a role in myelin stability and cell signaling.
Cholesterol:
While cholesterol is present in myelin and contributes to membrane fluidity and stability , it is not the main structural component responsible for insulation.
Choline:
Choline is a precursor for phosphatidylcholine , which is a major component of cell membranes but is not a key component of myelin.
Phosphagen:
Phosphagens (e.g., creatine phosphate) are involved in energy storage within cells, particularly in muscle and brain tissues, but they have no direct role in myelin insulation.
“If the cerebrum is the roof and the cerebellum is the basement, what structure serves as the ceiling in between?”
19 / 128
Category:
NeuroSciences – Anatomy
Which dural reflection separates the cerebrum from the cerebellum?
The tentorium cerebelli is a dural reflection that separates the cerebrum (specifically the occipital lobes) from the cerebellum . It is a horizontally oriented fold of dura mater that extends from the petrous part of the temporal bone and the transverse sinuses, creating a tent-like structure over the cerebellum.
Why the Other Options Are Wrong:
Falx cerebelli:
This structure is a small vertical dural reflection that separates the two hemispheres of the cerebellum , not the cerebrum from the cerebellum.
Falx cerebri:
This is a large, sickle-shaped dural reflection that separates the two cerebral hemispheres , not the cerebrum from the cerebellum.
Diaphragma sellae:
This is a small horizontal dural fold that covers the pituitary gland in the sella turcica of the sphenoid bone. It has an opening for the passage of the pituitary stalk (infundibulum) but has no role in separating the cerebrum from the cerebellum.
None of these:
Incorrect because the tentorium cerebelli is indeed the structure that separates the cerebrum from the cerebellum.
Think about the location and function of the Raphe nuclei in the brainstem and their role in regulating mood and sleep. Consider which neurotransmitter is most commonly associated with these nuclei and how it influences the transition from wakefulness to sleep
20 / 128
Category:
Neurosciences – Physiology
Which neurotransmitter is secreted by the Raphe nuclei that also plays a role in non-REM sleep?
The Raphe nuclei are a collection of neurons located in the brainstem, and they are the primary source of serotonin (5-HT) in the brain. Serotonin is a key neurotransmitter involved in regulating mood, appetite, and sleep-wake cycles. Specifically, serotonin is known to play a role in the initiation and maintenance of non-REM sleep , which is the restorative phase of sleep characterized by slow brain waves and reduced physiological activity.
During wakefulness, serotonin levels are high, promoting alertness. As sleep approaches, serotonin activity decreases, facilitating the transition to non-REM sleep. This is why serotonin is directly linked to non-REM sleep regulation.
Why the Other Options Are Wrong:
Gamma-aminobutyric acid (GABA):
GABA is the primary inhibitory neurotransmitter in the brain and is involved in promoting sleep by reducing neuronal activity. However, GABA is not primarily secreted by the Raphe nuclei. Instead, it is more associated with the hypothalamus and other sleep-regulating areas.
Acetylcholine:
Acetylcholine is involved in REM sleep (the dreaming phase) and wakefulness. It is secreted by neurons in the basal forebrain and brainstem, but not by the Raphe nuclei.
Noradrenaline:
Noradrenaline (norepinephrine) is associated with arousal, attention, and the fight-or-flight response. It is secreted by the locus coeruleus, not the Raphe nuclei, and is more active during wakefulness rather than non-REM sleep.
Adrenaline:
Adrenaline (epinephrine) is a hormone and neurotransmitter involved in the stress response and is not directly related to sleep regulation. It is secreted by the adrenal glands, not the Raphe nuclei.
This inhibitory neurotransmitter reduces neurotransmitter release by blocking calcium channels on the presynaptic membrane .
21 / 128
Category:
Neurosciences – Physiology
Which neurotransmitter acts on the presynaptic membrane as an inhibitor?
GABA (Gamma-aminobutyric acid) is the primary inhibitory neurotransmitter in the central nervous system (CNS) . It plays a critical role in presynaptic inhibition , which reduces neurotransmitter release from the presynaptic neuron.
Presynaptic inhibition occurs when GABA binds to GABA_B receptors on the presynaptic membrane, leading to:
Inhibition of voltage-gated calcium channels , preventing neurotransmitter release.
Opening of potassium channels , causing hyperpolarization and reducing excitability.
Decreased neurotransmitter release at the synaptic cleft , dampening the excitatory signal.
This mechanism is crucial for modulating excessive excitatory signals and maintaining neuronal balance .
Why the Other Options Are Incorrect:
Dopamine ❌
Dopamine can have inhibitory effects in some pathways (e.g., basal ganglia), but it does not primarily function as a presynaptic inhibitor .
It acts on D1 (excitatory) and D2 (inhibitory) receptors , mainly influencing reward and movement .
Serotonin ❌
Serotonin (5-HT) can have both excitatory and inhibitory effects , depending on the receptor subtype.
However, it does not primarily mediate presynaptic inhibition .
Epinephrine ❌
Epinephrine is mostly a hormone rather than a primary CNS neurotransmitter.
It has excitatory effects on adrenergic receptors and does not mediate presynaptic inhibition.
Acetylcholine ❌
Acetylcholine (ACh) is primarily excitatory , particularly at nicotinic receptors .
While muscarinic (M2) receptors can have inhibitory effects , ACh does not play a major role in presynaptic inhibition .
“Think about the water content of the intervertebral disc. Which MRI sequence highlights water-rich tissues as bright?”
22 / 128
Category:
Neuroscience – Radiology
In a T2WI section magnetic resonance imaging (MRI), the intervertebral disc appears as which of the following?
In T2-weighted MRI (T2WI) , tissues with high water content appear bright (hyperintense) . The intervertebral disc, particularly the nucleus pulposus (the inner gel-like core of the disc), contains a high amount of water. Therefore, it appears bright on T2WI. This brightness helps distinguish healthy discs from degenerated discs, which lose water content and appear darker.
Why the Other Options Are Incorrect:
Grey
Grey (intermediate signal intensity) is not characteristic of the intervertebral disc on T2WI. Grey signals are typically seen in tissues with moderate water content, such as muscle or cartilage, but not in the highly hydrated nucleus pulposus.
Black
Black (hypointense) signals on T2WI are seen in structures with low water content, such as cortical bone or calcified tissues. The intervertebral disc, being rich in water, does not appear black.
Jet black
Jet black is an extreme form of hypointensity, typically seen in structures like air or dense calcifications. The intervertebral disc does not appear jet black on T2WI.
Dark
Dark signals on T2WI are associated with low water content or fibrous tissues, such as the annulus fibrosus (the outer ring of the intervertebral disc). However, the nucleus pulposus, which is the dominant component of the disc, appears bright due to its high water content.
Think about what happens to the part of the axon that is separated from the cell body after injury. Which term describes the breakdown and clearance of this segment?”
23 / 128
Think about the consequences of increased intracranial pressure and brain herniation. What type of secondary hemorrhages occur in the brainstem due to vascular compression?”
24 / 128
Category:
Neurosciences – Pathology
An autopsy of the brain of a 40-year-old man reveals transtentorial herniation. Further examination reveals flame-shaped lesions in the midbrain and pons. Which term best describes this?
Duret hemorrhages are small, linear or flame-shaped hemorrhages that occur in the midbrain and pons as a result of transtentorial herniation . Transtentorial herniation occurs when increased intracranial pressure (e.g., due to a mass lesion or edema) causes the brain to herniate through the tentorial notch, compressing the brainstem. This compression disrupts the blood supply to the midbrain and pons, leading to secondary hemorrhages. Duret hemorrhages are a hallmark of severe brainstem injury due to herniation.
Why the Other Options Are Incorrect:
Saccular aneurysm
A saccular aneurysm is a balloon-like outpouching of a blood vessel, typically at branching points in the circle of Willis. It is not related to transtentorial herniation or brainstem hemorrhages. Saccular aneurysms are associated with subarachnoid hemorrhage when they rupture.
Intraparenchymal hemorrhages
Intraparenchymal hemorrhages are bleeds that occur within the brain tissue itself, often due to hypertension, trauma, or vascular malformations. While they can cause increased intracranial pressure, they are not specifically associated with the flame-shaped lesions in the midbrain and pons seen in Duret hemorrhages.
Slit hemorrhages
Slit hemorrhages are small, linear hemorrhages that occur in the retina, often associated with hypertensive retinopathy. They are not related to brainstem lesions or transtentorial herniation.
Lacunar infarcts
Lacunar infarcts are small, deep brain infarcts caused by the occlusion of small penetrating arteries, often due to hypertension or atherosclerosis. They are typically found in the basal ganglia, thalamus, or pons but are not associated with the flame-shaped hemorrhages seen in Duret hemorrhages.
“Think about the brain structure that processes fear and other strong emotions. Which structure helps you remember emotionally charged events?”
25 / 128
Category:
Neurosciences – Physiology
Which of the following brain structures is responsible for emotional memory?
The amygdala is a key brain structure responsible for emotional memory . It plays a central role in processing emotions, particularly fear and pleasure, and is involved in the formation and storage of memories associated with emotional events. The amygdala interacts closely with the hippocampus , which is responsible for forming declarative memories (facts and events), to link emotions to those memories. For example, the amygdala helps you remember a frightening experience or a joyful moment.
Why the Other Options Are Incorrect:
Cerebellum
The cerebellum is primarily involved in motor control , coordination, and balance. It also plays a role in motor learning and some forms of non-declarative memory (e.g., procedural memory, such as learning to ride a bike). However, it is not involved in emotional memory.
Hippocampus
The hippocampus is critical for forming and retrieving declarative memories (e.g., facts and events). While it works closely with the amygdala to link emotions to memories, it is not primarily responsible for emotional memory itself. Damage to the hippocampus impairs the ability to form new memories but does not directly affect emotional processing.
Basal ganglia
The basal ganglia are involved in motor control , habit formation , and reward processing . They play a role in procedural memory and reinforcement learning but are not directly involved in emotional memory.
Thalamus
The thalamus acts as a relay station for sensory and motor signals to the cerebral cortex. It also plays a role in regulating consciousness, sleep, and alertness. While it is involved in processing sensory information that may be associated with emotions, it is not directly responsible for emotional memory.
Think about the distribution of gray matter in the CNS. Is it confined to one region, or is it present throughout the brain and spinal cord?”
26 / 128
Category:
NeuroSciences – Anatomy
Which of the following is incorrect about the central nervous system gray matter?
It is only present in the cerebral region
This statement is incorrect because gray matter is not limited to the cerebral region . Gray matter is present throughout the CNS, including the spinal cord , brainstem , cerebellum , and cerebrum . In the spinal cord, gray matter is located centrally and has an H-shaped or butterfly-like appearance. In the brain, gray matter forms the outer layer (cortex) and is also present in deep nuclei.
Why the Other Options Are Correct:
It is present in all segments
This statement is correct. Gray matter is present in all segments of the CNS, from the spinal cord to the brain.
Gray matter is unmyelinated
This statement is correct. Gray matter consists primarily of neuronal cell bodies , dendrites , and unmyelinated axons . Myelinated axons are found in white matter.
It surrounds the central canal
This statement is correct. In the spinal cord, gray matter is arranged around the central canal , which is filled with cerebrospinal fluid (CSF).
It is H-shaped
This statement is correct. In the spinal cord, gray matter has an H-shaped or butterfly-like appearance when viewed in cross-section.
When the brain loses control over voluntary movement, muscles remain tense due to uninhibited spinal reflexes. Over time, they weaken but do not shrink rapidly. What kind of atrophy does this cause?”
27 / 128
Category:
Neurosciences – Physiology
Which of the following characterizes upper motor neuron lesions?
Upper motor neuron (UMN) lesions result in disuse atrophy , which occurs slowly due to lack of voluntary movement and chronic disuse of muscles .
Unlike lower motor neuron (LMN) lesions , where denervation leads to rapid, severe atrophy , in UMN lesions, muscle bulk is gradually lost due to inactivity .
The affected muscles still receive some tonic input from the spinal cord , preventing severe early atrophy.
Why the Other Options Are Incorrect:
Hypoactive Muscle Stretch Reflex – Incorrect
UMN lesions cause hyperreflexia , not hypoactive reflexes.
Loss of inhibitory control from the brain results in exaggerated muscle stretch reflexes (hyperreflexia) .
Hyporeflexia (decreased reflexes) is a feature of LMN lesions , not UMN lesions.
Flaccid Paralysis – Incorrect
UMN lesions cause spastic paralysis , not flaccid paralysis.
Flaccid paralysis (loss of muscle tone and reflexes) is a characteristic of LMN lesions , where there is direct damage to the peripheral motor neurons.
Fasciculations – Incorrect
Fasciculations (visible involuntary muscle twitches) are seen in LMN lesions , not UMN lesions.
They occur due to spontaneous discharges in denervated lower motor neurons .
Decreased Muscle Tone – Incorrect
UMN lesions cause increased muscle tone (spasticity), not decreased tone .
Loss of descending inhibitory control leads to hypertonia , which manifests as stiff, spastic muscles with increased resistance to movement .
Hypotonia (decreased muscle tone) is a feature of LMN lesions.
Think about the primary function of the corpus callosum. What happens to the fibers that normally connect the two hemispheres when the corpus callosum fails to develop?”
28 / 128
Category:
Neurosciences – Embryology
Agenesis of the corpus callosum is characterized by which of the following observations?
Commissural fibres are not connected
Agenesis of the corpus callosum is characterized by the absence or incomplete development of the corpus callosum , which is the largest commissural fiber bundle in the brain. These fibers normally connect the left and right cerebral hemispheres, allowing for communication between them. In agenesis of the corpus callosum, these commissural fibers either do not form or are improperly connected, leading to impaired interhemispheric communication.
Why the Other Options Are Incorrect:
Decreased gray matter bundles
This statement is incorrect because agenesis of the corpus callosum primarily affects white matter (commissural fibers), not gray matter. Gray matter consists of neuronal cell bodies, while white matter consists of myelinated axons. The condition does not directly involve a decrease in gray matter bundles.
Increased white matter bundles
This statement is incorrect because agenesis of the corpus callosum is characterized by the absence or reduction of white matter (specifically commissural fibers), not an increase. The lack of the corpus callosum means that the white matter bundles that would normally connect the hemispheres are missing or underdeveloped.
Cerebral hemispheres are not connected
This statement is partially correct but overly broad. While the corpus callosum is the primary structure connecting the cerebral hemispheres, other smaller commissures (e.g., the anterior commissure and hippocampal commissure) may still be present and provide some degree of interhemispheric connection. Therefore, it is more accurate to say that the commissural fibers of the corpus callosum are not connected rather than stating that the hemispheres are entirely disconnected.
Decreased white matter bundles
This statement is partially correct but not specific enough. While there is a decrease in white matter due to the absence of the corpus callosum, the key feature of agenesis of the corpus callosum is the lack of commissural fibers connecting the hemispheres. The term “decreased white matter bundles” is too general and does not specifically address the absence of the corpus callosum.
“Think about the pathways through which sensory and motor information enters the cerebellum. Which fibers carry this information and through which cerebellar peduncle do they travel?”
29 / 128
“Think about the brain structures involved in emotions, memory, and motivation. Which group of structures is interconnected and works together to regulate these functions?”
30 / 128
Category:
NeuroSciences – Anatomy
Which of the following structures are constituents of the limbic system?
Subcallosal, cingulate and parahippocampal gyri, hippocampal formation, amygdaloid nucleus, mammillary body and anterior thalamic nuclei
The limbic system is a network of interconnected structures that work together to regulate emotions, memory, and motivation. The key components of the limbic system include:
Subcallosal gyrus : Involved in emotional processing.
Cingulate gyrus : Plays a role in emotion formation and processing, as well as learning and memory.
Parahippocampal gyrus : Associated with memory encoding and retrieval.
Hippocampal formation : Critical for forming new memories (declarative memory).
Amygdaloid nucleus (amygdala) : Central to emotional responses, especially fear and aggression.
Mammillary bodies : Important for memory formation and recall.
Anterior thalamic nuclei : Involved in memory and emotional regulation.
These structures are interconnected and work together to integrate emotional and cognitive processes.
Why the Other Options Are Incorrect:
Cingulate gyrus and the uncus
While the cingulate gyrus is part of the limbic system, the uncus (a part of the parahippocampal gyrus) is not typically listed as a primary constituent of the limbic system. This option is incomplete and does not include other critical limbic structures.
Amygdaloid nucleus, red nucleus, and vestibular nucleus
The amygdaloid nucleus is part of the limbic system, but the red nucleus (involved in motor coordination) and the vestibular nucleus (involved in balance and spatial orientation) are not part of the limbic system. This option incorrectly includes non-limbic structures.
Hippocampal formation
The hippocampal formation is a key component of the limbic system, but this option is incomplete because it does not include other essential limbic structures like the amygdala, cingulate gyrus, or mammillary bodies.
Pulvinar of the thalamus and substantia nigra
The pulvinar of the thalamus is involved in visual processing, and the substantia nigra is part of the basal ganglia and is involved in motor control. Neither of these structures is part of the limbic system. This option is entirely incorrect.
“Oxygen and carbon dioxide need to reach the brain without restriction, but large proteins and charged molecules are blocked. What kind of substances can pass the BBB?”
31 / 128
Category:
Neurosciences – Physiology
Which of the following statements regarding the blood-brain barrier is true?
This question falls under the subject category: Physiology (Neurophysiology & Barrier Systems).
Correct Answer: It allows restricted passage of water and gases
The blood-brain barrier (BBB) is a selective permeability barrier that allows certain molecules to pass while restricting others .
Water and gases (such as oxygen and carbon dioxide) can cross the BBB through simple diffusion , but their passage is still regulated to maintain brain homeostasis.
Lipid-soluble substances (e.g., anesthetics, alcohol, nicotine) cross the BBB more easily than water-soluble molecules.
The BBB is formed by :
Tight junctions in endothelial cells of brain capillaries
Astrocyte foot processes , which help maintain barrier integrity
Pericytes , which regulate blood flow and BBB permeability
Why the Other Options Are Incorrect:
“It is well-developed in infants and children and becomes less efficient with age” – Incorrect
The BBB is immature in newborns and develops fully over time .
This is why newborns are more vulnerable to toxins, infections, and drugs that do not affect adults in the same way.
In aging, the BBB may show some decline , but it does not inherently become inefficient with age .
“The structure of the blood-brain barrier is identical in all areas of the nervous system” – Incorrect
The BBB is not uniform throughout the brain.
Some regions of the brain lack a BBB to allow for communication between the brain and blood. These areas are known as circumventricular organs (CVOs) and include:
Area postrema (vomiting center)
Median eminence and neurohypophysis (hormone release sites)
Pineal gland (melatonin secretion)
“It is absent in aggressive tumors of the brain” – Incorrect
While brain tumors disrupt the BBB , it is not completely absent .
Some aggressive tumors (e.g., glioblastomas ) induce new blood vessel formation (angiogenesis) , leading to a leaky BBB , but some barrier function remains .
“Drugs that are highly protein-bound can easily cross the blood-brain barrier” – Incorrect
Highly protein-bound drugs (e.g., warfarin, albumin-bound substances) do not easily cross the BBB .
Only lipid-soluble, non-protein-bound, and small molecules cross efficiently.
Water-soluble or large molecules require active transport mechanisms (e.g., glucose via GLUT1 transporter).
Think of a baby born with a swelling over the nasal bridge or forehead —which part of the skull must have a defect to allow brain herniation there?
32 / 128
Category:
Neurosciences – Embryology
Encephalocele most commonly involves which cranial fossa?
Encephalocele is a neural tube defect in which brain tissue and meninges herniate through a skull defect .
The location of the encephalocele depends on the site of the skull defect, and the most common site is the anterior cranial fossa , especially in frontal or ethmoidal bones .
These defects often present as:
Nasoethmoidal encephaloceles (most common)
Frontoethmoidal encephaloceles
These herniations may even protrude externally through the nose or forehead.
❌ Why the Other Options Are Incorrect:
Middle cranial fossa
Posterior cranial fossa
Can be involved in occipital encephaloceles , which are less common overall but more common in Western populations than Asia.
Still, anterior cranial fossa defects dominate globally.
Medial cranial fossa
Lateral cranial fossa
Your skin detects the cold wind as soon as you step outside. What type of receptor is responsible for this immediate sensation?”
33 / 128
Category:
Neurosciences – Physiology
Which receptors receive stimuli immediate from external surroundings?
Exteroreceptors are sensory receptors that detect immediate stimuli from the external environment .
They are located on or near the body surface (e.g., skin, mucous membranes).
They respond to external physical, thermal, mechanical, and chemical stimuli , such as:
Touch (Mechanoreceptors – e.g., Meissner’s corpuscles, Pacinian corpuscles)
Pain (Nociceptors)
Temperature (Thermoreceptors – e.g., Ruffini endings, Krause’s end bulbs)
These receptors allow the body to interact with and respond to environmental stimuli immediately .
Why the Other Options Are Incorrect:
Enteroreceptors – Incorrect
Enteroreceptors (interoceptors) detect internal bodily stimuli, such as blood pressure, organ stretch, and chemical changes in body fluids.
They are found in visceral organs and blood vessels , not in direct contact with the external surroundings.
Teloreceptors – Incorrect
Teloreceptors detect distant stimuli from the environment without direct contact (e.g., vision and hearing).
Examples:
Photoreceptors (rods & cones in the retina) – for vision
Auditory receptors (hair cells in the cochlea) – for hearing
Since teloreceptors process distant rather than immediate stimuli, they are incorrect for this question.
Proprioreceptors – Incorrect
Proprioreceptors detect body position, movement, and muscle tension .
They are found in muscles, tendons, and joints and help in balance and coordination .
Examples:
Muscle spindles (detect stretch)
Golgi tendon organs (detect tension)
These receptors respond to internal body movements , not external stimuli.
Chemoreceptors – Incorrect
Chemoreceptors detect chemical changes in the blood or external environment .
While some chemoreceptors (e.g., olfactory and taste receptors) receive external stimuli, they do not immediately detect physical environmental changes like exteroreceptors do .
Examples:
Carotid body chemoreceptors (monitor oxygen levels)
Taste buds (detect chemical composition of food)
“This glial cell type in the CNS provides structural support, regulates neurotransmitters, and plays a crucial role in maintaining the blood-brain barrier. What is it?”
34 / 128
“When you touch a hot object, the sensory receptors in your skin immediately generate an electrical change that may or may not trigger an action potential. What is this initial electrical response called?”
35 / 128
Category:
Neurosciences – Physiology
The nerve endings at the end of the receptor have a receptor potential due to which of the following?
potential is a graded change in membrane potential that occurs when a sensory receptor is stimulated. This change in potential is what ultimately triggers an action potential if the threshold is reached.
When a sensory receptor (such as a mechanoreceptor, thermoreceptor, or chemoreceptor) is stimulated, it generates a localized depolarization or hyperpolarization called a receptor potential .
This change in membrane potential occurs due to the opening or closing of ion channels in response to stimuli.
Unlike action potentials, receptor potentials are graded , meaning their amplitude depends on the strength of the stimulus .
Why the Other Options Are Incorrect:
Excitatory Post-Synaptic Potential (EPSP) – Incorrect
EPSPs occur in post-synaptic neurons at synapses , where neurotransmitters cause depolarization.
Receptor potentials occur in sensory receptors, not at synapses.
End Plate Potential – Incorrect
End plate potentials occur at the neuromuscular junction (NMJ) when acetylcholine (ACh) binds to receptors on skeletal muscle fibers , leading to muscle contraction.
Receptor potentials do not involve neurotransmitters at the NMJ .
Accommodation – Incorrect
Accommodation refers to the reduced responsiveness of a neuron to a prolonged stimulus , typically due to inactivation of voltage-gated Na⁺ channels .
While accommodation affects neuronal excitability, it does not directly generate receptor potentials .
Adaptation – Incorrect
Adaptation is a process where sensory receptors gradually reduce their response to a sustained stimulus (e.g., wearing clothes and not feeling them after some time).
It affects how long a receptor responds to a stimulus but does not directly generate receptor potentials .
“After a stroke, neurons undergo irreversible damage, and their cytoplasm turns a bright pink color due to protein breakdown. What histological change causes this?”
36 / 128
Category:
Neurosciences – Pathology
Neurons after injury become red because of which of the following?
This question falls under the subject category: Pathology (Neuropathology).
Correct Answer: Eosinophilia
Neurons become red after injury due to eosinophilia , which occurs as a result of neuronal cytoplasmic degeneration and protein denaturation .
When a neuron undergoes ischemic or hypoxic injury , it exhibits “red neuron” changes , which are seen in acute neuronal necrosis (e.g., stroke or hypoxia).
This red discoloration is due to the increased binding of eosin (an acidic dye) to denatured cytoplasmic proteins , leading to a deep eosinophilic (red) appearance under the microscope .
Characteristic features of “red neurons” include :
Eosinophilic cytoplasm (intensely pink or red)
Pyknosis (shrinkage and darkening of the nucleus)
Loss of Nissl substance (disappearance of basophilic ribosomal RNA granules)
Why the Other Options Are Incorrect:
Infiltration – Incorrect
Inflammatory infiltration occurs in infections or autoimmune conditions, leading to swelling and immune cell accumulation , but does not cause neurons to appear red .
Hemorrhage – Incorrect
Hemorrhage results from ruptured blood vessels , causing extravasation of red blood cells , but it does not directly turn neurons red .
Hemorrhagic strokes cause perivascular hemosiderin deposition , not eosinophilia of neurons.
Amyloid Deposition – Incorrect
Amyloid deposition is seen in Alzheimer’s disease and other neurodegenerative disorders, leading to plaques and neurofibrillary tangles , but it does not cause neuronal eosinophilia or red neurons .
Shrinkage of Nucleus – Incorrect
Nuclear shrinkage (pyknosis) is a feature of neuronal injury , but it does not cause red discoloration .
The red appearance is due to cytoplasmic eosinophilia , not nuclear changes alone.
“When you’re learning to play the piano, your brain constantly adjusts finger pressure and position based on how the keys feel. Which function of the motor cortex does this demonstrate?”
37 / 128
Category:
Neurosciences – Physiology
Sensory feedback helps the motor cortex to control which of the following?
This question falls under the subject category: Physiology (Neurophysiology & Motor Control).
Correct Answer: Precise Muscle Movements
The motor cortex (specifically the primary motor cortex, premotor cortex, and supplementary motor area ) relies on sensory feedback to fine-tune and control precise muscle movements .
Sensory feedback, primarily from the proprioceptive system, provides information about the position, stretch, and force of muscles , allowing the motor cortex to adjust movements in real time.
The somatosensory cortex , located just posterior to the motor cortex , processes tactile and proprioceptive inputs , which are then used by the motor cortex to refine motor execution.
This feedback is crucial for coordinated movements, adjusting force, and maintaining posture during motor activity.
Why the Other Options Are Incorrect:
Eye Movement – Incorrect
Eye movements are controlled by the brainstem (midbrain, pons) and cerebellum, not the motor cortex directly .
The frontal eye field (FEF) in the frontal cortex controls voluntary eye movement , but sensory feedback does not play a major role in fine control of eye movements the way it does for skeletal muscle.
Hormone Secretion from the Hypothalamus – Incorrect
The hypothalamus controls hormone secretion via the pituitary gland , but this process is regulated by neural and hormonal signals, not sensory feedback related to motor control .
The motor cortex does not influence hormonal secretion.
Oscillation of Vestibular System – Incorrect
The vestibular system , responsible for balance and spatial orientation , is regulated by the vestibular nuclei in the brainstem and the cerebellum , not the motor cortex .
While sensory feedback from the vestibular system affects posture and equilibrium , it is not primarily controlled by the motor cortex .
None of These – Incorrect
Sensory feedback plays a key role in refining muscle movement , making this option incorrect.
“This deep brain structure fine-tunes movement by inhibiting unwanted motion. When it fails, the result is tremors that disappear with voluntary activity. Which structure is it?
38 / 128
Category:
Neurosciences – Pathology
A lesion in which of the following is responsible for the symptoms of a man with resting tremors which disappear when working?
The presence of a resting tremor that disappears with voluntary movement is a hallmark sign of Parkinson’s disease , which results from a lesion in the basal ganglia, specifically the degeneration of the substantia nigra (pars compacta) .
The basal ganglia play a crucial role in the modulation of voluntary movements and muscle tone .
In Parkinson’s disease , the loss of dopaminergic neurons leads to resting tremors, bradykinesia (slowness of movement), rigidity, and postural instability .
The classic “pill-rolling tremor” is characteristic and disappears with purposeful activity , differentiating it from cerebellar tremors.
Why the Other Options Are Incorrect:
Pons – Incorrect
The pons is involved in motor relay, respiration, and facial movements , but it does not regulate movement initiation or suppress involuntary tremors .
Lesions in the pons cause motor weakness, facial palsy, or coordination deficits rather than resting tremors.
Medulla – Incorrect
The medulla oblongata controls autonomic functions such as breathing, heart rate, and reflexive actions , but it is not involved in fine motor control or tremors .
Lesions in the medulla typically cause respiratory failure, dysphagia, or cardiovascular instability , not Parkinsonian tremors.
Cerebellum – Incorrect
The cerebellum regulates coordination, balance, and fine motor control , but cerebellar tremors occur during voluntary movement (intention tremor), not at rest .
A cerebellar lesion causes ataxia, intention tremor, and dysmetria , which worsen with movement instead of disappearing.
Cerebrum – Incorrect
The cerebrum is responsible for higher cognitive functions, sensory processing, and voluntary movement , but not for movement modulation at the basal level .
Lesions in the cerebrum can cause paralysis, sensory loss, or cognitive dysfunction , but not resting tremors .
This brain region is responsible for emotions and memory processing, and it develops from the same part of the brain that gives rise to the cerebral cortex. Which one is it?”
39 / 128
Category:
NeuroSciences – Anatomy
The limbic cortex is a part of which of the following?
The limbic cortex is part of the telencephalon , which is the largest division of the forebrain (prosencephalon) and includes the cerebral cortex, basal ganglia, and limbic system .
The limbic system is crucial for emotion, memory, and motivation and includes:
Cingulate gyrus
Parahippocampal gyrus
Hippocampus
Amygdala
These structures are all derived from the telencephalon , making it the correct answer.
Why the Other Options Are Incorrect:
Rhombencephalon – Incorrect
The rhombencephalon (hindbrain) develops into the pons, medulla oblongata, and cerebellum , which are involved in motor control, autonomic functions, and coordination .
It does not include the limbic cortex.
Diencephalon – Incorrect
The diencephalon contains the thalamus, hypothalamus, epithalamus, and subthalamus .
While the hypothalamus is functionally linked to the limbic system (regulating emotions and autonomic functions), the limbic cortex itself is in the telencephalon .
Prosencephalon – Incorrect
The prosencephalon (forebrain) is the embryological precursor to both the telencephalon and diencephalon .
While the limbic cortex is within the prosencephalon , this term is too broad. The more specific correct answer is telencephalon .
Mesencephalon – Incorrect
The mesencephalon (midbrain) contains the tectum, tegmentum, and substantia nigra , involved in motor function, sensory processing, and reflexes .
It does not include any part of the limbic cortex .
“This sensory pathway helps you feel the texture of an object, know where your limbs are without looking, and appreciate the vibrations of music. Since it’s located at the back of the spinal cord, it’s left untouched in anterior cord syndrome. Can you name it?”
40 / 128
Category:
NeuroSciences – Anatomy
Which of the following tracts is spared in anterior cord syndrome?
This question falls under the subject category: Pathology (Neurology/Spinal Cord Injuries).
Correct Answer: Dorsal Column
In anterior cord syndrome , the dorsal column is spared because the lesion primarily affects the anterior two-thirds of the spinal cord , while the dorsal column is located in the posterior part of the spinal cord .
The dorsal column is responsible for fine touch, vibration, and proprioception .
Since it is located in the posterior spinal cord , it remains functionally intact in anterior cord syndrome.
Why the Other Options Are Incorrect:
Corticobulbar Tract – Incorrect
The corticobulbar tract carries motor signals to cranial nerves controlling facial and head movements .
This tract is located in the brainstem , not the spinal cord , so it is not relevant to anterior cord syndrome .
Anterior Spinothalamic Tract – Incorrect
The anterior spinothalamic tract carries crude touch and pressure .
It is located in the anterior spinal cord , which is affected in anterior cord syndrome , leading to loss of crude touch and pressure sensation .
Corticospinal Tract – Incorrect
The corticospinal tract (descending motor pathway) is located anteriorly in the spinal cord , making it vulnerable to damage in anterior cord syndrome .
Damage to this tract leads to bilateral motor weakness or paralysis below the level of injury.
Lateral Spinothalamic Tract – Incorrect
The lateral spinothalamic tract carries pain and temperature sensations .
It is located laterally but still within the affected anterior region , leading to loss of pain and temperature sensation below the lesion in anterior cord syndrome.
This structure in the midbrain is responsible for producing dopamine, and its degeneration leads to the characteristic tremors and rigidity of Parkinson’s disease. Can you identify it?”
41 / 128
Category:
Neurosciences – Pathology
Parkinson’s disease involves degeneration of which of the following?
Parkinson’s disease (PD) is primarily caused by the degeneration of dopaminergic neurons in the substantia nigra, particularly in the pars compacta (SNc) .
The substantia nigra is located in the midbrain and is part of the basal ganglia , which plays a crucial role in modulating movement .
Loss of dopamine-producing neurons leads to motor symptoms such as bradykinesia, resting tremor, rigidity, and postural instability .
The characteristic Lewy bodies (intracellular aggregates of α-synuclein ) are found in affected neurons.
Why the Other Options Are Incorrect:
Cerebellum – Incorrect
The cerebellum is involved in coordination and balance , but it is not directly affected in Parkinson’s disease .
Disorders of the cerebellum typically cause ataxia and dysmetria , which are different from the symptoms seen in PD.
Subthalamus – Incorrect
The subthalamic nucleus is part of the indirect pathway in the basal ganglia, and it plays a role in regulating movement.
While it is functionally involved in PD (often overactive due to loss of dopamine), it is not the primary site of degeneration .
Deep brain stimulation (DBS) of the subthalamic nucleus is used as a treatment for Parkinson’s disease.
Corpus Striatum – Incorrect
The corpus striatum consists of the caudate nucleus and putamen , which are involved in motor control.
Although Parkinson’s disease affects basal ganglia circuits, the primary site of neuron loss is the substantia nigra , not the striatum itself.
The striatum receives reduced dopamine input , leading to abnormal motor function.
Thalamus – Incorrect
The thalamus relays sensory and motor information to the cortex.
While it is indirectly involved in movement disorders, it does not undergo primary degeneration in Parkinson’s disease .
The thalamus is a target for DBS in tremor-dominant Parkinson’s disease , but it is not the main site of pathology.
“Even when you’re asleep, your body continues to breathe without conscious effort. The control of this vital function comes from a structure deep in the brainstem. Which one is it?”
42 / 128
Category:
Neurosciences – Physiology
Where is the respiratory center located?
This question falls under the subject category: Physiology (Neurophysiology).
Correct Answer: Medulla Oblongata
The respiratory center is primarily located in the medulla oblongata within the brainstem . It controls the rhythmic pattern of breathing by regulating inspiratory and expiratory muscle activity .
The respiratory center consists of three key groups:
Dorsal Respiratory Group (DRG) – Controls inspiration and integrates sensory input from the vagus and glossopharyngeal nerves.
Ventral Respiratory Group (VRG) – Controls forced expiration and some aspects of inspiration.
Pontine Respiratory Group (Pneumotaxic and Apneustic Centers in Pons) – Modulates the breathing pattern by adjusting the rate and depth of respiration .
The medulla oblongata automatically adjusts breathing based on chemoreceptor input , which detects carbon dioxide (CO₂), oxygen (O₂), and pH levels in the blood.
This regulation ensures appropriate gas exchange and maintains acid-base homeostasis .
Why the Other Options Are Incorrect:
Spinal Cord – Incorrect
While the phrenic nerve (C3-C5) in the spinal cord controls the diaphragm , the actual respiratory center that generates the breathing rhythm is not located in the spinal cord .
The spinal cord only relays signals from the medulla to respiratory muscles.
None of These – Incorrect
The respiratory center does exist , and it is located in the medulla oblongata .
Cerebrum – Incorrect
The cerebrum is responsible for voluntary breathing control , such as when you consciously hold your breath or take a deep breath.
However, the automatic, rhythmic control of breathing is handled by the medulla oblongata , not the cerebrum.
Midbrain – Incorrect
The midbrain controls reflexes, auditory, and visual processing , but it does not regulate respiration.
The respiratory center is located lower , in the medulla oblongata and pons.
“If you try walking with your eyes closed, your body still maintains balance, adjusting movements automatically. This structure fine-tunes coordination and helps you stand upright without consciously thinking about it.”
43 / 128
Category:
Neurosciences – Physiology
Which of the following is responsible for normal maintenance of tone and equilibrium?
The cerebellum is the primary structure responsible for maintaining muscle tone, posture, and equilibrium .
It receives sensory input from the vestibular system, spinal cord, and cerebral cortex to regulate balance and coordination.
The flocculonodular lobe (vestibulocerebellum) is particularly important for equilibrium and integrates information from the vestibular nuclei to control eye movements and balance.
The spinocerebellum regulates muscle tone and smooth motor execution.
Damage to the cerebellum leads to ataxia, impaired coordination, and postural instability .
Why the Other Options Are Incorrect:
Pons – Incorrect
The pons is a brainstem structure that serves as a relay center between the cerebrum and cerebellum.
It plays a role in motor control and coordination but does not directly regulate tone and equilibrium.
It contains nuclei for cranial nerves and assists in controlling respiration and facial movements , but its role in balance is indirect.
Substantia Nigra – Incorrect
The substantia nigra is part of the basal ganglia , located in the midbrain .
It is primarily involved in dopaminergic control of movement and is essential for smooth voluntary motion.
Degeneration of the substantia nigra leads to Parkinson’s disease , which affects voluntary movement , but it does not directly regulate equilibrium.
Midbrain – Incorrect
The midbrain contains important structures like the superior and inferior colliculi (responsible for visual and auditory reflexes) and red nucleus (involved in motor control).
While the red nucleus contributes to muscle tone , the midbrain as a whole does not regulate equilibrium as directly as the cerebellum does.
Medulla Oblongata – Incorrect
The medulla oblongata is involved in autonomic functions such as breathing, heart rate, and blood pressure regulation .
It contains the vestibular nuclei , which play a role in balance, but it does not directly control tone and equilibrium like the cerebellum does.
While it is crucial for reflexive postural adjustments , the cerebellum is the primary structure responsible for maintaining equilibrium.
“Think about the auditory pathway and where sound localization is processed. Which structure in the pons integrates auditory information from both ears to help determine the direction of sound?”
44 / 128
“These fibers are responsible for muscle contraction. Do they connect to the sensory part of the muscle spindle, or do they directly cause muscle movement?”
45 / 128
Category:
Neurosciences – Physiology
Which of the following is incorrect about alpha fibers?
Alpha motor fibers (α-fibers) are large, myelinated efferent fibers that innervate extrafusal skeletal muscle fibers and are responsible for generating forceful muscle contraction. They play a crucial role in the somatic motor system and are regulated by spinal reflex circuits .
Correct Answer: “They Attach to Intrafusal Fibers” (Incorrect Statement)
Why is “They Attach to Intrafusal Fibers” Incorrect?
Alpha motor neurons innervate extrafusal muscle fibers , which are the primary contractile fibers of skeletal muscle .
Intrafusal fibers are part of the muscle spindle , which detects muscle stretch and is instead innervated by gamma motor neurons (γ-fibers) , not alpha motor neurons.
Therefore, saying “they attach to intrafusal fibers” is incorrect because α-fibers only innervate extrafusal muscle fibers .
Why Are the Other Options Correct?
“They Are Inhibited by Renshaw Cells” ✅ (Correct)
Renshaw cells are inhibitory interneurons located in the spinal cord that provide negative feedback inhibition to alpha motor neurons via glycinergic synapses .
This mechanism prevents excessive excitation of motor neurons , helping to modulate muscle contraction and prevent overactivation .
“They Are Greater Than Gamma Fibers” ✅ (Correct)
Alpha fibers are larger in diameter and faster in conduction velocity than gamma fibers (γ-fibers) .
Alpha motor neurons conduct impulses at around 80–120 m/s , whereas gamma motor neurons conduct impulses at a slower rate (~10–45 m/s) .
Alpha motor neurons control forceful skeletal muscle contraction , while gamma motor neurons adjust muscle spindle sensitivity .
“They Are Efferent Fibers of the Lower Motor Neuron” ✅ (Correct)
Alpha motor neurons are the final common pathway for voluntary and reflex motor control .
They originate in the ventral horn of the spinal cord and send efferent signals to skeletal muscles , causing contraction.
They are classified as lower motor neurons (LMNs) because they directly stimulate muscle contraction .
“They Attach to Extrafusal Fibers” ✅ (Correct)
Extrafusal muscle fibers are the contractile units of skeletal muscle , generating movement.
Alpha motor neurons synapse directly on extrafusal muscle fibers , causing them to contract when stimulated.
“This small opening allows CSF to flow from the lateral ventricles into the midline third ventricle before continuing its journey downward. What is its name?”
46 / 128
Category:
NeuroSciences – Anatomy
The cerebrospinal fluid (CSF) leaves the lateral ventricles to flow into the third ventricle through which of the following structures?
Cerebrospinal fluid (CSF ) is produced by the choroid plexus in the lateral ventricles and follows a specific flow pathway through the ventricular system before being absorbed into the venous circulation.
Correct Answer: Foramen of Monro
Why is the Foramen of Monro the Correct Answer?
The foramen of Monro (interventricular foramen) connects the lateral ventricles to the third ventricle , allowing CSF to flow between them.
CSF is produced in the choroid plexus of the lateral ventricles and passes through the foramen of Monro to reach the third ventricle , where it continues its journey toward the fourth ventricle and eventually into the subarachnoid space .
Why Are the Other Options Incorrect?
Cisterna Magna
The cisterna magna (cerebellomedullary cistern) is a large subarachnoid space located beneath the cerebellum , where CSF collects after leaving the fourth ventricle .
It is not part of the pathway between the lateral and third ventricles .
Incorrect because it is located after the fourth ventricle, not between the lateral and third ventricles .
Foramen of Magendie
The foramen of Magendie is an exit point for CSF from the fourth ventricle into the cisterna magna (subarachnoid space).
It does not connect the lateral ventricles to the third ventricle .
Incorrect because it is involved in CSF drainage into the subarachnoid space, not ventricular communication .
Foramen of Luschka
The foramina of Luschka are paired lateral apertures that allow CSF to exit the fourth ventricle into the subarachnoid space .
Like the foramen of Magendie , these are outflow pathways, not ventricular connections .
Incorrect because they drain CSF into the subarachnoid space, not between the lateral and third ventricles .
Cerebral Aqueduct (Aqueduct of Sylvius)
The cerebral aqueduct connects the third ventricle to the fourth ventricle .
While it is an important part of CSF circulation , it is not involved in the passage of CSF from the lateral ventricles to the third ventricle .
Incorrect because it connects the third and fourth ventricles, not the lateral and third ventricles .
CSF Flow Pathway Recap
Lateral ventricles → Foramen of Monro → Third ventricle
Third ventricle → Cerebral aqueduct → Fourth ventricle
Fourth ventricle → Foramen of Magendie & Foramina of Luschka → Subarachnoid space & cisterna magna
CSF is reabsorbed via arachnoid granulations into the superior sagittal sinus
“This nucleus acts as the body’s ‘biological clock,’ receiving direct light input from the retina and regulating the sleep-wake cycle through melatonin secretion.”
47 / 128
Category:
NeuroSciences – Anatomy
Which of the following hypothalamic nuclei regulates circadian rhythms?
The hypothalamus plays a crucial role in regulating physiological processes, including circadian rhythms , which control the sleep-wake cycle, hormone release, and body temperature. One specific hypothalamic nucleus is known as the “master clock” of the body.
Correct Answer: Suprachiasmatic Nucleus (SCN)
Why is the Suprachiasmatic Nucleus (SCN) the Correct Answer?
The suprachiasmatic nucleus (SCN) is the primary regulator of circadian rhythms .
It receives direct input from retinal ganglion cells via the retinohypothalamic tract , allowing it to synchronize the body’s internal clock with light-dark cycles .
The SCN controls the release of melatonin from the pineal gland , regulating the sleep-wake cycle.
Lesions in the SCN disrupt normal circadian rhythms, leading to irregular sleep patterns and hormone dysregulation .
Why Are the Other Options Incorrect?
Supraoptic Nucleus
The supraoptic nucleus primarily produces antidiuretic hormone (ADH, vasopressin) , which regulates water balance.
It does not play a role in circadian rhythms.
Incorrect because it controls fluid balance, not sleep-wake cycles .
Ventromedial Nucleus
The ventromedial nucleus (VMN) is responsible for satiety and hunger regulation .
It does not control circadian rhythms but is involved in feeding behavior and metabolism .
Incorrect because it regulates food intake, not circadian cycles .
Arcuate Nucleus
The arcuate nucleus regulates hormone secretion, particularly appetite-controlling hormones (e.g., ghrelin, leptin, and neuropeptides) .
It also plays a role in dopamine-mediated inhibition of prolactin release .
Incorrect because it controls hormone regulation, not circadian rhythms .
Subthalamus
The subthalamic nucleus is part of the basal ganglia motor control circuit and is involved in movement regulation .
Lesions in the subthalamus cause hemiballismus , a movement disorder characterized by involuntary, flinging limb movements.
Incorrect because it is related to motor control, not circadian rhythm .
“This artery supplies the anterior two-thirds of a structure responsible for motor and sensory function. A thrombosis here leads to paraplegia with preserved proprioception.”
48 / 128
Category:
NeuroSciences – Anatomy
A 25-year-old man develops thrombosis of the anterior spinal artery. Which of the following regions will be affected the most?
The anterior spinal artery (ASA) is the primary blood supply to the anterior two-thirds of the spinal cord . A thrombosis in this artery leads to anterior spinal artery syndrome , which results in ischemia of the spinal cord .
Correct Answer: Spinal Cord
Why is the Spinal Cord the Correct Answer?
The anterior spinal artery (ASA) supplies the ventral (anterior) two-thirds of the spinal cord , including:
Corticospinal tract → Leads to bilateral motor weakness (paraplegia/quadriplegia) .
Spinothalamic tract → Causes bilateral loss of pain and temperature sensation .
Anterior horn cells (lower motor neurons) → Results in flaccid paralysis at the level of the lesion.
Autonomic pathways → Can lead to urinary and bowel dysfunction .
The dorsal columns (posterior third of the spinal cord) are spared because they are supplied by the posterior spinal arteries .
Clinical Features of Anterior Spinal Artery Syndrome
Bilateral lower motor neuron weakness at the level of the lesion (due to anterior horn involvement).
Bilateral upper motor neuron weakness below the lesion (due to corticospinal tract involvement).
Bilateral loss of pain and temperature sensation below the lesion (due to spinothalamic tract involvement).
Preserved vibration and proprioception (since the dorsal column is spared).
Why Are the Other Options Incorrect?
Midbrain
The midbrain is supplied by the posterior cerebral artery (PCA) and basilar artery , NOT the anterior spinal artery .
Incorrect because ASA does not supply the midbrain .
Pons
The pons is mainly supplied by the basilar artery (pontine branches), NOT the anterior spinal artery .
Incorrect because ASA does not provide the main blood supply to the pons .
Cerebellum
The cerebellum is supplied by the superior cerebellar artery (SCA), anterior inferior cerebellar artery (AICA), and posterior inferior cerebellar artery (PICA) .
The ASA does NOT supply the cerebellum , making this answer incorrect.
Medulla
The medulla is partially supplied by the anterior spinal artery , but its primary blood supply comes from the vertebral arteries and posterior inferior cerebellar artery (PICA) .
While a small part of the medulla (pyramids and medial lemniscus) may be affected, the spinal cord is the main structure affected in ASA thrombosis.
Incorrect because ASA thrombosis primarily causes spinal cord infarction rather than a significant medullary infarct.
“This neurotransmitter is responsible for reward and motivation. Excess activity in the mesolimbic pathway leads to hallucinations and delusions, while reduced activity in the mesocortical pathway results in cognitive impairment and apathy.”
49 / 128
Category:
Neurosciences – Pathology
What is the pathophysiology behind schizophrenia?
Schizophrenia is a neuropsychiatric disorder characterized by psychotic symptoms , including hallucinations, delusions, disorganized thought, and cognitive dysfunction. The pathophysiology is complex , but the dopamine hypothesis remains the most widely accepted explanation.
Correct Answer: Increased Dopamine
Why is Increased Dopamine the Correct Answer?
The dopamine hypothesis suggests that hyperactivity of dopaminergic pathways contributes to schizophrenia , particularly in the mesolimbic system .
Dopaminergic dysregulation in schizophrenia:
Increased dopamine activity in the mesolimbic pathway → Positive symptoms (hallucinations, delusions, paranoia).
Decreased dopamine activity in the mesocortical pathway → Negative symptoms (social withdrawal, flat affect, cognitive impairment).
Antipsychotic medications (e.g., haloperidol, risperidone, clozapine) work by blocking dopamine D2 receptors , reducing excessive dopamine transmission .
Why Are the Other Options Incorrect?
Decreased Acetylcholine
Acetylcholine is involved in memory and cognition , and while cholinergic dysfunction may play a role in cognitive symptoms , it is not the primary cause of schizophrenia .
Incorrect because the main neurotransmitter imbalance is dopamine, not acetylcholine .
Increased Glutamate
The glutamate hypothesis suggests that hypofunction of NMDA receptors (glutamate receptors) may contribute to schizophrenia.
However, decreased glutamate activity , rather than increased, is more commonly implicated in schizophrenia.
Incorrect because hypoactivity of glutamate, not hyperactivity, is more likely involved .
Decreased Serotonin
While serotonin is involved in mood and cognition , schizophrenia is primarily a dopamine-related disorder .
Atypical antipsychotics (e.g., clozapine) block both dopamine and serotonin (5-HT2A) receptors , suggesting some involvement, but dopamine is still the primary neurotransmitter .
Incorrect because serotonin plays a secondary role compared to dopamine .
None of These
Since dopamine dysregulation is a well-established cause of schizophrenia , this option is incorrect.
“This cerebellar zone is responsible for fine-tuning limb movements. Damage to it leads to difficulty in performing rapid alternating movements, such as pronation-supination of the hands.”
50 / 128
Category:
NeuroSciences – Anatomy
Which of the following symptoms will be the result of a lesion of the intermediate zone of the cerebellum?
The cerebellum is divided into functional zones, each responsible for different aspects of motor control. The intermediate zone (paravermis) plays a key role in coordinating limb movements , particularly distal limb control and motor execution .
Correct Answer: Dysdiadochokinesia
Why is Dysdiadochokinesia the Correct Answer?
Dysdiadochokinesia refers to impaired ability to perform rapid alternating movements (e.g., pronation-supination of the hands, foot tapping).
The intermediate zone of the cerebellum (paravermal region) is responsible for coordinating limb movements , particularly through its connections with:
Lateral corticospinal tract (for distal limb control).
Rubrospinal tract (for fine motor adjustments).
Lesions in this area lead to limb ataxia , intention tremors , and dysdiadochokinesia , all signs of impaired motor coordination.
Why Are the Other Options Incorrect?
Vertigo
Vertigo is a symptom of vestibular dysfunction , which is primarily associated with lesions of the flocculonodular lobe (vestibulocerebellum) .
Since the intermediate zone does not regulate balance via the vestibular system , vertigo is incorrect.
Nystagmus
Nystagmus (involuntary rhythmic eye movements) is also associated with damage to the vestibulocerebellum (flocculonodular lobe) .
Since the intermediate zone does not directly control eye movements , this is incorrect.
Dysarthria
Dysarthria (slurred or impaired speech) occurs due to lesions in the cerebellar vermis , particularly affecting the fastigial nucleus , which controls axial and speech muscles.
The intermediate zone is more involved in limb coordination , so dysarthria is incorrect.
Resting Tremor
Resting tremor is a hallmark of Parkinson’s disease , caused by dysfunction in the basal ganglia (substantia nigra) .
Cerebellar lesions typically cause intention tremors (which occur during movement), not resting tremors.
Incorrect because the cerebellum does not cause resting tremors .
“This type of infarct occurs in the most vulnerable areas of the brain when overall blood pressure changes. Think about what happens to brain tissue at the junction of two arteries when perfusion is inadequate.”
51 / 128
Category:
Neurosciences – Pathology
Which of the following is correct regarding a watershed infarct?
Whenever answering pathology questions, always prioritize the root cause (etiology) over a specific manifestation unless the question is explicitly asking for anatomical locations!
A watershed infarct (border zone infarct) occurs in regions of the brain that are supplied by the distal ends of two major arteries , making them particularly vulnerable to hypoperfusion . These infarcts are commonly seen after episodes of hypotension or shock , which reduce blood flow to these “border zones” .
Correct Answer: It is Seen After Hypotensive Episodes
Why is “It is Seen After Hypotensive Episodes” the Correct Answer?
Watershed infarcts occur due to global hypoperfusion rather than embolic or thrombotic occlusions of major arteries.
Common causes include:
Severe hypotension (e.g., cardiac arrest, shock, prolonged hypotension).
Hypovolemic states (e.g., dehydration, hemorrhage).
Severe carotid artery stenosis leading to decreased perfusion to distal vascular territories.
Since watershed areas are at the junctions of two arterial supplies , they receive less blood when overall cerebral perfusion is reduced.
Why Are the Other Options Incorrect?
“It May Occur at the Border Zone Between Anterior and Middle Cerebral Arteries”
While this statement is partially true , watershed infarcts can also occur between the middle and posterior cerebral arteries .
Since the primary cause is hypotension , the more precise and fundamental answer is “it is seen after hypotensive episodes.”
Most common watershed areas affected:
Cortical watershed infarcts – between anterior cerebral artery (ACA) and middle cerebral artery (MCA) or MCA and posterior cerebral artery (PCA) .
Subcortical watershed infarcts – in the deep white matter between the lateral striate and anterior choroidal arteries.
Incorrect because a broader explanation (hypotension) is a better descriptor.
“It Occurs When There Is Liquefactive Necrosis”
While liquefactive necrosis does occur in all types of cerebral infarcts , it is not specific to watershed infarcts .
All brain infarcts lead to liquefactive necrosis , so this is not the defining characteristic of a watershed infarct.
Incorrect because watershed infarcts are defined by their cause (hypoperfusion), not the type of necrosis .
“A Paradoxical Occurrence in Which Ischemic Area Becomes Infused with CSF and Life-Threatening Cerebral Edema Ensues”
This describes “malignant infarction” due to major artery occlusion, which can lead to brain swelling and herniation , particularly in MCA infarcts .
Watershed infarcts are caused by hypoperfusion rather than a massive occlusion with CSF accumulation .
Incorrect because watershed infarcts do not involve CSF infusion or paradoxical cerebral edema .
“It Occurs Due to Hyperviscosity of Blood”
Hyperviscosity syndromes (e.g., polycythemia vera, multiple myeloma) can lead to small-vessel occlusions , but they are not a common cause of watershed infarcts .
Watershed infarcts are caused by systemic hypotension , not blood hyperviscosity.
Incorrect because hypoperfusion, not increased viscosity, is the primary cause .
“This transporter is responsible for delivering glucose to the brain in an insulin-independent manner. A deficiency in this transporter leads to seizures due to low glucose in cerebrospinal fluid.”
52 / 128
Category:
Neurosciences – Biochemistry
Glucose enters the brain through which of the following transporters?
Glucose is the primary energy source for the brain, and it must cross the blood-brain barrier (BBB) to reach neurons and glial cells. This transport occurs via facilitated diffusion , which requires specific glucose transporters (GLUTs ).
Correct Answer: GLUT-1
Why is GLUT-1 the Correct Answer?
GLUT-1 is the primary glucose transporter responsible for moving glucose across the blood-brain barrier (BBB) and into astrocytes and endothelial cells .
It is an insulin-independent transporter, meaning glucose uptake into the brain is not regulated by insulin (unlike GLUT-4 in muscle and adipose tissue).
GLUT-1 is highly expressed in:
Endothelial cells of the BBB – allowing glucose entry into the brain.
Astrocytes – which help shuttle glucose to neurons.
Deficiency of GLUT-1 leads to Glucose Transporter Type 1 Deficiency Syndrome , characterized by hypoglycorrhachia (low CSF glucose), seizures, developmental delay, and microcephaly .
Why Are the Other Options Incorrect?
GLUT-4
GLUT-4 is the insulin-dependent glucose transporter found in skeletal muscle, cardiac muscle, and adipose tissue .
It does not play a role in glucose transport into the brain.
Incorrect because the brain does not rely on insulin-dependent glucose uptake .
GLUT-2
GLUT-2 is a bidirectional glucose transporter found in the liver, pancreas, kidney, and intestines .
It is important for glucose sensing and regulation of insulin secretion , but not for glucose transport across the BBB .
Incorrect because it is not present in the blood-brain barrier .
GLUT-13 (HMIT – H+/myo-inositol transporter)
GLUT-13 (also called HMIT) is not primarily a glucose transporter; it transports myo-inositol , an important osmolyte in the brain.
Incorrect because it does not transport glucose .
SGLT-1 (Sodium-Glucose Cotransporter 1)
SGLT-1 is a sodium-dependent glucose transporter found in the small intestine and renal proximal tubules , where it absorbs glucose via active transport .
Unlike GLUT transporters, which use facilitated diffusion , SGLT-1 requires sodium gradients .
Incorrect because SGLT-1 is not present in the BBB or brain tissue .
“This vein is responsible for draining deep brain structures, including the thalamus and choroid plexus. What two tributaries would best serve this function?”
53 / 128
Category:
NeuroSciences – Anatomy
The internal cerebral vein is formed by the union of which of the following?
The internal cerebral vein (ICV) is a major deep venous structure of the brain that drains deep brain structures, including the thalamus, basal ganglia, and deep white matter . It is formed by the union of two main veins in the deep brain.
Correct Answer: Thalamostriate and Choroidal Veins
Why Are the Thalamostriate and Choroidal Veins the Correct Answer?
The internal cerebral vein is formed at the level of the interventricular foramen (of Monro) by the junction of:
Thalamostriate vein – drains the thalamus and caudate nucleus .
Choroidal vein – drains the choroid plexus of the lateral ventricles .
Once formed, the internal cerebral veins travel posteriorly, merge at the velum interpositum , and join with the basal veins of Rosenthal to form the great cerebral vein (of Galen) .
This venous system is crucial for draining deep brain structures into the straight sinus .
Why Are the Other Options Incorrect?
Superficial and Deep Middle Cerebral Vein
These veins are part of the superficial venous system , not the deep system.
The superficial middle cerebral vein drains lateral cortical areas into the cavernous sinus , while the deep middle cerebral vein contributes to the basal vein of Rosenthal , not the internal cerebral vein.
Incorrect because these veins do not form the ICV .
Anterior Cerebral Vein and Striate Vein
The anterior cerebral vein is part of the superficial system and does not contribute to the internal cerebral vein.
The striate veins drain the basal ganglia but do not directly form the ICV.
Incorrect because these veins do not join to form the ICV .
Superior Cerebral Vein and the Great Cerebral Vein
Superior cerebral veins drain into the superior sagittal sinus .
The great cerebral vein (of Galen) is actually formed by the internal cerebral veins , rather than contributing to them.
Incorrect because these veins do not form the ICV .
Superficial Cerebral Vein and the Basal Vein
Superficial cerebral veins drain into the dural venous sinuses , not the deep venous system.
The basal vein of Rosenthal joins the internal cerebral vein to form the great cerebral vein (of Galen) , but it does not form the ICV itself .
Incorrect because these veins are not the primary contributors to ICV formation .
“This enzyme plays a key role in neurotransmission at cholinergic synapses. “
54 / 128
Category:
Neurosciences – Biochemistry
What is the function of acetylcholinesterase?
Acetylcholinesterase (AChE) is an enzyme responsible for the breakdown of acetylcholine (ACh) , a crucial neurotransmitter in both the central nervous system (CNS) and peripheral nervous system (PNS) . The enzyme is essential for terminating synaptic transmission at cholinergic synapses , particularly in neuromuscular junctions, autonomic ganglia, and the brain .
Correct Answer: To Hydrolyze Acetylcholine
Why is “To Hydrolyze Acetylcholine” the Correct Answer?
Acetylcholinesterase catalyzes the hydrolysis (breakdown) of acetylcholine into choline and acetate , thereby terminating neurotransmission .
This breakdown is necessary to prevent prolonged stimulation of cholinergic receptors, which could lead to continuous muscle contraction or excessive neural activity.
The reaction is as follows:
Acetylcholine+H2O→Choline+Acetate
AChE is a target of nerve agents and organophosphate insecticides , which inhibit the enzyme, leading to toxic accumulation of acetylcholine and excessive cholinergic activity (e.g., muscle paralysis, bradycardia, respiratory failure).
Why Are the Other Options Incorrect?
To Esterify Acetylcholine
Esterification is a process of forming an ester bond , but AChE breaks down acetylcholine rather than synthesizing it.
Incorrect because AChE does not esterify acetylcholine ; it hydrolyzes it.
To Synthesize Acetylcholine
Acetylcholine synthesis is carried out by choline acetyltransferase (ChAT) , not acetylcholinesterase.
ChAT catalyzes the reaction:Choline+Acetyl-CoA→Acetylcholine+CoA Choline + Acetyl-CoA → Acetylcholine + CoA
Incorrect because AChE degrades acetylcholine, not synthesizes it.
To Add an Acetyl Group to Choline
This function is performed by choline acetyltransferase , not AChE.
Incorrect because AChE does not add acetyl groups ; it hydrolyzes acetylcholine.
To Add an Ester Group to Choline
Acetylcholine itself is an ester , but AChE does not add ester groups ; it removes them by hydrolysis .
Incorrect because AChE breaks down esters rather than adding them.
“A person with a cerebellar lesion tends to fall toward the side of the lesion. Which side of the body would be affected if the right cerebellar hemisphere were damaged?”
55 / 128
Category:
NeuroSciences – Anatomy
Which of the following clinical features will be seen in a lesion of the cerebellar hemisphere?
The cerebellar hemispheres are primarily responsible for coordinating voluntary movements of the ipsilateral limbs . A lesion in one hemisphere results in ipsilateral motor deficits , including tremors, ataxia, dysmetria, and hypotonia.
Correct Answer: Person Falls to the Right
Why is “Person Falls to the Right” the Correct Answer?
Cerebellar hemispheric lesions cause ipsilateral deficits because cerebellar output affects motor pathways that decussate twice (once in the cerebellum and again in the corticospinal or rubrospinal tracts).
If the right cerebellar hemisphere is damaged, the patient will exhibit poor coordination and balance on the right side and is likely to fall to the right .
Patients with cerebellar lesions commonly experience ipsilateral ataxia, dysmetria, and difficulty maintaining balance while standing or walking .
Why Are the Other Options Incorrect?
Negative Romberg Sign
The Romberg test is used to assess proprioception , not cerebellar function.
A positive Romberg sign (loss of balance when the eyes are closed) suggests sensory ataxia due to dorsal column dysfunction (e.g., vitamin B12 deficiency, tabes dorsalis).
Cerebellar ataxia occurs regardless of vision , so the Romberg test is typically negative in cerebellar lesions.
Incorrect because cerebellar dysfunction does not affect proprioception .
Tremors in the Right Hand
While cerebellar lesions can cause intention tremors , they occur on the ipsilateral side of the lesion.
If the lesion is in the left cerebellar hemisphere , tremors will be in the left hand .
Incorrect because the question does not specify which cerebellar hemisphere is affected.
Swaying Left to Right with Eyes Closed
A patient with a cerebellar lesion sways even with eyes open .
If the Romberg sign were positive , it would indicate sensory ataxia , not cerebellar dysfunction.
Incorrect because cerebellar ataxia is not dependent on visual input .
Decreased Muscle Tone in the Right Upper and Lower Limbs
Cerebellar lesions can cause hypotonia , but isolated hypotonia without ataxia is uncommon .
The primary symptoms of cerebellar hemisphere lesions are coordination deficits, not just muscle tone reduction .
Incorrect because hypotonia alone is not a primary diagnostic feature .
“This structure in the dorsal horn of the spinal cord is responsible for processing pain before sending the signal across to the opposite side of the spinal cord and up to the brain.”
56 / 128
Category:
NeuroSciences – Anatomy
Which structure acts as the second-order neuron for the spinothalamic tract?
The spinothalamic tract is responsible for transmitting pain, temperature, and crude touch from the body to the brain. It is part of the anterolateral system (ALS) and follows a three-neuron relay system :
First-order neurons – Sensory neurons in the dorsal root ganglion (DRG) detect pain and temperature and send signals to the spinal cord .
Second-order neurons – Located in the dorsal horn of the spinal cord (specifically in the substantia gelatinosa and nucleus proprius) , where they decussate (cross over) via the anterior white commissure and ascend in the spinothalamic tract to the thalamus.
Third-order neurons – Located in the ventral posterior lateral (VPL) nucleus of the thalamus , which projects to the primary somatosensory cortex (postcentral gyrus) .
Correct Answer: Substantia Gelatinosa
Why is the Substantia Gelatinosa the Correct Answer?
The substantia gelatinosa (Rexed lamina II) is a key relay center in the dorsal horn of the spinal cord where pain and temperature signals are processed.
It is part of the second-order neuron system , which:
Receives first-order sensory input.
Relays pain and temperature signals.
Decussates across the anterior white commissure before ascending in the spinothalamic tract .
Damage to this region leads to contralateral pain and temperature loss below the lesion due to early decussation in the spinal cord.
Why Are the Other Options Incorrect?
Nucleus Gracilis
The nucleus gracilis is part of the dorsal column-medial lemniscus (DCML) pathway , responsible for fine touch, vibration, and proprioception from the lower body .
It has no role in pain or temperature transmission , making it incorrect.
Nucleus Cuneatus
The nucleus cuneatus is also part of the DCML pathway , handling fine touch and proprioception from the upper body .
It is not involved in the spinothalamic tract and is incorrect.
Inferior Colliculus
The inferior colliculus is part of the auditory pathway , processing sound localization from the cochlear nuclei.
It has no involvement in pain, temperature, or sensory pathways of the spinothalamic tract , making it incorrect.
Nucleus Dorsalis (Clarke’s Column)
Clarke’s column (nucleus dorsalis) is part of the spinocerebellar tract , which carries unconscious proprioception to the cerebellum.
It is not involved in pain or temperature transmission , making it incorrect.
“This condition causes a breakdown of the blood-brain barrier, allowing plasma proteins to leak into the CSF. It also leads to an increase in WBC count and a decrease in glucose levels.”
57 / 128
Category:
Neurosciences – Pathology
Increased protein in cerebrospinal fluid occurs in which of the following conditions?
Cerebrospinal fluid (CSF ) analysis is a crucial diagnostic tool for evaluating neurological conditions . Increased protein levels in CSF typically indicate inflammation, infection, or a disruption of the blood-brain barrier .
Correct Answer: Bacterial Infection
Why is bacterial infection the correct answer?
Bacterial infections , such as bacterial meningitis , lead to a marked increase in CSF protein due to:
Breakdown of the blood-brain barrier .
Inflammatory response and leakage of plasma proteins into CSF.
Increased immune cell activity producing proteins.
CSF findings in bacterial meningitis typically show:
High protein (>100 mg/dL, often >250 mg/dL)
Low glucose (<40 mg/dL, or <50% of blood glucose)
High WBC count (neutrophilic predominance)
Elevated opening pressure on lumbar puncture
Why Are the Other Options Incorrect?
Increased Intracranial Pressure (ICP)
Elevated ICP does not directly increase CSF protein levels .
While ICP may result from brain tumors, hydrocephalus, or hemorrhage , CSF protein levels remain relatively normal unless accompanied by an inflammatory or infectious process .
Incorrect because ICP does not independently increase protein concentration .
Repeated Lumbar Puncture
Multiple lumbar punctures may lead to trauma-induced CSF contamination with blood , but this does not cause a true pathological increase in CSF protein .
Protein elevation may be seen if there is trauma to blood vessels , but this is not a primary cause of elevated CSF protein.
Incorrect because it does not consistently result in increased CSF protein.
Acute Water Intoxication
Water intoxication leads to hyponatremia and brain edema, but it does not cause increased CSF protein levels.
The BBB remains intact unless severe brain swelling causes hemorrhagic damage.
Incorrect because water intoxication affects osmolarity, not protein levels in CSF .
None of These
Since bacterial infection is a valid cause of increased CSF protein, this option is incorrect.
“Think about where fine touch and proprioception signals first relay before crossing to the opposite side. These medullary structures are named after body regions they serve.”
58 / 128
Category:
NeuroSciences – Anatomy
Which structure(s) acts as the second-order neuron of the dorsal column?
The dorsal column-medial lemniscus (DCML) pathway is responsible for transmitting fine touch, vibration, conscious proprioception, and pressure sensations from the body to the brain. This pathway involves three neurons :
First-order neurons – sensory neurons in the dorsal root ganglia (DRG) that ascend ipsilaterally in the spinal cord via the fasciculus gracilis and fasciculus cuneatus to reach the medulla.
Second-order neurons – located in the nucleus gracilis and nucleus cuneatus of the medulla , where they decussate (cross over) and form the medial lemniscus .
Third-order neurons – located in the ventral posterior lateral (VPL) nucleus of the thalamus , which project to the primary somatosensory cortex (postcentral gyrus).
Correct Answer: Nucleus Cuneatus and Nucleus Gracilis
Why are the nucleus cuneatus and nucleus gracilis the correct answer?
These nuclei serve as the second-order neurons in the DCML pathway .
They receive input from first-order neurons carrying fine touch, vibration, and proprioception .
After synapsing here, second-order neurons decussate (cross) at the medulla and ascend via the medial lemniscus toward the thalamus.
Why Are the Other Options Incorrect?
Clarke’s Column (Nucleus Dorsalis)
Clarke’s column is involved in the spinocerebellar pathway , which transmits unconscious proprioception to the cerebellum .
It does not serve as a second-order neuron for the dorsal column pathway .
Incorrect because it is part of the spinocerebellar tract, not the DCML .
Suprachiasmatic Nucleus (SCN)
The SCN is the master circadian rhythm regulator in the hypothalamus.
It has no role in sensory transmission or the dorsal column pathway .
Incorrect because it is responsible for sleep-wake cycles, not somatosensation .
Nucleus Dorsalis
This is another name for Clarke’s column , which belongs to the spinocerebellar tract , not the DCML pathway .
Incorrect for the same reason as Clarke’s column.
Superior Colliculus
The superior colliculus is part of the midbrain and is involved in visual reflexes and eye movements , particularly in orienting the head toward stimuli.
It has no role in the dorsal column pathway .
Incorrect because it is a visual processing center, not a sensory relay for touch and proprioception .
“This process involves degeneration occurring in a specific direction relative to the site of injury. Would a neuron break down toward or away from the cell body when an axon is severed?”
59 / 128
Category:
Neurosciences – Pathology
Which statement is incorrect about Wallerian degeneration?
Wallerian degeneration is the process of axonal degeneration and clearance that occurs distal to an injury when a nerve fiber is damaged. It occurs in both the peripheral nervous system (PNS) and central nervous system (CNS) but has different regenerative capacities in each system.
Correct Answer: “It is Retrograde” (Incorrect Statement)
Why is “It is Retrograde” Incorrect?
Wallerian degeneration is an anterograde (distal) process, NOT retrograde.
When an axon is injured, degeneration occurs distal to the site of injury , meaning it progresses away from the cell body (anterograde direction).
Retrograde changes, also known as chromatolysis , may occur in the neuron’s cell body , but Wallerian degeneration itself is strictly anterograde .
Why Are the Other Statements Correct?
“It occurs in the peripheral nervous system (PNS)” ✅ (Correct)
In the PNS , Wallerian degeneration allows for axon regeneration due to the presence of Schwann cells , which provide a regenerative environment by secreting neurotrophic factors .
“The axon and the myelin sheath disappear” ✅ (Correct)
After an injury, the axon degenerates distal to the lesion , and the myelin sheath breaks down .
Macrophages and Schwann cells clear the debris in the PNS, while microglia perform this function in the CNS.
“It occurs in the central nervous system (CNS)” ✅ (Correct)
Wallerian degeneration does occur in the CNS, but axon regeneration is very limited due to the presence of oligodendrocytes and inhibitory factors (e.g., Nogo proteins, astrocyte scarring).
Unlike the PNS, where Schwann cells promote regeneration, oligodendrocytes do not support axon regrowth.
“Schwann cells proliferate” ✅ (Correct)
In the PNS , Schwann cells proliferate and create a growth-promoting environment , forming bands of Büngner , which guide the regenerating axon.
This is why peripheral nerves have better regeneration potential than central nerves .
“This neuron does not cause direct muscle contraction but instead fine-tunes sensory feedback by maintaining muscle spindle tension. It works alongside alpha motor neurons in regulating movement and reflexes.”
60 / 128
Category:
Neurosciences – Physiology
Which of the following is a neuron that forms synaptic junctions with intrafusal skeletal muscle fibers?
The intrafusal skeletal muscle fibers are specialized muscle fibers found within the muscle spindle , a sensory receptor involved in detecting muscle stretch and proprioception . The neuron that forms synaptic junctions with these fibers plays a critical role in modulating muscle spindle sensitivity.
Correct Answer: Gamma Motor Neuron
Why is the Gamma Motor Neuron the Correct Answer?
Gamma motor neurons innervate the intrafusal muscle fibers of the muscle spindle.
Their function is to adjust the sensitivity of the muscle spindle to stretch , ensuring that it remains responsive across different muscle lengths.
They maintain muscle spindle tension , allowing continued proprioceptive feedback even when the muscle is contracted.
This helps regulate muscle tone and reflex activity .
Why Are the Other Options Incorrect?
Purkinje Cell
Purkinje cells are large inhibitory neurons found in the cerebellum .
They do not synapse on muscle fibers but rather modulate motor coordination and balance .
Incorrect because Purkinje cells are part of the cerebellar cortex, not the muscle spindle system .
Granule Cell
Granule cells are the smallest neurons in the cerebellum and play a role in excitatory synapses within the cerebellar cortex .
They do not project to muscle fibers.
Incorrect because granule cells are involved in cerebellar processing, not direct motor control .
Alpha Motor Neuron
Alpha motor neurons innervate extrafusal muscle fibers , which are responsible for voluntary muscle contraction .
While both alpha and gamma motor neurons are involved in motor function, only gamma motor neurons control intrafusal fibers .
Incorrect because alpha motor neurons control force production, not spindle sensitivity .
Pyramidal Cell
Pyramidal cells are large projection neurons in the cerebral cortex (motor cortex), responsible for initiating voluntary movement via the corticospinal tract .
They do not directly innervate muscle fibers.
Incorrect because pyramidal cells control higher-level motor commands, not direct muscle spindle activation .
🔵 Imagine a machine with ropes pulling heavy weights. What kind of safety mechanism would you want to prevent the ropes from snapping if the load gets too heavy — a device that monitors stretch, pressure, force, or something else? Think about the consequences of excessive tension.
61 / 128
Category:
Neurosciences – Physiology
What is the function of the Golgi tendon organ?
The Golgi tendon organ (GTO) is a proprioceptive sensory receptor located at the junction between muscles and tendons. Its primary function is to monitor muscle tension and prevent excessive force generation that could lead to muscle or tendon damage.
Correct Answer: Prevent Extreme Contraction
Why is Preventing Extreme Contraction the Correct Answer?
Golgi tendon organs are activated when a muscle generates excessive force.
They send inhibitory signals via Ib afferent fibers to the spinal cord, which activates interneurons that inhibit alpha motor neurons supplying the contracting muscle.
This leads to muscle relaxation , preventing tendon rupture or excessive contraction .
This is known as the Golgi tendon reflex (inverse myotatic reflex) or autogenic inhibition .
Why Are the Other Options Incorrect?
Damage the Contractile Forces
The GTO does not damage contractile forces ; rather, it regulates muscle tension to prevent injury .
Incorrect because GTOs are protective, not destructive.
Equalize Forces Between Individual Muscle Fibers
Muscle fibers within a single muscle do not need force equalization via the GTO.
The muscle spindle is more involved in fine-tuning individual fiber tension rather than the GTO.
Incorrect because GTOs regulate overall muscle force, not individual fiber forces.
Inhibit the Force
While the GTO does inhibit excessive force, this option is too vague and does not clearly explain its primary function (preventing excessive contraction).
Incorrect because it does not specify the protective role of the GTO in preventing damage .
Nullify the Contractile Forces
The GTO does not completely nullify contraction but rather modulates it to prevent injury.
Incorrect because muscles still contract, just in a regulated manner.
“If you touch a hot object or experience a sharp pain, your body reacts quickly. This sensation must travel through a fast, direct pathway to the brain. Which spinal tract carries this information?”
62 / 128
Category:
NeuroSciences – Anatomy
Pain and temperature sensations are carried by which of the following?
The nervous system has specialized pathways for transmitting different types of sensory information. Pain and temperature sensations specifically follow a well-defined tract in the spinal cord that ultimately projects to the brain for perception.
Correct Answer: Lateral Spinothalamic Tract
Why is the Lateral Spinothalamic Tract the Correct Answer?
The lateral spinothalamic tract is responsible for transmitting:
Pain (nociception)
Temperature (thermoreception)
Crude touch (to some extent, along with the anterior spinothalamic tract)
The pathway follows a three-neuron relay system :
First-order neurons : Sensory receptors in the skin detect pain or temperature and send signals through dorsal root ganglia into the spinal cord.
Second-order neurons : These neurons are located in the dorsal horn of the spinal cord and immediately decussate (cross to the opposite side) via the anterior white commissure before ascending in the lateral spinothalamic tract .
Third-order neurons : Located in the ventral posterior lateral (VPL) nucleus of the thalamus , they relay information to the somatosensory cortex (postcentral gyrus).
This decussation means that damage to the lateral spinothalamic tract on one side results in contralateral (opposite side) pain and temperature deficits below the lesion.
Why Are the Other Options Incorrect?
Rubrospinal Pathway
This tract originates in the red nucleus of the midbrain and is involved in motor control, particularly of limb flexors .
It does not carry sensory information , making this answer incorrect.
Dorsal Column
The dorsal column-medial lemniscus (DCML) pathway transmits:
Fine touch (tactile sensation)
Vibration
Conscious proprioception
It does not carry pain or temperature signals , making this answer incorrect.
Anterior Spinothalamic Tract
The anterior spinothalamic tract carries crude touch and pressure , not pain and temperature.
While both the anterior and lateral spinothalamic tracts contribute to the anterolateral system , pain and temperature are specifically carried by the lateral spinothalamic tract .
Spinocerebellar Pathway
This pathway is responsible for unconscious proprioception , helping to coordinate movement via input to the cerebellum .
It does not carry pain or temperature information , making it an incorrect answer.
“The brain needs protection from harmful substances while still allowing essential nutrients in. This barrier is not a single structure but a combination of different elements working together. What could these be?”
63 / 128
Category:
Neurosciences – Histology
The blood-brain barrier is formed by which of the following?
The blood-brain barrier (BBB) is a highly selective barrier that protects the brain from toxins, pathogens, and fluctuations in systemic blood composition while allowing essential nutrients to pass through. It is composed of multiple structures working together to maintain CNS homeostasis.
Correct Answer: All of These
Why is “All of These” the Correct Answer?
The BBB is formed by several key components :
Tight Junctions Between Capillary Endothelial Cells
The capillary endothelial cells in the brain have tight junctions (zonula occludens), preventing the paracellular movement of substances.
These junctions force molecules to pass through the endothelial cells (transcellular transport ) instead of between them, creating a highly selective barrier .
Pericytes Present Around the Capillaries
Pericytes are contractile cells that wrap around capillaries , regulating blood flow and barrier integrity .
They help control the permeability of the BBB and play a role in the immune response within the CNS.
The Impermeable, Non-Fenestrated, Basement Membrane of the Capillaries
The capillary basement membrane is continuous and non-fenestrated , acting as a mechanical and biochemical barrier that prevents large molecules and pathogens from entering the CNS.
Pseudopods of the Astrocytes
Astrocyte foot processes (pseudopods) surround the capillaries and release signals that maintain and regulate the integrity of the BBB .
They help in nutrient transport , repair, and modulation of neurovascular coupling .
Since all of these components collectively contribute to the BBB , the correct answer is “All of These.”
Why Are the Other Individual Options Incorrect on Their Own?
Each option represents an essential component of the BBB, but the barrier is not formed by just one of them alone —it is a combination of all these structures that provides the selective permeability characteristic of the BBB.
“This syndrome results from damage to a brain structure that plays a crucial role in fear processing and emotional regulation. Think about what would happen if someone completely lost their fear response.”
64 / 128
Category:
Neurosciences – Pathology
Hyperorality, hypersexuality, and decreased emotions are all associated with which syndrome?
The combination of hyperorality (excessive oral fixation), hypersexuality, and decreased emotional response strongly suggests a disorder involving the amygdala , which is responsible for processing emotions, fear, and behavior regulation.
Correct Answer: Klüver-Bucy Syndrome
Why is Klüver-Bucy syndrome the correct answer?
Klüver-Bucy syndrome results from bilateral lesions of the amygdala , leading to:
Hyperorality – excessive exploration of objects with the mouth.
Hypersexuality – inappropriate sexual behaviors.
Docility and emotional blunting – reduced fear and aggression.
Memory disturbances – difficulty recognizing familiar objects (visual agnosia).
It is often caused by:
Herpes simplex virus (HSV-1) encephalitis – a common infectious cause.
Trauma, stroke, or neurodegenerative diseases affecting the temporal lobes .
Why Are the Other Options Incorrect?
Dandy-Walker Syndrome
A congenital brain malformation affecting the cerebellum and fourth ventricle .
Symptoms include hydrocephalus, ataxia, and developmental delay but not behavioral changes like hyperorality and hypersexuality .
Incorrect because it primarily affects motor coordination and not the amygdala.
Guillain-Barré Syndrome (GBS)
An autoimmune peripheral nerve disorder causing ascending muscle weakness and paralysis .
No involvement of the amygdala, and it does not cause behavioral abnormalities.
Incorrect because GBS affects the peripheral nervous system, not behavior or emotions .
Wernicke-Korsakoff Syndrome
Caused by thiamine (Vitamin B1) deficiency , commonly in chronic alcoholics.
Symptoms include confusion, ataxia, ophthalmoplegia (Wernicke’s encephalopathy), and memory deficits with confabulation (Korsakoff syndrome) .
Incorrect because it affects memory and coordination rather than causing hyperorality or hypersexuality .
Thalamic Pain Syndrome (Dejerine-Roussy Syndrome)
Occurs after a stroke affecting the thalamus , leading to chronic neuropathic pain and sensory disturbances .
It does not cause hypersexuality, hyperorality, or emotional blunting .
Incorrect because it affects pain perception, not behavioral responses.
“This condition results from the failure of a major white matter structure to develop properly, leading to widely spaced frontal horns of the lateral ventricles, creating the characteristic ‘bat-wing’ appearance on imaging.”
65 / 128
Category:
NeuroSciences – Anatomy
The brain magnetic resonance image (MRI) of an 8-year-old child shows a bat-wing deformity. Which condition is the most likely cause of this?
A bat-wing deformity on brain MRI refers to the widening of the lateral ventricles , particularly in the region of the frontal horns, which gives the appearance of bat wings. This radiological finding is classically associated with agenesis of the corpus callosum (ACC) , a condition where the corpus callosum fails to develop properly.
Correct Answer: Agenesis of Corpus Callosum
Why is agenesis of the corpus callosum the correct answer?
The corpus callosum is a major white matter structure connecting the two cerebral hemispheres.
In ACC , the absence of this structure leads to enlargement and abnormal separation of the lateral ventricles , resulting in the characteristic bat-wing appearance on MRI.
This condition can be isolated or associated with genetic disorders like Aicardi syndrome or Andermann syndrome .
Clinical features of ACC may include:
Developmental delay
Intellectual disability
Impaired motor coordination (dyspraxia)
Vision and speech impairments
Seizures (in severe cases)
Why Are the Other Options Incorrect?
Lissencephaly
Lissencephaly is a neuronal migration disorder where the brain surface lacks normal folds, appearing smooth .
MRI shows pachygyria (broad, thick gyri) or agyria (absence of gyri) , not the bat-wing deformity .
Incorrect because lissencephaly is associated with abnormal cortical development, not agenesis of the corpus callosum .
Holoprosencephaly
Holoprosencephaly is a midline brain defect where the forebrain fails to divide properly into two hemispheres.
MRI shows fused lateral ventricles, absent interhemispheric fissure, and abnormal midline structures .
Incorrect because bat-wing deformity is caused by ventricular separation, not fusion .
Polymicrogyria
Polymicrogyria is characterized by excessive small, disorganized gyri , leading to an irregular cortical surface.
MRI shows abnormal cortical folding patterns , but not ventricular enlargement or separation .
Incorrect because polymicrogyria affects the cortex, not the corpus callosum.
Neuronal Heterotopia
This is a neuronal migration disorder where neurons fail to reach their proper cortical position, remaining in ectopic locations .
MRI typically shows gray matter nodules in abnormal locations (periventricular or subcortical) , but not ventricular enlargement .
Incorrect because it does not cause the bat-wing appearance .
“Think about a structured tool that is widely used in emergency settings to determine whether a patient is alert, drowsy, or in a coma. It assigns numerical scores based on specific responses.”
66 / 128
Category:
Neurosciences – Physiology
How is the level of consciousness of a patient tested?
Assessing a patient’s level of consciousness (LOC) is a crucial step in neurological evaluation. It helps determine the severity of brain dysfunction, especially in cases of head trauma, stroke, or metabolic disturbances. The Glasgow Coma Scale (GCS) is the gold standard for assessing consciousness.
Correct Answer: Glasgow Coma Scale (GCS)
Why is the Glasgow Coma Scale the correct answer?
The Glasgow Coma Scale (GCS) objectively measures a patient’s level of consciousness based on three components:
Eye Opening Response (E)
Verbal Response (V)
Motor Response (M)
Scores range from 3 to 15 :
Severe brain injury (coma): GCS ≤ 8
Moderate brain injury: GCS 9-12
Mild brain injury: GCS 13-15
It is commonly used in emergency settings, head injuries, and critical care monitoring.
Why Are the Other Options Incorrect?
Higher Mental Function
Higher mental function tests (e.g., memory, attention, judgment, and abstract thinking) are useful for assessing cognitive function , not level of consciousness .
These tests help evaluate dementia, psychiatric disorders, and cortical dysfunction but do not determine LOC.
Incorrect because LOC is primarily assessed using GCS.
Gait Disturbances
Abnormal gait can indicate neuromuscular disorders, cerebellar dysfunction, or Parkinson’s disease , but it does not directly assess consciousness level .
A patient can have a gait disturbance while being fully conscious.
Incorrect because gait abnormalities do not determine LOC.
Pupillary Reflex
The pupillary light reflex assesses brainstem integrity and can indicate brain injury , but it does not directly measure consciousness level .
Pupillary changes (e.g., fixed dilated pupils) can suggest severe brain injury, but a patient can be unconscious with normal pupillary reflexes .
Incorrect because it evaluates brainstem function, not LOC.
Deep Tendon Reflexes (DTRs)
DTRs assess spinal cord function and peripheral nervous system integrity , not consciousness .
A patient can be in a coma but still have preserved reflexes .
Incorrect because DTRs evaluate motor pathways, not LOC.
“This structure is found within the ventricles and acts as a specialized filtration system, producing the fluid that surrounds and protects the brain and spinal cord. What is it?”
67 / 128
Category:
Neurosciences – Physiology
Which of the following structures is responsible for the production of cerebrospinal fluid?
Cerebrospinal fluid (CSF ) is a clear, colorless fluid that cushions the brain and spinal cord, provides nutrients, and removes metabolic waste. It is primarily produced by a specialized structure in the ventricles of the brain.
Correct Answer: Choroid Plexus
Why is the choroid plexus the correct answer?
The choroid plexus is a vascular structure located in the lateral, third, and fourth ventricles of the brain.
It is composed of modified ependymal cells that filter plasma from capillaries and actively secrete CSF .
CSF circulates through the ventricular system and subarachnoid space, where it provides cushioning and metabolic support to the CNS.
Why Are the Other Options Incorrect?
Basal Ganglia
The basal ganglia are a group of subcortical nuclei involved in motor control, coordination, and movement regulation .
They have no role in CSF production , making this answer incorrect.
Hypothalamus
The hypothalamus regulates autonomic functions, hormone secretion, and homeostasis , but it does not produce CSF .
While it has roles in thirst, hunger, and temperature regulation, it is unrelated to CSF synthesis.
Corpus Callosum
The corpus callosum is a white matter structure that connects the left and right cerebral hemispheres.
It does not contain any secretory or vascular structures involved in CSF production.
Thalamus
The thalamus is a major sensory relay center in the brain, processing and transmitting sensory and motor signals to the cortex.
It plays no role in CSF production , making this an incorrect answer.
“This thalamic nucleus serves as a bridge between movement planning centers like the basal ganglia and cerebellum and the part of the brain responsible for executing voluntary actions. Which function does this best describe?”
68 / 128
Category:
Neurosciences – Physiology
The ventral lateral nucleus of the thalamus is concerned with which of the following functions?
The ventral lateral (VL) nucleus of the thalamus is a key relay center for motor control , integrating signals from the cerebellum and basal ganglia to coordinate voluntary movement. It plays a critical role in motor planning and execution by transmitting information to the primary motor cortex (precentral gyrus).
Correct Answer: Motor Activity
Why is motor activity the correct answer?
The ventral lateral nucleus receives input from:
The cerebellum (dentate nucleus) – for motor coordination.
The basal ganglia (globus pallidus and substantia nigra) – for voluntary movement control.
It projects to the primary motor cortex (precentral gyrus) and premotor cortex , which are responsible for initiating and regulating voluntary movements .
Dysfunction of this nucleus can lead to motor deficits , such as those seen in Parkinson’s disease and cerebellar disorders .
Why Are the Other Options Incorrect?
Circadian Rhythm
The suprachiasmatic nucleus (SCN) of the hypothalamus is the primary center for circadian rhythm regulation , not the ventral lateral nucleus.
The VL nucleus is involved in motor function , making this answer incorrect.
Alertness
The intralaminar nuclei of the thalamus , particularly the centromedian nucleus , and the reticular activating system (RAS) play a major role in maintaining alertness .
The ventral lateral nucleus does not regulate alertness , so this choice is incorrect.
Sensory Control
Sensory information is processed by the ventral posterior nucleus (VPL/VPM) of the thalamus , not the ventral lateral nucleus.
The ventral lateral nucleus is motor-related , making this answer incorrect.
Consciousness
The intralaminar nuclei and thalamic reticular nucleus are responsible for consciousness and wakefulness , often in conjunction with the brainstem’s reticular formation .
The ventral lateral nucleus has no direct role in consciousness , making this answer incorrect.
“Think about which hypothalamic nucleus is responsible for nurturing behaviors like bonding and lactation. This nucleus is also involved in stress responses but plays a critical role in releasing a hormone that promotes maternal functions.”
69 / 128
Category:
Neurosciences – Physiology
Suckling causes the release of oxytocin from which of the hypothalamus nuclei?
Suckling is a key physiological stimulus that triggers the release of oxytocin , which plays a crucial role in the milk ejection reflex (let-down reflex) during lactation. Oxytocin is synthesized by specific hypothalamic nuclei and released from the posterior pituitary into the bloodstream.
Correct Answer: Paraventricular Nucleus
Why is the paraventricular nucleus the correct answer?
The paraventricular nucleus (PVN) of the hypothalamus is the primary site for oxytocin production , along with the supraoptic nucleus .
During suckling , sensory afferent signals from the nipples travel via the spinothalamic tract to the hypothalamus , stimulating the paraventricular nucleus to release oxytocin.
Oxytocin is then secreted from the posterior pituitary into the bloodstream, causing contraction of the myoepithelial cells in the mammary glands , leading to milk ejection .
The paraventricular nucleus is also involved in stress responses by releasing corticotropin-releasing hormone (CRH) but is primarily responsible for oxytocin release in response to suckling.
Why Are the Other Options Incorrect?
Anterior Nucleus
The anterior nucleus is primarily involved in thermoregulation (heat dissipation) and parasympathetic activity .
It is not responsible for oxytocin release , making this answer incorrect.
Posterior Nucleus
The posterior nucleus is involved in heat conservation (sympathetic activity) and plays a role in autonomic functions .
It has no direct role in oxytocin production , making this choice incorrect.
Supraoptic Nucleus
The supraoptic nucleus does produce oxytocin and antidiuretic hormone (ADH/vasopressin) , but the paraventricular nucleus plays the dominant role in oxytocin release in response to suckling.
While the supraoptic nucleus contributes to oxytocin production, the primary response to suckling comes from the paraventricular nucleus , making this a less correct choice.
Suprachiasmatic Nucleus (SCN)
The SCN is the master circadian rhythm regulator , controlling sleep-wake cycles based on light input.
It has no role in oxytocin release or lactation , making this answer incorrect.
“This space becomes clinically relevant in cases of head trauma, where bridging veins rupture. It lies just beneath the toughest meningeal layer but does not normally exist in healthy brains.”
70 / 128
Category:
NeuroSciences – Anatomy
Subdural space is located between which of the following layers?
The subdural space is a potential space that lies between the dura mater and the arachnoid mater . It is not a true space under normal conditions but can become apparent in pathological situations, such as subdural hematomas , where blood accumulates due to venous rupture.
Correct Answer: Arachnoid Mater and Dura Mater
Why is this the correct answer?
The meninges are composed of three layers:
Dura mater (outermost layer) – tough and fibrous.
Arachnoid mater (middle layer) – delicate, web-like.
Pia mater (innermost layer) – thin and closely adheres to the brain.
The subdural space lies between the dura mater and arachnoid mater and is only present in pathological conditions , such as when bridging veins rupture, leading to subdural hematoma .
Why Are the Other Options Incorrect?
Dura Mater and Pia Mater
The pia mater is directly attached to the brain and spinal cord, with the arachnoid mater in between .
There is no direct space between the dura mater and pia mater , making this answer incorrect.
Pia Mater and Arachnoid Mater
The space between the pia and arachnoid mater is the subarachnoid space , which contains cerebrospinal fluid (CSF) .
This is NOT the subdural space , making this answer incorrect.
All of These
Since only the dura mater and arachnoid mater form the subdural space, this answer is incorrect.
None of These
The subdural space does exist (pathologically) between the dura mater and arachnoid mater, so this answer is incorrect.
“The brain structure most closely tied to smell must have a strong link to both the limbic system and the hypothalamus. Which pathway carries olfactory signals while also regulating autonomic responses?”
71 / 128
Category:
NeuroSciences – Anatomy
A 25-year-old-man comes to the outpatient department with complaints of an altered sense of smell after sustaining an injury to the area of the brain where the hypothalamus is located. In which associated structure is the lesion most likely to be located?
Why is the Medial Forebrain Bundle the Correct Answer?
The Medial Forebrain Bundle (MFB) is a major neural pathway connecting the olfactory and limbic structures to the hypothalamus and brainstem .
It carries olfactory, autonomic, and reward-related signals and connects the olfactory cortex, septal area, and hypothalamus .
Damage to the MFB can disrupt olfactory input processing, leading to altered sense of smell .
Why Are the Other Options Incorrect?
Mamillothalamic Fibers
These fibers connect the mammillary bodies (part of the hypothalamus) to the anterior nucleus of the thalamus and are part of the Papez circuit , which is crucial for memory processing.
Lesions in this pathway cause memory deficits (e.g., Wernicke-Korsakoff syndrome) but do not affect olfaction .
Since this pathway is unrelated to smell, it is incorrect.
Thalamohypothalamic Fibers
These fibers are part of the communication network between the thalamus and hypothalamus , mainly regulating autonomic functions and sensory integration .
They do not directly process olfactory information, making this an incorrect choice.
Amygdalohypothalamic Fibers
The amygdalohypothalamic fibers play a role in emotion regulation and autonomic responses (e.g., fear response, hormonal stress responses).
While the amygdala receives some olfactory input , lesions here primarily result in emotional disturbances rather than olfactory dysfunction .
This choice is incorrect because the amygdala does not directly process olfactory signals .
Tegmental Fibers
These fibers originate in the tegmentum of the midbrain , playing a role in arousal, autonomic regulation, and motor functions .
They do not contribute to olfactory processing, making this answer incorrect.
“The corpus callosum helps different brain regions work together. If this connection is missing, which aspect of human function—thinking, moving, or sensing—would be most affected?”
72 / 128
Category:
Neurosciences – Pathology
Agenesis of the corpus callosum is associated with which of the following symptoms?
Why is Mental Retardation (Intellectual Disability) the Correct Answer?
The corpus callosum plays a crucial role in interhemispheric communication, which is essential for cognitive functions such as problem-solving, memory, and reasoning.
Many individuals with ACC exhibit intellectual disability , ranging from mild to severe, depending on the extent of the agenesis and any associated brain malformations.
Cognitive deficits often include:
Delayed milestones (e.g., walking, talking, toilet training)
Poor problem-solving skills
Difficulty with abstract thinking and executive function
Social and communication difficulties
ACC is often part of genetic syndromes (e.g., Aicardi syndrome, Andermann syndrome), which can further contribute to intellectual disability.
Why is Loss of Coordination Also a Possible Answer?
Some individuals with ACC show ataxia, clumsiness, and difficulty coordinating complex motor tasks due to impaired interhemispheric communication.
Motor coordination deficits can be present but are not the primary feature —intellectual disability is a more consistent finding.
This is why loss of coordination may be seen, but mental retardation is the more definitive answer .
Why are the Other Options Incorrect?
Loss of Sensory Control
The corpus callosum does not control sensory function directly.
Sensory input is primarily processed in the thalamus and somatosensory cortex .
While information transfer between hemispheres may be impaired, it does not lead to a complete loss of sensory control .
Loss of Speech
Some individuals with ACC may have delayed language development and difficulty understanding complex speech.
However, complete loss of speech is NOT a defining feature.
Speech function is primarily controlled by Broca’s and Wernicke’s areas , which are unaffected in isolated ACC.
Loss of Motor Control
The motor system (corticospinal tract, basal ganglia, and cerebellum) remains functional in ACC.
While coordination may be impaired, motor control (i.e., ability to move) is preserved .
“This hypothalamic nucleus is essential for the fight-or-flight response, working closely with the sympathetic nervous system. Think about which system needs to be activated when you are faced with danger or stress.”
73 / 128
Category:
Neurosciences – Physiology
Which of the following hypothalamic nucleus is responsible for increasing the heart rate and blood pressure?
The hypothalamus plays a critical role in autonomic regulation, including controlling heart rate and blood pressure. Different hypothalamic nuclei have specialized functions, and one of them is particularly involved in sympathetic nervous system activation, which increases heart rate and blood pressure .
Correct Answer: Lateral Nucleus
Why is the lateral nucleus the correct answer?
The lateral hypothalamic nucleus is primarily involved in sympathetic nervous system activation , which leads to increased heart rate and blood pressure .
It also plays a role in hunger regulation and arousal .
Activation of the lateral nucleus stimulates the release of norepinephrine and epinephrine , leading to vasoconstriction and tachycardia, which are essential responses in situations like stress or fight-or-flight reactions.
Why are the other options incorrect?
Anterior Nucleus
The anterior nucleus has the opposite effect—it regulates parasympathetic activity , which promotes vasodilation and reduces heart rate .
It is also involved in heat dissipation (e.g., sweating to cool down the body).
Since we are looking for a nucleus that increases heart rate and blood pressure, this is incorrect.
Suprachiasmatic Nucleus (SCN)
The SCN is the master clock of the body, responsible for regulating circadian rhythms based on light exposure.
It does NOT have a direct role in blood pressure or heart rate control.
Since it is unrelated to sympathetic activation, this choice is incorrect.
Tuberomamillary Nucleus
This nucleus is the primary source of histaminergic neurons , which play a role in wakefulness and arousal but do not directly control heart rate and blood pressure.
While wakefulness can influence autonomic tone, this nucleus is NOT the main regulator of cardiovascular responses.
Therefore, this is incorrect.
Supraoptic Nucleus
The supraoptic nucleus produces antidiuretic hormone (ADH/vasopressin) , which regulates water balance and blood pressure by controlling kidney function.
While ADH can influence blood pressure through water retention, it does not directly increase heart rate .
The key distinction is that the supraoptic nucleus regulates fluid balance rather than acute cardiovascular sympathetic responses, making this an incorrect answer.
“This pathway is essential for precise sensations that require fine discrimination. Consider what type of sensory test would be most affected in diseases like vitamin B12 deficiency or tabes dorsalis.”
74 / 128
Category:
Neurosciences – Physiology
The dorsal column pathway is best tested by checking which of the following sensations?
The dorsal column-medial lemniscus (DCML) pathway is responsible for conveying fine touch, vibration, conscious proprioception, and pressure sensations from the body to the brain. Testing for these sensory modalities helps assess the integrity of this pathway.
Correct Answer: Vibration
Why is vibration the correct answer?
The dorsal column pathway carries sensory information related to:
Vibration (pacinian corpuscles)
Fine touch (meissner corpuscles, merkel cells)
Conscious proprioception (muscle spindles, golgi tendon organs)
Pressure (ruffini endings)
Vibration sense is best tested using a tuning fork (128 Hz) , typically applied to bony prominences like the distal interphalangeal joint of the fingers or toes.
Loss of vibration sensation is an early sign of dorsal column dysfunction , often seen in conditions such as vitamin B12 deficiency (subacute combined degeneration), tabes dorsalis (tertiary syphilis), and diabetic neuropathy .
Why are the other options incorrect?
Temperature
Temperature sensation is carried by the spinothalamic tract , NOT the dorsal column pathway.
The lateral spinothalamic tract specifically transmits pain and temperature information.
Since temperature is not part of the dorsal column system, this is incorrect.
Unconscious Proprioception
Unconscious proprioception (e.g., joint position for automatic movements) is transmitted via the spinocerebellar tracts to the cerebellum, not the dorsal column.
The dorsal column pathway carries conscious proprioception , meaning awareness of limb position.
Therefore, this is an incorrect answer.
Light Touch
Light touch is carried by both the dorsal column and the anterior spinothalamic tract , meaning it is NOT exclusively a function of the dorsal column.
Because it has dual pathways , lesions of the dorsal column may not completely abolish light touch sensation.
Thus, this is not the best answer.
Pain
Pain sensation is transmitted by the lateral spinothalamic tract , NOT the dorsal column.
Damage to the dorsal column does not affect pain perception, making this choice incorrect.
“Think about which hormones are produced by the supraoptic and paraventricular nuclei. What would happen if the body suddenly stopped making one of these hormones? Consider its role in maintaining homeostasis and fluid balance.”
75 / 128
Category:
Neurosciences – Physiology
Lesions of the supraoptic and paraventricular nuclei would result in which of the following disorders?
The supraoptic and paraventricular nuclei of the hypothalamus play a crucial role in the synthesis and release of antidiuretic hormone (ADH, also known as vasopressin) and oxytocin . Damage to these nuclei impairs the production and secretion of ADH, leading to an inability to concentrate urine, which is characteristic of diabetes insipidus (DI).
Correct Answer: Diabetes Insipidus
Why is diabetes insipidus the correct answer?
The supraoptic and paraventricular nuclei produce ADH , which regulates water balance by acting on the kidneys to promote water reabsorption.
Lesions in these nuclei disrupt ADH production , leading to excessive urine output (polyuria) and excessive thirst (polydipsia), which are hallmark features of central diabetes insipidus .
The kidneys fail to concentrate urine, causing the excretion of large volumes of dilute urine .
Why are the other options incorrect?
Kallmann Syndrome
This condition is caused by a failure of GnRH-producing neurons to migrate from the olfactory placode to the hypothalamus.
It results in hypogonadotropic hypogonadism and anosmia (loss of sense of smell) due to associated olfactory bulb hypoplasia.
The supraoptic and paraventricular nuclei are NOT involved in GnRH secretion, making this an incorrect choice.
Korsakoff Syndrome
Korsakoff syndrome is a chronic memory disorder resulting from thiamine (vitamin B1) deficiency , often seen in chronic alcoholics.
It primarily affects the mammillary bodies and thalamus , leading to anterograde and retrograde amnesia , confabulation, and personality changes.
Since the supraoptic and paraventricular nuclei are NOT involved in memory processing, this answer is incorrect.
Klüver-Bucy Syndrome
This syndrome is caused by bilateral lesions of the amygdala , leading to symptoms such as hyperphagia, hypersexuality, docility, and loss of fear responses .
It is NOT associated with hypothalamic nuclei lesions, making this an incorrect answer.
Disturbances in Circadian Rhythms
Circadian rhythm regulation is primarily controlled by the suprachiasmatic nucleus (SCN) of the hypothalamus, which responds to light cues.
While the supraoptic and paraventricular nuclei are involved in fluid balance and hormone secretion, they are NOT the key regulators of circadian rhythms.
Therefore, this is incorrect.
“This autoimmune disease disrupts communication at the neuromuscular junction, leading to muscle weakness that worsens with activity.”
76 / 128
Category:
Neurosciences – Pathology
Which of the following is an autoimmune disease affecting the nicotinic acetylcholine receptors?
Understanding Myasthenia Gravis (MG)
Myasthenia gravis (MG) is an autoimmune disorder that targets the nicotinic acetylcholine receptors (nAChRs) at the neuromuscular junction .
Autoantibodies bind to these receptors, leading to decreased synaptic transmission , resulting in muscle weakness and fatigue .
Symptoms worsen with activity (fatigue effect) and improve with rest.
Key Features of Myasthenia Gravis:
✅ Muscle weakness (especially ocular, bulbar, and limb muscles ). ✅ Ptosis (drooping eyelids) and diplopia (double vision) . ✅ Fluctuating weakness (worsens with exertion, improves with rest). ✅ Improves with acetylcholinesterase inhibitors (e.g., edrophonium test or pyridostigmine treatment ).
Why the Other Options Are Incorrect
Amyotrophic lateral sclerosis (ALS) – Incorrect
ALS is a neurodegenerative disease affecting upper and lower motor neurons , leading to progressive muscle weakness and wasting .
It is not an autoimmune disorder and does not involve acetylcholine receptors .
Guillain-Barré syndrome – Incorrect
Guillain-Barré syndrome (GBS) is an autoimmune disorder , but it affects peripheral nerves via demyelination , leading to ascending paralysis .
It does not target the nicotinic acetylcholine receptors .
Multiple sclerosis (MS) – Incorrect
MS is an autoimmune disease affecting the central nervous system , specifically the myelin sheath .
It does not involve acetylcholine receptors .
Duchenne muscular dystrophy (DMD) – Incorrect
DMD is a genetic disorder caused by dystrophin gene mutations , leading to progressive muscle degeneration .
It is not autoimmune and does not involve the nicotinic acetylcholine receptors.
“This thalamic nucleus processes information related to a certain sense before sending it to the cerebral cortex.”
77 / 128
Category:
NeuroSciences – Anatomy
The medial geniculate body (MGB) receives input from which of the following sensory pathways?
Understanding the Medial Geniculate Body (MGB)
The medial geniculate body (MGB) is part of the thalamus , specifically involved in the auditory pathway .
It acts as the relay center for auditory signals traveling from the inferior colliculus (midbrain) to the primary auditory cortex in the temporal lobe .
Pathway of Auditory Signals through the MGB:
Cochlear receptors (hair cells) → Cochlear nerve (CN VIII) → Cochlear nuclei (brainstem).
Inferior colliculus (midbrain) receives input via the lateral lemniscus.
Medial geniculate body (thalamus) processes auditory information.
Primary auditory cortex (temporal lobe) receives input from MGB.
Since the MGB is specific to auditory processing , it is the correct answer.
Why the Other Options Are Incorrect
Olfactory – Incorrect
The olfactory pathway bypasses the thalamus and projects directly to the olfactory cortex and limbic system .
It does not relay through the MGB .
Tactile – Incorrect
Tactile (touch and proprioception) information is processed via the ventral posterolateral (VPL) nucleus of the thalamus, not the MGB .
Optic – Incorrect
Visual signals travel through the lateral geniculate body (LGB) of the thalamus, not the MGB .
Gustatory – Incorrect
Taste (gustation) information is processed via the ventral posteromedial (VPM) nucleus of the thalamus, not the MGB .
“This structure, responsible for producing cerebrospinal fluid, is suspended in an upper region of one of the brain’s fluid-filled chambers.”
78 / 128
Category:
NeuroSciences – Anatomy
Where is the choroid plexus located?
Understanding the Location of the Choroid Plexus
The choroid plexus is a network of modified ependymal cells and blood vessels that produces cerebrospinal fluid (CSF) . It is found in specific parts of the ventricular system of the brain:
Lateral Ventricles → Located in the medial part of the roof of the inferior horn and extends into the body of the lateral ventricles.
Third Ventricle → Located in the roof , suspended from the fornix .
Fourth Ventricle → Located in the roof , extending into the lateral recesses.
Why the Correct Answer is Right
✅ The choroid plexus of the third ventricle is located in its roof because:
It is formed by the tela choroidea , a layer of pia mater and ependymal cells .
It produces cerebrospinal fluid (CSF) , which circulates through the ventricular system.
It is suspended from the fornix , forming a vascular fringe in the roof of the third ventricle.
Why the Other Options Are Incorrect
Floor of lateral ventricle – Incorrect
The choroid plexus of the lateral ventricles is found in the roof , not the floor.
Floor of fourth ventricle – Incorrect
The choroid plexus in the fourth ventricle is located in the roof , specifically the inferior medullary velum , not the floor.
Floor of third ventricle – Incorrect
The choroid plexus of the third ventricle is found in the roof , suspended from the fornix , not in the floor.
Lateral boundary of fourth ventricle – Incorrect
The choroid plexus of the fourth ventricle extends into the lateral recesses , but it is not located in the lateral boundary .
“This reflex changes as the nervous system matures, showing a different response in infants compared to adults.”
79 / 128
Category:
Neurosciences – Physiology
Which of the following is a reflex that develops over time?
Why Watering of the Mouth (Salivation) is the Correct Answer
Salivation (water secretion in the mouth) is a learned or conditioned reflex in some cases.
While the basic salivary reflex is present at birth (in response to food), it can be modified over time through conditioning (e.g., Pavlovian response).
Example:
Initially, an infant only salivates in response to food in the mouth .
Over time, as the brain develops, salivation can be triggered by the mere sight, smell, or thought of food —this is a learned response that develops with experience.
Why the Other Options Are Incorrect
Plantar Reflex – Incorrect
While the Babinski sign disappears with age, this is due to corticospinal tract maturation rather than reflex development.
The reflex does not “develop” over time , but rather modifies from an infantile response to an adult response .
Reflex development typically refers to new reflexes forming , not just modifications of existing ones.
Abdominal Reflex – Incorrect
This is a spinal reflex present early in life, and it does not require learning or experience to develop .
Corneal Reflex – Incorrect
This protective blink reflex is present from birth and does not undergo significant changes over time.
None of These – Incorrect
Since watering of the mouth (salivation) can be learned and conditioned , one of the options is indeed correct.
“Consider whether the risk comes from frequency or from rapid, life-threatening complications involving cranial nerves and vital structures.”
80 / 128
Category:
NeuroSciences – Anatomy
Which of the following is the most common and dangerous site of thrombosis?
The correct answer depends on the interpretation of “most common and dangerous.”
If we focus on most common , then superior sagittal sinus thrombosis (SSST) is the most frequently affected site in cerebral venous sinus thrombosis (CVST).
If we focus on most dangerous , then cavernous sinus thrombosis (CST) is more acutely life-threatening due to its rapid progression, cranial nerve involvement, and risk of brainstem complications.
Why the Superior Sagittal Sinus is the Most Common Site of Thrombosis
Superior sagittal sinus thrombosis (SSST) accounts for the majority of CVST cases .
It can lead to: ✅ Increased intracranial pressure (due to impaired venous drainage). ✅ Headache (most common symptom) , seizures, and papilledema. ✅ Bilateral hemorrhagic infarctions (often in the parasagittal region).
It is dangerous , but not necessarily the most acutely life-threatening .
Why Cavernous Sinus Thrombosis is the Most Dangerous
Cavernous sinus thrombosis (CST) is less common but more rapidly fatal if untreated because: ✅ It involves cranial nerves III, IV, V1, V2, and VI , leading to ophthalmoplegia and vision loss . ✅ It can result in septicemia, meningitis, and brainstem compression . ✅ It spreads via facial venous connections (danger triangle of the face) , making it a direct infection route to the brain . ✅ Mortality is high if untreated , while SSST is often treatable with anticoagulation.
Final Answer Based on Different Interpretations
If the question asks for the most common site: Superior sagittal sinus ✅
If the question asks for the most dangerous site: Cavernous sinus ✅
“This nucleus plays a crucial role in regulating food intake, and damage to it can lead to excessive eating and obesity.”
81 / 128
Category:
NeuroSciences – Anatomy
Which nucleus of the hypothalamus contains the satiety center?
Understanding the Ventromedial Nucleus and Satiety Control
The ventromedial nucleus (VMN) of the hypothalamus plays a key role in inhibiting hunger and promoting satiety .
It receives input from leptin , a hormone released by adipose tissue that signals fullness.
Lesions in the ventromedial nucleus lead to hyperphagia (excessive eating) and obesity.
Why the Other Options Are Incorrect
Supraoptic nucleus – Incorrect
The supraoptic nucleus primarily produces antidiuretic hormone (ADH/vasopressin) , which regulates water balance.
It does not control hunger or satiety.
Posterolateral nucleus – Incorrect
The posterolateral hypothalamic area is involved in sympathetic nervous system activation and arousal , not satiety.
Paraventricular nucleus – Incorrect
The paraventricular nucleus (PVN) regulates autonomic function, stress response, and oxytocin release , but it does not act as the primary satiety center .
Suprachiasmatic nucleus – Incorrect
The suprachiasmatic nucleus (SCN) is the body’s master clock , regulating circadian rhythms based on light input from the retina.
It does not control hunger or satiety .
These cells are responsible for myelination outside the brain and spinal cord, helping in nerve regeneration after injury.
82 / 128
Category:
Neurosciences – Histology
Which of the following statements is correct regarding the Schwann cells?
Schwann cells are glial cells that are responsible for the myelination of axons in the peripheral nervous system (PNS) . Each Schwann cell wraps around a single axon segment, providing insulation and increasing the speed of nerve impulse conduction through saltatory conduction .
🔹 Key Features of Schwann Cells:
Found in the PNS
Myelinate one axon segment per Schwann cell
Aid in nerve regeneration after injury
This makes the statement “They are produced in the peripheral nervous system” correct.
Why Others Are Incorrect?
❌ One Schwann cell covers 30 nerve fibers:
Incorrect because each Schwann cell myelinates only one axon segment .
In contrast, oligodendrocytes (CNS glial cells) can myelinate multiple axons .
❌ They are produced in the central nervous system:
Incorrect because Schwann cells are in the PNS .
Oligodendrocytes are the CNS myelinating cells.
❌ Multiple sclerosis is a demyelinating disorder that affects the Schwann cells:
Incorrect because MS affects the CNS (oligodendrocytes), not Schwann cells .
Guillain-Barré syndrome (GBS) is a demyelinating disease of the PNS , affecting Schwann cells.
❌ Schwann cells cover all cranial nerves:
Incorrect because some cranial nerves (like the optic nerve) are myelinated by oligodendrocytes (since they are part of the CNS).
Schwann cells only myelinate peripheral cranial nerves (e.g., CN III–XII).
LMN lesions lead to weakness with decreased muscle tone and reflexes , while UMN lesions cause increased tone and reflexes .
83 / 128
Category:
NeuroSciences – Anatomy
Which of the following is a sign of lower motor neuron (LMN) lesions?
Lower motor neuron (LMN) lesions occur when there is damage to the alpha motor neurons in the anterior horn of the spinal cord or their axons in the peripheral nerves .
🔹 Key Features of LMN Lesions:
Flaccid paralysis (loss of muscle tone and voluntary movement)
Muscle atrophy (wasting due to denervation)
Absent or reduced reflexes (hyporeflexia or areflexia)
Fasciculations (spontaneous muscle twitching)
Flaccid paralysis occurs because LMNs directly innervate muscles , and their loss results in loss of voluntary and reflexive movements .
Why Others Are Incorrect?
❌ Spasticity:
Seen in upper motor neuron (UMN) lesions , not LMN lesions.
Characterized by increased muscle tone and resistance to passive movement.
❌ Clasp-knife response:
Sign of UMN lesions , especially in corticospinal tract damage .
Refers to initial resistance followed by sudden release during passive movement.
❌ Hyperactive stretch reflex:
UMN lesion feature due to loss of inhibitory control from the brain.
LMN lesions cause diminished or absent reflexes , not hyperactive ones.
❌ Disuse atrophy:
Occurs in UMN lesions due to lack of use but with intact nerve supply .
LMN lesions cause neurogenic atrophy , which is rapid and severe .
IPSP occurs when the inside of the neuron becomes more negative , making it harder to reach the threshold for firing an action potential. This often happens when an ion moves into the cell, increasing its negativity.
84 / 128
Category:
Neurosciences – Physiology
Inhibitory postsynaptic potential is caused by which of the following?
An inhibitory postsynaptic potential (IPSP) makes the postsynaptic neuron less likely to fire an action potential by hyperpolarizing the membrane. This occurs when the inside of the neuron becomes more negative , moving further away from the threshold for excitation.
🔹 Main mechanisms for IPSP:
Chloride (Cl⁻) influx : Brings negative charge into the cell, hyperpolarizing it.
Potassium (K⁺) efflux : Carries positive charge out, also causing hyperpolarization.
Among these, chloride influx is the primary mechanism for IPSP, mediated by GABA-A and glycine receptors .
Why Others Are Incorrect?
❌ Bicarbonate influx:
Not a major ion responsible for IPSPs in neurons.
❌ Calcium influx:
Depolarizes the cell, making it more likely to fire (excitatory).
❌ Potassium influx:
Would make the inside less negative (depolarization), not hyperpolarized.
IPSP occurs due to potassium efflux, not influx.
❌ Sodium influx:
Depolarizes the membrane, making it excitatory (EPSP ), not inhibitory.
“This brain region fine-tunes movement and is often affected in degenerative disorders that cause involuntary motor symptoms.”
85 / 128
Category:
Neurosciences – Pathology
A 78-year-old man presents to the outpatient department with a tremor that disappears on movement. The doctor suspects the possibility of a lesion in his brain. Where is the lesion most likely to be located?
Understanding the Clinical Presentation
A tremor that disappears on movement is characteristic of a resting tremor , commonly seen in Parkinson’s disease .
Parkinson’s disease is caused by degeneration of dopaminergic neurons in the substantia nigra (part of the basal ganglia) .
Key Features of Parkinsonian Tremor:
✅ Resting tremor (“pill-rolling”) → Tremor is present at rest but disappears with movement. ✅ Bradykinesia → Slowness of movement. ✅ Rigidity (cogwheel rigidity) → Resistance to passive movement. ✅ Postural instability → Difficulty maintaining balance.
Since the basal ganglia regulate movement , damage here results in Parkinsonian symptoms , making it the most likely site of the lesion.
Why the Other Options Are Incorrect
Pons – Incorrect
The pons contains the corticospinal tracts, cranial nerve nuclei, and cerebellar connections , but it is not primarily involved in resting tremors .
Pontine lesions usually cause motor or sensory deficits rather than tremors.
Medulla – Incorrect
The medulla regulates autonomic functions (breathing, heart rate, swallowing) .
Lesions here cause cranial nerve deficits, dysphagia, or respiratory issues , not a Parkinsonian tremor.
Spinal cord – Incorrect
The spinal cord primarily carries motor and sensory pathways .
A spinal lesion would cause weakness, sensory loss, or reflex abnormalities , not a resting tremor.
Cerebellum – Incorrect
The cerebellum is responsible for coordination and balance .
Cerebellar lesions cause an intention tremor (tremor that worsens with movement), not a resting tremor .
A TIA is a temporary event, while a stroke is defined by symptoms persisting beyond a specific time threshold.
86 / 128
Category:
Neurosciences – Community Medicine + Behavioural Sciences
According to the World Health Organization, how long is the loss of cerebral function for it to be called a stroke?
The World Health Organization (WHO) defines a stroke as a sudden loss of cerebral function due to a vascular cause that lasts more than 24 hours or leads to death.
🔹 Key Criteria for Stroke (WHO Definition):
Neurological deficit of vascular origin
Lasts more than 24 hours
Can be ischemic (87%) or hemorrhagic (13%)
If symptoms resolve within 24 hours , it is classified as a transient ischemic attack (TIA) instead of a stroke.
Why Others Are Incorrect?
❌ More than 3 minutes:
While brain tissue starts to suffer ischemic damage in minutes, a stroke is defined based on symptom duration , not onset speed.
❌ Less than 1 hour:
Neurological deficits lasting less than 24 hours are categorized as TIA , not a stroke.
❌ More than 1 hour:
Some strokes last more than an hour, but the official definition requires 24 hours or more .
❌ At least 3 days:
A stroke does not need to last 3 days to be classified as one. 24 hours is the cutoff .
“This term refers to a region of the brainstem and cerebellum that plays a crucial role in motor coordination and autonomic functions.”
87 / 128
Category:
Neurosciences – Embryology
What is another name for the hindbrain?
Understanding Brain Development
The brain develops from three primary vesicles in the embryo:
Prosencephalon (Forebrain) → Becomes the Telencephalon and Diencephalon .
Mesencephalon (Midbrain) → Remains as the Midbrain .
Rhombencephalon (Hindbrain) → Develops into the Metencephalon and Myelencephalon .
Thus, the hindbrain is also called the rhombencephalon .
Why the Other Options Are Incorrect
Metencephalon – Incorrect
The metencephalon is a subdivision of the hindbrain (rhombencephalon) that gives rise to the pons and cerebellum , but it is not the full hindbrain .
Diencephalon – Incorrect
The diencephalon is part of the forebrain (prosencephalon) and includes the thalamus, hypothalamus, and epithalamus , not the hindbrain.
Myelencephalon – Incorrect
The myelencephalon is another subdivision of the hindbrain , giving rise to the medulla oblongata , but it does not represent the entire hindbrain.
Mesencephalon – Incorrect
The mesencephalon is the midbrain , which is distinct from the hindbrain.
These sinuses are inside the dura , not between dura and arachnoid. Their structure allows bidirectional blood flow due to the absence of valves .
88 / 128
Category:
NeuroSciences – Anatomy
Which of the following is incorrect regarding dural venous sinuses?
Dural venous sinuses are large, valveless channels that drain venous blood from the brain into the internal jugular vein . They are formed by the splitting of the dura mater layers and are NOT located between the arachnoid and dura mater .
🔹 Correct Location:
Between the periosteal and meningeal layers of the dura mater
The arachnoid mater is external to these sinuses , separated by the subdural space
Thus, the statement “They are present between the arachnoid and dura mater” is incorrect.
Why Others Are Correct?
✅ The flow is bidirectional:
Venous blood can reverse direction due to the absence of valves and the connection with emissary veins.
✅ They are devoid of smooth muscles:
Unlike normal veins, dural venous sinuses lack smooth muscle , making them structurally unique.
✅ They are lined by endothelium:
The walls of these sinuses are lined by simple squamous endothelial cells .
✅ They have no valves:
Unlike systemic veins, dural venous sinuses lack valves , allowing free communication between intracranial and extracranial veins.
CSF exits the fourth ventricle through a structure named after an anatomist. It allows fluid to flow into the subarachnoid space , ensuring proper circulation.
89 / 128
Category:
NeuroSciences – Anatomy
The cerebrospinal fluid leaves the fourth ventricle through which of the following?
Cerebrospinal fluid (CSF ) is produced in the choroid plexus of the lateral, third, and fourth ventricles . It flows through a series of openings before reaching the subarachnoid space , where it is eventually absorbed.
🔹 Flow of CSF: 1️⃣ Lateral ventricles → Foramen of Monro 2️⃣ Third ventricle → Cerebral aqueduct 3️⃣ Fourth ventricle → Foramen of Magendie (midline) & Foramina of Luschka (lateral) 4️⃣ Subarachnoid space → Arachnoid villi → Venous system
🔹 The Foramen of Magendie is a midline opening in the fourth ventricle that allows CSF to exit into the subarachnoid space .
Why Others Are Incorrect?
❌ Arachnoid villi:
These are structures in the superior sagittal sinus responsible for CSF absorption , not its exit from the fourth ventricle.
❌ Foramen of Monro:
Connects the lateral ventricles to the third ventricle , not the fourth ventricle.
❌ Cerebral aqueduct:
Connects the third ventricle to the fourth ventricle , but does not allow CSF to leave the fourth ventricle.
❌ Central canal:
Extends downward from the fourth ventricle into the spinal cord , but most CSF exits through Foramen of Magendie & Foramina of Luschka into the subarachnoid space .
“This type of lesion affects the transmission of specific sensory information to the brain, leading to difficulties.”
90 / 128
Category:
Neurosciences – Pathology
Lesion of the dorsal column results in which of the following?
Understanding the Dorsal Column-Medial Lemniscus Pathway
The dorsal column carries sensory information related to: ✅ Fine touch (discriminative touch) ✅ Proprioception (position sense) ✅ Vibration sense
The pathway consists of:
Fasciculus gracilis (lower limb)
Fasciculus cuneatus (upper limb)
A lesion in the dorsal column results in sensory ataxia , meaning the individual loses proprioception , making coordinated movement difficult.
Why the Correct Answer is Right
✅ Sensory ataxia is the result of a dorsal column lesion.
Ataxia means lack of coordination , and in this case, it occurs due to impaired proprioception , not motor dysfunction.
Patients may demonstrate a positive Romberg sign (instability when closing their eyes) because they rely on vision for balance when proprioception is lost.
Why the Other Options Are Incorrect
Think small deep arteries, high blood pressure → tiny strokes in deep brain structures .
91 / 128
Category:
Neurosciences – Pathology
Lacunar infarcts are caused by hypertension in which of the following?
Lacunar infarcts are small, deep infarcts that result from occlusion of small penetrating arteries supplying the deep structures of the brain, such as the basal ganglia, thalamus, internal capsule, and pons .
🔹 Main cause: Chronic hypertension and diabetes 🔹 Mechanism: Hypertension leads to lipohyalinosis and microatheromas , causing narrowing or occlusion of small penetrating arteries . 🔹 Effects: Can cause pure motor stroke, pure sensory stroke, or ataxic hemiparesis depending on the affected area.
Why Others Are Incorrect?
❌ Large-sized artery:
Large arteries (e.g., internal carotid, vertebral arteries ) are involved in atherosclerosis and embolic strokes , not lacunar infarcts.
❌ Medium-sized artery:
Medium-sized arteries (e.g., middle cerebral artery, anterior cerebral artery ) are involved in cortical infarcts , not deep lacunar infarcts.
❌ Communicating artery:
These arteries form part of the Circle of Willis and are more commonly affected in aneurysms , not lacunar infarcts.
❌ All of these:
Only penetrating arteries are involved in lacunar infarcts, so this option is incorrect.
“This groove accommodates a major artery and runs along the midline of the brainstem’s bridge-like structure.”
92 / 128
Category:
NeuroSciences – Anatomy
Which anatomical feature is present on the anterior surface of the pons?
Understanding the Anatomy of the Anterior Surface of the Pons
The anterior surface of the pons is characterized by a central indentation called the basilar groove , which accommodates the basilar artery . This groove runs vertically along the midline of the pons .
Why the Other Options Are Incorrect
Sulcus limitans → Incorrect
The sulcus limitans is found on the floor of the fourth ventricle (posterior surface of the pons) , not the anterior surface.
Vestibular area → Incorrect
The vestibular area is located on the posterior surface of the pons , within the rhomboid fossa of the fourth ventricle.
Facial colliculus → Incorrect
The facial colliculus is also on the posterior surface of the pons , formed by the looping fibers of the facial nerve (CN VII) over the abducens nucleus (CN VI) .
Median sulcus → Incorrect
The median sulcus is located in the midline of the floor of the fourth ventricle , not on the anterior pons.
“Think about the part of the cerebellum that is closely connected to the vestibular system. Which lobe integrates sensory information about head position and movement to maintain balance and posture?”
93 / 128
Category:
NeuroSciences – Anatomy
An old man was having difficulty in walking and maintaining his posture. A lesion in which of the following structures will cause these symptoms?
The flocculonodular lobe is part of the cerebellum and is primarily responsible for maintaining balance and coordinating eye movements. It receives input from the vestibular system (which senses head position and movement) and sends output to the vestibular nuclei, which influence posture and gait. A lesion in the flocculonodular lobe disrupts this balance control, leading to symptoms such as difficulty walking (ataxia) and maintaining posture, often referred to as truncal ataxia . Patients may also exhibit nystagmus (involuntary eye movements) due to the lobe’s role in coordinating eye movements.
Why the Other Options Are Incorrect:
Anterior lobe of cerebellum
The anterior lobe of the cerebellum is primarily involved in coordinating voluntary movements, particularly in the lower limbs. A lesion here would cause gait ataxia (staggering or unsteady walking), but it would not specifically affect posture or balance to the same extent as a flocculonodular lobe lesion. The anterior lobe is more associated with motor execution rather than balance maintenance.
Pons
The pons is part of the brainstem and contains nuclei that relay signals between the cerebrum, cerebellum, and spinal cord. While a lesion in the pons can cause a variety of neurological deficits (e.g., facial weakness, sensory loss, or ataxia), it is not specifically associated with the characteristic balance and posture issues seen in this case. Pontine lesions typically involve more widespread symptoms due to its role as a relay center.
Posterior lobe of cerebellum
The posterior lobe of the cerebellum is involved in planning and fine-tuning voluntary movements, particularly in the upper limbs. A lesion here would cause limb ataxia (clumsiness or incoordination of the arms and hands) rather than the truncal ataxia and gait disturbances described in the question.
Medulla oblongata
The medulla oblongata is part of the brainstem and controls vital functions such as breathing, heart rate, and blood pressure. A lesion here could cause life-threatening issues like respiratory failure or cardiovascular instability, as well as symptoms like dysphagia (difficulty swallowing) or dysarthria (difficulty speaking). However, it is not directly responsible for balance and posture, which are primarily controlled by the flocculonodular lobe of the cerebellum.
“This reflex involves adjustments in vision that require conscious perception and higher-level processing beyond simple brainstem reflexes.”
94 / 128
Category:
Neurosciences – Physiology
Which of the following visual reflexes involves the cerebral cortex?
Understanding the Accommodation Reflex
The accommodation reflex is the process by which the eye adjusts for near vision by: ✅ Lens thickening (ciliary muscle contraction) ✅ Pupil constriction (to enhance focus) ✅ Convergence of the eyes (to align both eyes on a near object)
Unlike brainstem-mediated reflexes , the accommodation reflex requires input from the cerebral cortex , particularly the visual association areas in the occipital lobe .
Pathway of the Accommodation Reflex:
Retina → Optic nerve → Lateral geniculate nucleus (thalamus) → Visual cortex (occipital lobe)
Visual cortex → Pretectal area → Edinger-Westphal nucleus (midbrain)
Edinger-Westphal nucleus → Ciliary ganglion → Contraction of ciliary muscle and pupil constriction
Since cortical processing is involved in interpreting the visual target and initiating focus adjustment , this reflex differs from the others, which are mediated by the brainstem without cortical involvement .
Why the Other Options Are Incorrect (These Reflexes Are Brainstem-Mediated and Do Not Require the Cerebral Cortex)
Corneal reflex → Brainstem-mediated (Trigeminal and Facial Nerve Reflex)
Stimulus: Touching the cornea.
Pathway:
Afferent: Trigeminal nerve (CN V1) → Brainstem.
Efferent: Facial nerve (CN VII) → Orbicularis oculi muscle (eye blink).
No cerebral cortex involvement.
Direct pupillary reflex → Brainstem-mediated (Midbrain Reflex, CN II and CN III)
Stimulus: Light shined into one eye.
Pathway:
Afferent: Optic nerve (CN II) → Pretectal nucleus (midbrain).
Efferent: Edinger-Westphal nucleus (CN III) → Pupillary sphincter constriction.
No cortical involvement; purely a midbrain reflex.
Consensual reflex → Brainstem-mediated (Same as Direct Pupillary Reflex but in Opposite Eye)
Light in one eye causes pupil constriction in the opposite eye via bilateral pretectal nucleus activation.
No cortical involvement.
Visual body reflex → Brainstem-mediated (Tectospinal Tract, Reflexive Head and Neck Movements)
This reflex adjusts head and body position in response to sudden visual stimuli.
Mediated by the superior colliculus and tectospinal tract , not the cerebral cortex.
“This system influences levels of awareness and responsiveness, playing a role in regulating different states of consciousness”
95 / 128
Category:
Neurosciences – Physiology
What is the function of the reticular activating system?
Understanding the Reticular Activating System (RAS)
The reticular activating system (RAS) is a network of neurons in the brainstem , particularly in the midbrain, pons, and medulla .
It plays a critical role in consciousness, arousal, and maintaining wakefulness .
The RAS receives sensory input and modulates cortical activity , ensuring the brain remains alert and responsive to stimuli.
Why the Correct Answer is Right
✅ Alertness and wakefulness are the primary functions of the RAS.
The RAS stimulates the cerebral cortex , keeping us awake and attentive.
Damage to the RAS (e.g., trauma, stroke, or tumors) can lead to coma or altered states of consciousness .
The thalamus and hypothalamus also interact with the RAS to regulate the sleep-wake cycle .
Why the Other Options Are Incorrect
Control of emotion and behavior – Incorrect
Emotion and behavior are primarily regulated by the limbic system (amygdala, hippocampus, prefrontal cortex) , not the RAS.
Control of autonomic functions – Incorrect
Autonomic functions (heart rate, blood pressure, digestion) are controlled by the hypothalamus and brainstem autonomic centers , not the RAS.
Sensory relay – Incorrect
Sensory relay is a function of the thalamus , which processes and directs sensory information to the cortex.
The RAS modulates sensory perception , but it is not a primary sensory relay center .
Memory – Incorrect
Memory formation and recall are controlled by the hippocampus, limbic system, and cerebral cortex , not the RAS.
“This disorder is characterized by a certain type of movement, resting tremors, and muscle stiffness, rather than excessive or uncontrolled motion.”
96 / 128
“This syndrome is linked to changes in emotional behavior and impulse control, rather than direct impairment of voluntary movement.”
97 / 128
Category:
NeuroSciences – Anatomy
Which of the following is inappropriate regarding Kluver-Bucy syndrome?
Understanding Klüver-Bucy Syndrome
Klüver-Bucy syndrome results from bilateral lesions of the medial temporal lobes , particularly the amygdala .
It is characterized by: ✅ Emotional blunting (lack of fear and emotions, placidity) ✅ Hyperphagia (excessive eating) ✅ Hypersexuality (inappropriate sexual behavior) ✅ Visual agnosia (difficulty recognizing objects visually) ✅ Oral fixation (tendency to put objects in the mouth)
Why the Correct Answer is Wrong (Not a Feature of Klüver-Bucy Syndrome)
✅ Bilateral motor paralysis is NOT a feature of Klüver-Bucy syndrome.
Motor paralysis (inability to move limbs) results from damage to the motor cortex or corticospinal tracts .
Klüver-Bucy syndrome primarily affects emotional and behavioral functions, not voluntary movement.
Why the Other Options Are Correct (Features of Klüver-Bucy Syndrome)
Lack of emotions → Correct
Due to amygdala damage , patients show emotional blunting and reduced fear responses.
Placidity → Correct
Patients become unusually calm and docile , even in stressful situations.
Increased sexual drive → Correct
Hypersexuality is a hallmark of Klüver-Bucy syndrome, leading to inappropriate sexual behavior.
Increased hunger → Correct
Patients experience hyperphagia , leading to excessive eating and oral fixation.
“This descending motor pathway originates from a midbrain structure involved in refining movements, particularly flexion of the upper limbs.”
98 / 128
Category:
NeuroSciences – Anatomy
Which of the following is formed by efferent fibers from the red nucleus to the spinal cord?
Understanding the Rubrospinal Tract
The red nucleus , located in the midbrain (tegmentum of the mesencephalon) , plays a role in motor coordination .
The rubrospinal tract originates from the red nucleus , crosses at the ventral tegmental decussation , and descends to the spinal cord .
This tract modulates flexor muscle tone and fine motor control , particularly in the upper limbs.
Why the Correct Answer is Right
✅ The rubrospinal tract originates from the red nucleus and projects to the spinal cord , influencing movement by exciting flexor muscles and inhibiting extensor muscles.
Why the Other Options Are Incorrect
Corticospinal tract → Incorrect
The corticospinal tract originates from the motor cortex (precentral gyrus) and controls voluntary movement , but it does not arise from the red nucleus .
Olivospinal tract → Incorrect
The olivospinal tract is not a major recognized pathway in humans; the inferior olivary nucleus is primarily involved in cerebellar motor coordination.
Vestibulospinal tract → Incorrect
The vestibulospinal tract originates from the vestibular nuclei (brainstem) and is responsible for postural control and balance , not red nucleus output.
Reticulospinal tract → Incorrect
The reticulospinal tract arises from the reticular formation and is involved in posture and locomotion , but it does not originate from the red nucleus.
“This neurotransmitter plays a key role in mood regulation and is synthesized from an amino acid commonly associated with dietary sources like turkey and bananas.”
99 / 128
“Damage to this region affects communication in a way that the person may not even realize their own difficulty. Consider whether the issue is with speaking, understanding, or another aspect of language processing.”
100 / 128
Category:
NeuroSciences – Anatomy
What signs and symptoms will be observed as a consequence of a lesion in Wernicke’s area?
Understanding Wernicke’s Area and Its Function
Wernicke’s area is located in the superior temporal gyrus of the dominant hemisphere (usually the left hemisphere in right-handed individuals).
It is primarily responsible for language comprehension , including understanding spoken and written words .
A lesion here results in Wernicke’s aphasia , which is characterized by: ✅ Fluent but meaningless speech (word salad) ✅ Poor comprehension of spoken and written language ✅ Difficulty repeating words or sentences ✅ Unawareness of language deficits (anosognosia)
Why the Correct Answer is Right
✅ “Inability to understand written or spoken words” is correct because:
Wernicke’s area is essential for processing and understanding language .
A lesion here causes receptive aphasia , meaning the patient cannot comprehend speech or text but can still produce fluent speech.
Why the Other Options Are Incorrect
“Inability to produce fluent speech” – Incorrect
Speech production is controlled by Broca’s area , located in the frontal lobe .
In Wernicke’s aphasia , speech is fluent but nonsensical , not absent or effortful.
“Inability to appreciate the texture of objects” – Incorrect
Texture perception is controlled by the somatosensory cortex (parietal lobe) , not Wernicke’s area.
“Inability to read or write” – Incorrect
While reading comprehension is affected , writing ability depends on multiple regions, including the angular gyrus and Broca’s area .
Pure alexia (inability to read) occurs due to left occipitotemporal damage , not isolated Wernicke’s area damage.
“Inability to interpret sounds” – Incorrect
Primary auditory processing occurs in the primary auditory cortex (Heschl’s gyrus, temporal lobe) , not Wernicke’s area.
Wernicke’s area specifically processes language , not basic sound perception.
“Consider which cranial nerves are located in the medial versus lateral parts of the medulla, and which functions would remain unaffected in a lateral medullary stroke.”
101 / 128
Category:
NeuroSciences – Anatomy
Ischemia of the dorsal and lateral parts of the medulla will not present with which of the following symptoms?
Understanding Lateral Medullary (Wallenberg) Syndrome
Ischemia affecting the lateral part of the medulla is most commonly due to an infarction in the posterior inferior cerebellar artery (PICA) , leading to lateral medullary syndrome (Wallenberg syndrome) .
The lateral medulla contains: ✅ Spinothalamic tract → Carries pain and temperature sensation from the contralateral body. ✅ Spinal trigeminal nucleus → Carries pain and temperature sensation from the ipsilateral face. ✅ Inferior cerebellar peduncle and vestibular nuclei → Causes gait and limb ataxia due to cerebellar dysfunction. ✅ Nucleus ambiguus (CN IX, X) → Leads to dysarthria, dysphagia, and hoarseness .
Why the Correct Answer is Right
✅ Paralysis of the tongue is NOT a feature of lateral medullary syndrome
The hypoglossal nerve (CN XII) , which controls tongue movement , is located in the medial medulla , not the lateral medulla.
Ischemia of the medial medulla (caused by infarction in the anterior spinal artery ) leads to medial medullary syndrome , which presents with ipsilateral tongue paralysis (CN XII involvement).
Since lateral medullary ischemia does not affect CN XII , paralysis of the tongue is NOT expected in this case.
Why the Other Options Are Incorrect (These Symptoms DO Occur)
Gait and limb ataxia → Occurs due to involvement of the inferior cerebellar peduncle .
Loss of pain → Occurs due to damage to the spinothalamic tract (contralateral) and spinal trigeminal nucleus (ipsilateral face) .
Loss of temperature sensation → Occurs for the same reason as pain loss (spinothalamic tract and trigeminal nucleus involvement).
Dysarthria → Occurs due to nucleus ambiguus (CN IX, X) involvement , leading to difficulty in speech and swallowing.
“These deep brain structures work together to regulate movement, and their dysfunction is linked to disorders like Parkinson’s disease.”
102 / 128
Category:
NeuroSciences – Anatomy
The caudate nucleus, putamen, and the globus pallidus collectively make up which structure?
Understanding the Basal Nuclei (Basal Ganglia)
The basal nuclei (also known as the basal ganglia ) are a group of subcortical gray matter structures that play a crucial role in motor control, procedural learning, and habit formation .
The three primary components of the basal nuclei are: ✅ Caudate nucleus ✅ Putamen ✅ Globus pallidus
The putamen and globus pallidus together form the lentiform nucleus , while the caudate nucleus and putamen together form the striatum .
These structures work together to regulate voluntary movement and prevent unwanted movements, with dysfunction leading to disorders like Parkinson’s disease and Huntington’s disease .
Why the Other Options Are Incorrect
“This structure is hidden deep inside the brain, within the lateral sulcus, rather than being visible on the medial surface.”
103 / 128
Category:
NeuroSciences – Anatomy
Which of the following structures is NOT present on the medial side of the cerebrum?
Understanding the Medial Side of the Cerebrum
The medial side of the cerebrum includes structures primarily located along the longitudinal fissure , which separates the two cerebral hemispheres.
Structures Present on the Medial Side:
✅ Corpus Callosum – The largest white matter tract , connecting the two cerebral hemispheres. ✅ Paracentral Lobule – Located around the central sulcus , involved in motor and sensory functions of the lower limb. ✅ Cuneus – A wedge-shaped area in the occipital lobe , responsible for processing visual information . ✅ Precuneus – Located in the parietal lobe , involved in higher-order cognitive functions .
Structure NOT Present on the Medial Side:
❌ Insula – The insula (insular cortex) is buried deep within the lateral sulcus , between the frontal, temporal, and parietal lobes .
It is not visible on the medial surface of the cerebrum.
It plays roles in taste, visceral sensation, and emotional processing .
Why the Other Options Are Incorrect (These Structures Are Medial)
Corpus Callosum → Medial ; connects the hemispheres.
Paracentral Lobule → Medial ; controls lower limb motor and sensory functions.
Cuneus → Medial ; part of the occipital lobe for visual processing.
Precuneus → Medial ; involved in cognition and visuospatial processing.
“This type of neuron is a major excitatory cell in the cerebral cortex but is not part of the cerebellum’s architecture.”
104 / 128
Category:
Neurosciences – Histology
Which types of cells are not present in the cerebellar cortex?
Understanding the Cellular Composition of the Cerebellar Cortex
The cerebellar cortex consists of three layers:
Molecular Layer (Outer Layer)
Contains stellate cells and basket cells (both inhibitory interneurons).
Contains the dendrites of Purkinje cells and parallel fibers of granule cells .
Purkinje Layer (Middle Layer)
Contains Purkinje cells , which are the sole output neurons of the cerebellar cortex.
Granular Layer (Inner Layer)
Contains granule cells and Golgi cells .
Granule cells are the only excitatory neurons in the cerebellar cortex.
Why the Correct Answer is Right
✅ Pyramidal cells are NOT present in the cerebellar cortex.
Pyramidal cells are excitatory neurons found in the cerebral cortex , particularly in the motor and association areas .
They are NOT part of the cerebellar cortex , which is primarily composed of Purkinje cells and interneurons .
Why the Other Options Are Incorrect (These Cells Are Present in the Cerebellar Cortex)
Stellate cells – Present in the molecular layer ; inhibitory interneurons.
Golgi cells – Present in the granular layer ; inhibitory interneurons that regulate granule cells.
Granule cells – Present in the granular layer ; the only excitatory neurons in the cerebellar cortex.
Basket cells – Present in the molecular layer ; inhibitory interneurons that modulate Purkinje cells.
“These specialized motor neurons are among the largest in the human brain and are found in a layer responsible for sending signals to the spinal cord.”
105 / 128
Category:
NeuroSciences – Anatomy
Which statement is incorrect about pyramidal cell morphology?
Understanding Pyramidal Cell Morphology
Pyramidal cells are large excitatory neurons found in the cerebral cortex , particularly in the motor and association areas . They play a crucial role in corticospinal and corticobulbar pathways .
They have a triangular (pyramidal) cell body with:
A single, long apical dendrite extending toward the pial surface .
Short lateral basal dendrites extending from the base.
An axon that arises from the base and typically projects to subcortical or spinal structures.
Betz cells , a special type of large pyramidal neuron , are found in layer V (deep layer) of the primary motor cortex (precentral gyrus) .
Why the Incorrect Statement is Wrong
✅ “Betz cells are present in the superficial layer” is incorrect because:
Betz cells are NOT in the superficial layer.
They are found in layer V (internal pyramidal layer) of the motor cortex.
The superficial layers (layers I–III) contain smaller pyramidal neurons and association neurons , but not Betz cells .
Why the Other Statements Are Correct
The axon arises from the base of the cell body – Correct
Pyramidal cell axons originate from the base and project either to other cortical areas (association fibers ) or deeper structures (corticospinal or corticobulbar tracts ).
Dendrites from the apex are highly branched and extend to the surface – Correct
The apical dendrite extends toward the pial surface , branching to receive synaptic inputs from other cortical neurons.
Short lateral dendrites extend from the base – Correct
Basal dendrites extend laterally from the base, connecting to nearby neurons within the cortex.
The apices point towards the pial surface of the cortex – Correct
Pyramidal cells are oriented with their apical dendrites extending toward the outer (pial) surface of the cerebral cortex.
“Peripheral nerves can regenerate, but the process depends on factors like injury type and Schwann cell support. The rate is measured in millimeters per day, not in large numbers.”
106 / 128
Category:
Neurosciences – Physiology
What is the rate of nerve regeneration after an injury?
Understanding Nerve Regeneration
After a peripheral nerve injury , axons can regenerate under the right conditions, primarily due to the support of Schwann cells .
The rate of axonal regrowth varies but is generally estimated to be 1–3 mm/day in humans.
CNS nerves (brain and spinal cord) do not regenerate effectively due to inhibitory factors in the myelin and glial scar formation.
Why the Correct Answer is Right
✅ Peripheral nerve regeneration occurs at a rate of approximately 1–3 mm/day.
This rate depends on age, health status, severity of injury, and distance from the injury site .
Crushed nerves regenerate faster than severed nerves , as the existing pathway guides regrowth.
The process involves Wallerian degeneration , where the distal portion of the axon degenerates while Schwann cells promote regrowth.
Why the Incorrect Options Are Wrong
100 mm/day – Incorrect
This is far too fast and unrealistic for nerve regeneration.
Even the fastest regenerating axons (like small sensory nerves) do not exceed 5 mm/day .
200 mm/day – Incorrect
No biological system supports such an extreme rate of axonal regrowth.
This rate is not physiologically possible .
8 mm/day – Incorrect
While regeneration speeds can vary , 8 mm/day is beyond the typical observed rate in humans.
0.1 mm/day – Incorrect
This rate is too slow for peripheral nerves.
However, CNS neurons have extremely limited regrowth potential and may have near-zero regeneration rates .
“This patient’s symptoms suggest an injury affecting lower limb function and bladder control. Which spinal region regulates these functions?”
107 / 128
Category:
Neuroscience – Radiology
A 36-year-old man fell from a height of 10 feet. As a consequence, he now suffers from incontinence and weakness in the lower limbs. Which of the following statements is correct?
Understanding the Clinical Scenario
This patient presents with:
Lower limb weakness
Incontinence (suggesting involvement of the sacral spinal cord or its pathways )
History of trauma (fall from height)
Given this combination of neurological and bladder dysfunction , a spinal cord injury at the lumbosacral level is the most likely cause. Imaging of the lumbosacral region (MRI or CT scan) is essential to assess spinal cord compression, fractures, or nerve root injury.
Why the Other Options Are Incorrect
The middle cerebral artery is involved – Incorrect
The middle cerebral artery (MCA) supplies the lateral aspects of the frontal, temporal, and parietal lobes .
MCA infarcts typically cause contralateral upper limb and facial weakness , aphasia (if dominant hemisphere is affected) , or neglect (if non-dominant hemisphere is involved) .
It does NOT cause isolated lower limb weakness or incontinence , as the medial frontal lobe (leg motor area) is supplied by the anterior cerebral artery (ACA) .
The posterior cerebral artery is involved – Incorrect
The posterior cerebral artery (PCA) supplies the occipital lobe, thalamus, and parts of the midbrain .
PCA strokes result in visual deficits (homonymous hemianopia), thalamic pain syndrome, and sometimes memory impairment .
It does not cause lower limb weakness or incontinence .
Involvement of cervical spine C4 level – Incorrect
The C4 spinal level is located in the cervical spine and controls diaphragm function (via the phrenic nerve) .
Injury here would result in respiratory compromise, upper limb weakness, and quadriplegia , not isolated lower limb weakness and incontinence .
Imaging of the dorsal spine should be done – Incorrect
While the thoracic spine (dorsal spine) is important for motor control of the lower limbs , the bladder and bowel functions are controlled by the sacral segments (S2-S4) .
If the patient has incontinence , the injury is more likely in the lumbosacral region than in the dorsal (thoracic) spine .
“These brain waves are most prominent when you’re awake but relaxed, often with your eyes closed, before transitioning into sleep.”
108 / 128
Category:
Neurosciences – Physiology
What is the frequency range of alpha waves?
Understanding Brain Waves and Their Frequencies
Brain waves are classified based on their frequency (measured in cycles per second or Hz). Each type of wave is associated with specific mental states:
Delta Waves (Less than 3.5 Hz)
Slowest waves, associated with deep sleep and unconscious states .
Theta Waves (4-7 Hz)
Linked to light sleep, deep relaxation, and meditation .
Alpha Waves (8-13 Hz)
Seen in relaxed, wakeful states , such as when the eyes are closed but the person is awake.
Commonly detected in the occipital region during resting wakefulness .
Beta Waves (14-30 Hz)
Associated with active thinking, problem-solving, and focused mental activities .
Gamma Waves (35-140 Hz)
The fastest waves, linked to high-level cognitive processing, attention, and perception .
Since alpha waves have a frequency range of 8-13 Hz , the correct answer is 8-13 cycles per second .
Why the Other Options Are Incorrect
14-80 cycles per second → Incorrect
This range includes beta waves (14-30 Hz) and gamma waves (35-140 Hz) , which are associated with active thinking and cognition , not relaxation.
Less than 3.5 cycles per second → Incorrect
This corresponds to delta waves , which occur in deep sleep and unconscious states .
4-7 cycles per second → Incorrect
This corresponds to theta waves , which occur in light sleep and deep relaxation .
35-140 cycles per second → Incorrect
This range corresponds to gamma waves , which are associated with high-level cognition and sensory processing .
“This part of the lateral ventricle lies in the frontal lobe and is closely related to the head of a structure involved in motor and cognitive functions.”
109 / 128
Category:
NeuroSciences – Anatomy
A newborn baby was brought to the neurology outpatient department because the large size of the fetal head-MRI shows fluid in the lateral ventricle. The appropriate part of the lateral ventricle formed by the head of the caudate nucleus is?
Understanding the Relationship Between the Caudate Nucleus and the Lateral Ventricle
The lateral ventricles are C-shaped cavities within the cerebral hemispheres that contain cerebrospinal fluid (CSF). Each lateral ventricle has four major parts:
Anterior horn (frontal lobe)
Body (parietal lobe)
Posterior horn (occipital lobe)
Inferior horn (temporal lobe)
The caudate nucleus is a C-shaped structure that runs along the lateral ventricle. It has three main parts:
Head → Located in the anterior horn of the lateral ventricle.
Body → Lies along the body of the ventricle .
Tail → Extends into the inferior horn .
Since the head of the caudate nucleus is specifically located in the anterior horn of the lateral ventricle , that is the correct answer.
Why the Other Options Are Incorrect
Medial horn – Incorrect
There is no medial horn in the lateral ventricle. This is not an anatomical term used for ventricular anatomy.
Posterior horn – Incorrect
The posterior horn is located in the occipital lobe and is not related to the head of the caudate nucleus.
Inferior horn – Incorrect
The inferior horn extends into the temporal lobe . The tail of the caudate nucleus is found here, not the head .
Superior horn – Incorrect
There is no superior horn in the lateral ventricle. This is not an anatomical term used for ventricular anatomy.
Think of VMH as the body’s appetite brake —if damaged, the brakes fail, leading to uncontrolled eating and obesity .
110 / 128
Category:
Neurosciences – Physiology
An obese woman suffers from a mid-hypothalamic lesion. Which of the following nuclei is involved?
The ventromedial hypothalamic nucleus (VMH) plays a crucial role in satiety and appetite regulation . It acts as the “satiety center” , signaling the body to stop eating.
Lesion of VMH → Leads to hyperphagia (excessive eating) and obesity .
Stimulation of VMH → Suppresses appetite and reduces body weight.
A mid-hypothalamic lesion affecting the ventromedial hypothalamic nucleus disrupts this balance, causing uncontrolled eating and weight gain .
Why Others Are Incorrect?
❌ Supraoptic nucleus:
Controls ADH (vasopressin) and oxytocin release .
Lesions cause diabetes insipidus , not obesity.
❌ Posterior hypothalamic nucleus:
Regulates heat conservation and sympathetic responses .
Lesions cause hypothermia , not hyperphagia.
❌ Preoptic nucleus:
Controls thermoregulation and reproductive hormone release .
Lesions cause temperature dysregulation , not obesity.
❌ Suprachiasmatic nucleus:
Regulates circadian rhythms (sleep-wake cycle).
Lesions cause sleep disturbances , not hyperphagia.
Think “A” for Alar, Afferent (sensory), and Above (dorsal side of the spinal cord) —it processes incoming signals .
111 / 128
Category:
Neurosciences – Embryology
Which one of the following is formed by cells in the alar plate?
embryonic development, the neural tube differentiates into two key regions:
1️⃣ Alar plate (dorsal) → Sensory functions 2️⃣ Basal plate (ventral) → Motor functions
The alar plate gives rise to sensory structures , including the posterior (dorsal) gray horn of the spinal cord, which processes sensory input (pain, touch, temperature).
Why Others Are Incorrect?
❌ Autonomic ganglia:
Derived from neural crest cells , not the neural tube.
❌ None of these:
Incorrect, as posterior gray horn is formed by the alar plate .
❌ Motor neurons:
Derived from the basal plate , not the alar plate.
❌ Anterior gray horn:
Also derived from the basal plate , which forms motor neurons .
Have you ever felt your foot “fall asleep” and then experience a tingling, buzzing, or prickling feeling as it “wakes up”? That’s paresthesia !
112 / 128
Category:
Neurosciences – Physiology
What is meant by paresthesia?
Paresthesia refers to an abnormal sensory experience described as “pins and needles,” tingling, burning, or numbness . It results from nerve dysfunction or compression .
Causes:
Nerve compression or ischemia (e.g., sitting too long in one position)
Neuropathy (e.g., diabetes)
Vitamin deficiency (e.g., B12)
Nerve injury
Why Others Are Incorrect?
❌ Loss of sensitivity: This is anesthesia , not paresthesia.
❌ Loss of pain sensation: This is analgesia , linked to nerve damage or anesthesia.
❌ Decreased sensitivity: This is hypoesthesia , partial loss of sensation, not tingling.
❌ Increased sensitivity: This is hyperesthesia/allodynia , where normal touch feels painful.
“This amino acid serves as the backbone for neurotransmitters that regulate mood, alertness, and the fight-or-flight response.”
113 / 128
Category:
Neurosciences – Biochemistry
Which of the following amino acids is required for the synthesis of dopamine, epinephrine and norepinephrine?
Understanding Catecholamine Synthesis
Dopamine, epinephrine, and norepinephrine belong to a group of neurotransmitters called catecholamines , which are synthesized from the amino acid tyrosine . The pathway follows these steps:
Tyrosine → (Tyrosine hydroxylase) → L-DOPA
L-DOPA → (Dopa decarboxylase) → Dopamine
Dopamine → (Dopamine β-hydroxylase) → Norepinephrine
Norepinephrine → (Phenylethanolamine-N-methyltransferase) → Epinephrine
Thus, tyrosine is the essential precursor for all three catecholamines.
Why the Other Options Are Incorrect
Valine is a branched-chain amino acid (BCAA) involved in muscle metabolism and energy production. It is not involved in catecholamine synthesis.
Methionine is involved in methylation reactions via S-adenosylmethionine (SAM) . While SAM contributes to the conversion of norepinephrine to epinephrine , methionine is not the precursor of dopamine or norepinephrine .
Glutamate is a key excitatory neurotransmitter and is the precursor of GABA , but not of dopamine, epinephrine, or norepinephrine.
Tryptophan is the precursor for serotonin and melatonin , not catecholamines.
“This structure is responsible for fine motor coordination and balance. Its cortex is highly organized but differs in complexity from the cerebral cortex.”
114 / 128
Category:
NeuroSciences – Anatomy
Which one of the following is not true about the cerebellar cortex?
Understanding the Cerebellar Cortex
The cerebellar cortex is highly organized and consists of three distinct layers , not six. These layers are:
Molecular Layer (Outer Layer)
Contains dendrites of Purkinje cells , stellate cells , and basket cells .
Receives input from parallel fibers of granule cells .
Purkinje Layer (Middle Layer)
Contains a single row of Purkinje cells , which are the only output neurons of the cerebellar cortex .
Their axons project to the deep cerebellar nuclei , sending inhibitory signals.
Granular Layer (Inner Layer)
Contains granule cells , the most numerous neurons in the brain .
Receives input from mossy fibers , which originate from different parts of the CNS.
Why the Correct Option is Wrong (False Statement)
✅ “It has six distinct layers” is false because:
Unlike the cerebral cortex , which has six layers , the cerebellar cortex only has three layers .
The three-layered structure is a defining feature of the cerebellum’s histology.
Why the Other Options Are Correct (True Statements)
It has a middle Purkinje layer – True
The Purkinje layer is the middle layer of the cerebellar cortex.
It contains a single layer of Purkinje cells , which serve as the primary output neurons of the cerebellum .
It has three distinct layers – True
The cerebellar cortex has three layers :
Molecular layer (outer)
Purkinje layer (middle)
Granular layer (inner)
It has an inner granular layer – True
The granular layer is the deepest (innermost) layer of the cerebellar cortex.
It contains granule cells , which excite Purkinje cells via parallel fibers .
It has an outer molecular layer – True
The molecular layer is the outermost layer of the cerebellar cortex.
It contains dendrites of Purkinje cells, parallel fibers of granule cells, and inhibitory interneurons (stellate and basket cells) .
“This structure acts as the brain’s central relay station for nearly all sensory signals, except smell, before they reach the cerebral cortex.”
115 / 128
Category:
NeuroSciences – Anatomy
Which one of the following structures is the largest component of the diencephalon?
Understanding the Diencephalon
The diencephalon is a part of the forebrain (prosencephalon) and is located between the cerebral hemispheres and midbrain . It consists of four main components:
Thalamus – The largest component, primarily involved in sensory relay and integration .
Hypothalamus – Controls autonomic functions, hormone regulation, and homeostasis .
Epithalamus – Includes the pineal gland , which regulates circadian rhythms and melatonin production .
Subthalamus – Functionally related to the basal ganglia , involved in motor control .
Why the Correct Answer is Right
✅ Thalamus – The Largest Component of the Diencephalon
The thalamus accounts for about 80% of the diencephalon .
It serves as the major relay center for sensory and motor signals traveling to the cerebral cortex.
Almost all sensory information (except olfaction) is processed in the thalamus before reaching the cortex.
It is divided into several nuclei, each with specific functions related to sensory processing, motor control, and consciousness regulation .
Why the Incorrect Options Are Wrong
A) Epithalamus – Incorrect
The epithalamus is a small structure containing the pineal gland and habenular nuclei .
It regulates melatonin secretion and circadian rhythms , but it is not the largest part of the diencephalon.
B) Subthalamus – Incorrect
The subthalamus is small and functionally associated with the basal ganglia , playing a role in motor control .
It is not the largest structure of the diencephalon.
D) Hypothalamus – Incorrect
The hypothalamus is crucial for autonomic control, hormone regulation, and homeostasis , but it is smaller than the thalamus .
It connects to the pituitary gland and regulates functions like temperature control, hunger, thirst, and circadian rhythms .
E) None of these – Incorrect
One of the given options (Thalamus ) is indeed the largest part of the diencephalon, so this option is incorrect.
“Think about which part of the hypothalamus is responsible for parasympathetic activation—slowing the heart and promoting digestion and urination. It’s also involved in cooling down the body when overheated.”
116 / 128
Category:
Neurosciences – Physiology
Which hypothalamic nuclei are responsible for decreasing heart rate and blood pressure, and increasing the contraction of the urinary bladder?
Understanding the Role of the Hypothalamus in Autonomic Regulation
The hypothalamus plays a crucial role in autonomic nervous system (ANS) regulation, influencing both the sympathetic and parasympathetic nervous systems.
The anterior and preoptic nuclei are associated with parasympathetic functions, which reduce heart rate and blood pressure and promote bladder contraction .
These nuclei work by activating parasympathetic outflow from the brainstem and sacral spinal cord (S2-S4) .
Why the Correct Answer is Right
✅ Anterior and Preoptic Nuclei → Parasympathetic Regulation
The anterior nucleus is involved in cooling mechanisms and parasympathetic activation (rest and digest functions).
The preoptic nucleus regulates autonomic functions, including slowing the heart rate and increasing bladder contraction (via the pelvic splanchnic nerves).
Together, these nuclei contribute to vasodilation, decreased blood pressure, decreased heart rate (bradycardia), and increased detrusor muscle contraction (for urination).
Why the Incorrect Options Are Wrong
A) Ventromedial Nucleus – Incorrect
This nucleus is primarily involved in satiety control and appetite suppression .
It does not play a significant role in autonomic regulation of heart rate or bladder function.
C) Supraoptic and Paraventricular Nuclei – Incorrect
These nuclei control fluid balance and hormone secretion rather than autonomic function.
They produce vasopressin (ADH) and oxytocin , which are related to water retention and uterine contractions , not heart rate or bladder control.
D) Lateral Hypothalamic Nucleus – Incorrect
The lateral hypothalamic nucleus is associated with hunger and feeding behavior .
It stimulates food intake and has no significant role in heart rate or bladder contraction.
E) Posterior and Lateral Nuclei – Incorrect
These nuclei are involved in sympathetic activation (opposite to what the question is asking).
They increase heart rate and blood pressure and are responsible for heat conservation (shivering and vasoconstriction), not parasympathetic functions.
“This function helps to protect and stimulate vital organs during stress, ensuring survival in a fight-or-flight situation.”
117 / 128
Category:
Neurosciences – Physiology
Which one of the following is a function of the sympathetic nervous system?
Understanding the Sympathetic Nervous System (SNS)
The sympathetic nervous system (SNS) is part of the autonomic nervous system (ANS) and is responsible for the “fight-or-flight” response . It prepares the body for stressful or emergency situations by:
Increasing heart rate and cardiac output
Dilating pupils (to allow more light for better vision)
Decreasing digestive functions (to conserve energy)
Promoting vasoconstriction in certain blood vessels (to redirect blood to essential organs like the brain and muscles)
Why the Correct Answer is Right
✅ Vasoconstriction (A)
The sympathetic nervous system causes vasoconstriction in most blood vessels via activation of α1-adrenergic receptors .
This helps increase blood pressure and redirect blood flow to vital organs during stress or emergencies.
However, it can also cause vasodilation in skeletal muscles via β2 receptors to improve oxygen supply for physical activity.
Why the Incorrect Options Are Wrong
B) Decreased heart rate – Incorrect
The parasympathetic nervous system (PNS) is responsible for decreasing heart rate , not the SNS.
The SNS actually increases heart rate by stimulating β1-adrenergic receptors in the heart.
C) Increased GIT motility – Incorrect
The parasympathetic nervous system promotes GIT motility and digestion.
The sympathetic nervous system inhibits GIT motility to conserve energy for emergency responses.
D) Defecation – Incorrect
Defecation is controlled by the parasympathetic nervous system , which increases peristalsis and relaxation of the internal anal sphincter .
The sympathetic system actually inhibits defecation by contracting the internal anal sphincter .
E) Pupil constriction – Incorrect
The sympathetic nervous system causes pupil dilation (mydriasis) via α1-adrenergic receptor activation in the iris.
Pupil constriction (miosis) is a parasympathetic function , mediated by the oculomotor nerve (CN III) .
“This structure is part of the basal ganglia and plays a crucial role in the indirect pathway of movement control. Damage here leads to a failure of movement inhibition, resulting in slow, writhing, involuntary motions.”
118 / 128
Category:
NeuroSciences – Anatomy
A lesion in which structure will cause athetosis?
A lesion in the globus pallidus (especially the external segment, GPe ) disrupts the indirect pathway of the basal ganglia , leading to a loss of inhibitory control over unwanted movements. This results in athetosis , characterized by slow, involuntary, writhing movements of the hands, fingers, and sometimes the face.
Why the Globus Pallidus is the Key Structure
The basal ganglia control voluntary movement through two pathways:
Direct Pathway → Facilitates movement
Indirect Pathway → Inhibits movement
The external segment of the globus pallidus (GPe) plays a critical role in the indirect pathway , which normally suppresses unwanted movements.
A lesion in the GPe disrupts this inhibition , causing excessive, involuntary movements seen in athetosis.
This is why athetosis is often seen in diseases that affect the globus pallidus, like perinatal hypoxic-ischemic injury (leading to choreoathetosis in cerebral palsy).
Why the Other Options Are Wrong
A) Subthalamus – Incorrect
A lesion in the subthalamic nucleus leads to hemiballismus (violent, flinging movements of one limb), not athetosis.
B) Substantia Nigra – Incorrect
Damage to the substantia nigra is primarily linked to Parkinson’s disease , leading to rigidity and bradykinesia rather than athetosis.
D) Caudate Nucleus – Incorrect
The caudate nucleus is involved in cognitive and motor function.
Its damage is more commonly associated with Huntington’s disease , which presents with chorea (sudden, jerky movements), not athetosis.
E) Putamen – Incorrect
The putamen is important in movement regulation, but isolated lesions here are more associated with dystonia rather than athetosis .
While the putamen and globus pallidus work together , athetosis is more directly linked to the globus pallidus , especially GPe dysfunction .
“Dural folds are made of dura mater and help support brain structures, while this structure is part of the endocrine system and regulates hormones.”
119 / 128
Category:
NeuroSciences – Anatomy
Which one of the following is not a dural fold?
Understanding Dural Folds
Dural folds (also known as dural septa ) are extensions of the dura mater that help compartmentalize the brain and provide structural support. These folds help stabilize the brain within the cranial cavity and house major venous sinuses .
The four major dural folds include:
Falx cerebri – Separates the two cerebral hemispheres.
Tentorium cerebelli – Separates the cerebrum from the cerebellum.
Falx cerebelli – Separates the two cerebellar hemispheres.
Diaphragma sellae – Covers the hypophyseal fossa and has an opening for the pituitary stalk.
Since all the above structures are dural folds , the odd one out is hypophysis cerebri .
Why the Incorrect Options are Actually Dural Folds
A) Diaphragma sellae – Incorrect
The diaphragma sellae is a small dural fold covering the sella turcica , which houses the pituitary gland (hypophysis cerebri) .
It has a small opening for the infundibulum (pituitary stalk), allowing communication between the hypothalamus and pituitary gland.
B) Falx cerebri – Incorrect
The falx cerebri is a large, sickle-shaped dural fold that extends between the two cerebral hemispheres .
It attaches to the crista galli of the ethmoid bone and contains the superior and inferior sagittal sinuses .
C) Tentorium cerebelli – Incorrect
The tentorium cerebelli is a dural fold that separates the cerebrum from the cerebellum .
It forms a tent-like structure and contains the transverse sinuses .
D) Falx cerebelli – Incorrect
The falx cerebelli is a dural fold that separates the two cerebellar hemispheres .
It is smaller than the falx cerebri and contains parts of the occipital sinus .
Why the Correct Option is NOT a Dural Fold
E) Hypophysis cerebri – Correct Answer
Hypophysis cerebri is another name for the pituitary gland , which is an endocrine gland located within the sella turcica of the sphenoid bone.
It is not a dural fold , but it is covered by the diaphragma sellae , which is a dural fold.
“This foramen serves as the passageway for a nerve branch responsible for sensory innervation of the upper jaw, including the cheeks, nasal cavity, and upper teeth.”
120 / 128
Category:
NeuroSciences – Anatomy
Which one of the following nerves passes through foramen rotundum?
Understanding the Foramen Rotundum and Its Contents
The foramen rotundum is an opening in the greater wing of the sphenoid bone that serves as a passageway for the maxillary nerve (V2) , a branch of the trigeminal nerve (cranial nerve V) .
The trigeminal nerve (CN V) has three major divisions:
Ophthalmic nerve (V1) – Passes through the superior orbital fissure
Maxillary nerve (V2) – Passes through the foramen rotundum
Mandibular nerve (V3) – Passes through the foramen ovale
The maxillary nerve (V2) is responsible for sensory innervation of the midface, upper teeth, nasal cavity, and palate .
Why the Incorrect Options are Wrong
A) Lesser petrosal nerve – Incorrect
The lesser petrosal nerve carries parasympathetic fibers from the glossopharyngeal nerve (CN IX) to the otic ganglion , which then innervates the parotid gland.
It does not pass through the foramen rotundum; instead, it passes through the foramen ovale .
B) Trigeminal nerve – Incorrect
The trigeminal nerve (CN V) as a whole does not pass through a single foramen; rather, it divides into three branches , each passing through a different foramen:
V1 (Ophthalmic nerve) → Superior orbital fissure
V2 (Maxillary nerve) → Foramen rotundum
V3 (Mandibular nerve) → Foramen ovale
Since only the maxillary division (V2) passes through the foramen rotundum , the best answer is maxillary nerve rather than the entire trigeminal nerve.
C) Vagus nerve – Incorrect
The vagus nerve (CN X) exits the skull via the jugular foramen , not the foramen rotundum.
It primarily provides parasympathetic innervation to thoracic and abdominal organs.
D) Mandibular nerve – Incorrect
The mandibular nerve (V3) , another branch of the trigeminal nerve, exits through the foramen ovale , not the foramen rotundum.
It provides both sensory and motor functions, including motor control of the muscles of mastication .
“The cerebellum originates from a structure within the hindbrain that also forms a bridge-like structure connecting the brainstem and higher centers.”
121 / 128
Category:
Neurosciences – Embryology
What is the cerebellum embryologically derived from?
Understanding the Embryological Development of the Cerebellum
During early embryonic development, the neural tube gives rise to three primary brain vesicles:
Prosencephalon (Forebrain) → Develops into the Telencephalon and Diencephalon
Mesencephalon (Midbrain) → Remains as the Midbrain
Rhombencephalon (Hindbrain) → Develops into the Metencephalon and Myelencephalon
The metencephalon , a subdivision of the rhombencephalon (hindbrain) , gives rise to two major structures:
The cerebellum originates from the dorsal aspect of the metencephalon , specifically from the alar plate of the neural tube.
Why the Correct Option is Right
The metencephalon is the precursor to the cerebellum and pons .
It arises from the rhombencephalon , but specifically differentiates into the cerebellum, making metencephalon the most precise answer.
Why the Incorrect Options are Wrong
A) Rhombencephalon – Incorrect
The rhombencephalon (hindbrain) is the larger embryological division that gives rise to both the metencephalon (pons and cerebellum) and the myelencephalon (medulla oblongata).
While the cerebellum is derived from the rhombencephalon, the more precise answer is the metencephalon .
B) Telencephalon – Incorrect
The telencephalon arises from the prosencephalon (forebrain) and gives rise to the cerebral cortex, basal ganglia, and limbic structures .
It has no contribution to the formation of the cerebellum.
C) Diencephalon – Incorrect
The diencephalon , which also develops from the prosencephalon (forebrain) , forms the thalamus, hypothalamus, and epithalamus .
It does not contribute to cerebellar development.
D) Myelencephalon – Incorrect
The myelencephalon , a part of the rhombencephalon , develops into the medulla oblongata .
It does not contribute to the formation of the cerebellum.
“Think about the part of the brain that helps refine and regulate movements without initiating them. It plays a key role in disorders like Parkinson’s disease and Huntington’s disease.”
122 / 128
Category:
NeuroSciences – Anatomy
Which of the following is the corpus striatum a part of?
Understanding the Corpus Striatum and Its Role
The corpus striatum is a major component of the basal ganglia , which plays a critical role in motor control, procedural learning, habit formation, and certain cognitive and emotional functions. It is composed of:
Caudate nucleus
Putamen
Globus pallidus (which is sometimes classified separately but is functionally related)
These structures work together to regulate voluntary motor movements and coordination by modulating input from the cerebral cortex and influencing the activity of the thalamus.
Why the Correct Option is Right
The basal ganglia is a collection of nuclei located deep within the cerebral hemispheres. It is primarily involved in movement regulation and coordination.
The corpus striatum specifically refers to the caudate nucleus and putamen , which are integral parts of the basal ganglia.
The basal ganglia receives input from the cerebral cortex and sends output to the thalamus, influencing motor pathways and behaviors.
Why the Incorrect Options are Wrong
B) Subthalamus – Incorrect
The subthalamus is a separate structure located below the thalamus and plays a role in modulating motor activity.
While it is functionally connected to the basal ganglia, it is not part of the corpus striatum.
C) Limbic System – Incorrect
The limbic system is primarily associated with emotions, memory, and behavior. It includes structures such as the amygdala, hippocampus, and cingulate gyrus.
The corpus striatum is not part of the limbic system, as its primary function is motor control rather than emotional regulation.
D) Reticular Formation – Incorrect
The reticular formation is a network of interconnected neurons located in the brainstem, responsible for arousal, sleep-wake cycles, and autonomic functions.
It has no direct role in the motor functions controlled by the corpus striatum.
E) Thalamus – Incorrect
The thalamus is the relay center for sensory and motor signals between the cerebral cortex and other parts of the brain.
While it interacts with the basal ganglia, it is a distinct structure and not part of the corpus striatum.
Which part of the brain controls voluntary movement and contains the caudate nucleus, putamen, and globus pallidus ?
123 / 128
Category:
NeuroSciences – Anatomy
The corpus striatum is a major part of the basal ganglia , which is responsible for regulating motor control, procedural learning, and habit formation .
The corpus striatum consists of: ✅ Caudate nucleus ✅ Putamen ✅ Globus pallidus (both internal and external segments)
The functional subdivisions of the basal ganglia include:
Striatum (Caudate + Putamen) → Receives input from the cerebral cortex.
Globus Pallidus (Internal & External) → Main output structure to the thalamus.
The basal ganglia modulate voluntary movements through the direct and indirect pathways , working closely with the thalamus and motor cortex .
Why the Other Options Are Wrong:
Subthalamus (Incorrect )
The subthalamic nucleus is part of the diencephalon , not the corpus striatum.
It is involved in the indirect motor pathway of the basal ganglia.
Limbic System (Incorrect )
The limbic system regulates emotion, memory, and motivation .
It includes structures like the hippocampus, amygdala, and cingulate gyrus , not the corpus striatum .
Reticular Formation (Incorrect )
The reticular formation is located in the brainstem and is responsible for wakefulness, consciousness, and autonomic control .
It does not involve the corpus striatum .
Thalamus (Incorrect )
The thalamus is a sensory relay center in the diencephalon , which relays information to the cerebral cortex.
It is functionally connected to the basal ganglia , but it is not a part of the corpus striatum .
Hint:
Which cranial nerve controls most of the extraocular muscles and has its nucleus located in the midbrain at the level of the superior colliculus ?
124 / 128
Category:
NeuroSciences – Anatomy
Which cranial nerve nucleus is present at the level of the superior colliculus?
The oculomotor nucleus (CN III) is located at the level of the superior colliculus in the midbrain . It is responsible for controlling most of the extraocular muscles , which facilitate eye movement.
The oculomotor nucleus gives rise to the oculomotor nerve (CN III) , which innervates: ✅ Superior rectus ✅ Inferior rectus ✅ Medial rectus ✅ Inferior oblique ✅ Levator palpebrae superioris (eyelid elevation)
Additionally, the Edinger-Westphal nucleus (which provides parasympathetic fibers to the eye for pupil constriction and accommodation ) is located adjacent to the oculomotor nucleus at this level.
Why the Other Options Are Wrong:
Trochlear Nucleus (CN IV) (Incorrect )
The trochlear nucleus is located at the level of the inferior colliculus , not the superior colliculus.
It gives rise to the trochlear nerve (CN IV) , which innervates the superior oblique muscle .
Abducens Nucleus (CN VI) (Incorrect )
The abducens nucleus is located in the pons , at the level of the facial colliculus , not the midbrain.
It controls the lateral rectus muscle , allowing for lateral eye movement.
Dorsal Nucleus of Vagus (CN X) (Incorrect )
The dorsal motor nucleus of the vagus is found in the medulla , not in the midbrain.
It provides parasympathetic output to thoracic and abdominal organs .
Facial Motor Nucleus (CN VII) (Incorrect )
The facial motor nucleus is located in the pons , not the midbrain.
It controls facial expression muscles .
Which trisomy syndrome presents with midline facial defects and holoprosencephaly ?
125 / 128
Category:
Neurosciences – Embryology
Holoprosencephaly is a malformation characterized by an inability of the cerebral hemispheres to separate at the midline. Which genetic disorder is this malformation associated with?
Holoprosencephaly (HPE) is a congenital brain malformation caused by failure of the forebrain (prosencephalon) to properly divide into two cerebral hemispheres during early embryonic development (weeks 4–6).
This malformation is strongly associated with : ✅ Trisomy 13 (Patau syndrome) ✅ Mutations in SHH (Sonic Hedgehog) gene
Trisomy 13 (Patau syndrome) presents with:
Holoprosencephaly
Midline facial defects (e.g., cleft lip/palate, cyclopia, hypotelorism)
Microcephaly
Severe intellectual disability
Polydactyly and congenital heart defects
Why the Other Options Are Wrong:
Trisomy 21 (Down Syndrome) (Incorrect )
Down syndrome is associated with brachycephaly, intellectual disability, epicanthal folds, and congenital heart defects (AV canal defects) .
Holoprosencephaly is NOT a characteristic feature.
Trisomy 17 (Incorrect )
No well-defined syndrome linked to Trisomy 17 .
It is usually not viable in humans .
Trisomy 15 (Incorrect )
Trisomy 15 is rare and typically lethal.
Abnormalities of chromosome 15 are more commonly associated with Prader-Willi and Angelman syndromes , not holoprosencephaly.
Trisomy 18 (Edwards Syndrome) (Incorrect )
Trisomy 18 is associated with:
Micrognathia, clenched hands, rocker-bottom feet, congenital heart defects
Severe intellectual disability
Holoprosencephaly is NOT a common feature.
Which spinal cord syndrome presents with ipsilateral motor and proprioception loss but contralateral pain and temperature loss ?
126 / 128
Category:
NeuroSciences – Anatomy
What is another name for the hemisection of the spinal cord?
Brown-Séquard syndrome (BSS) is a hemisection (one-sided injury) of the spinal cord , typically due to trauma, tumor, or ischemia . This leads to a characteristic pattern of neurological deficits due to the involvement of both ascending sensory and descending motor pathways .
Clinical Features of Brown-Séquard Syndrome:
Side of Lesion
Deficit
Pathway Affected
Ipsilateral (same side)
Motor paralysis
Corticospinal tract (descending motor pathway)
Ipsilateral (same side)
Loss of vibration & proprioception
Dorsal column-medial lemniscus pathway
Contralateral (opposite side)
Loss of pain & temperature sensation
Spinothalamic tract (crosses in spinal cord)
Motor loss (weakness/paralysis) occurs below the level of the lesion on the same side due to corticospinal tract damage .
Loss of proprioception & vibration is ipsilateral because dorsal columns do not cross until the medulla .
Loss of pain & temperature occurs contralaterally because spinothalamic fibers cross at the level of entry in the spinal cord .
Why the Other Options Are Wrong:
Weber Syndrome (Incorrect )
Midbrain stroke syndrome affecting the crus cerebri and CN III .
Causes ipsilateral oculomotor palsy and contralateral hemiparesis .
Syringomyelia (Incorrect )
Cystic cavity formation in the spinal cord , typically in the cervical region .
Causes bilateral loss of pain & temperature in a “cape-like” distribution due to damage to the spinothalamic tract .
Wallenberg Syndrome (Lateral Medullary Syndrome) (Incorrect )
Posterior inferior cerebellar artery (PICA) infarct , affecting the lateral medulla .
Symptoms include ipsilateral facial pain/temp loss , contralateral body pain/temp loss , and ataxia .
Benedikt Syndrome (Incorrect )
Midbrain infarct affecting the red nucleus and oculomotor nerve .
Leads to contralateral ataxia/tremor and ipsilateral CN III palsy .
Which viral encephalitis is known for temporal lobe involvement, hemorrhagic CSF, and lymphocytosis ?
127 / 128
Category:
Neurosciences – Pathology
A 26-year-old man comes to the outpatient department (OPD) with a high fever and cold sores around the mouth. Lab investigations reveal lymphocytosis, red blood cells (RBCs), and normal glucose in the cerebrospinal fluid (CSF). What is the most likely diagnosis?
The patient’s high fever, cold sores around the mouth (herpetic lesions), and cerebrospinal fluid (CSF) findings strongly suggest herpes simplex virus (HSV) encephalitis , which is the most common sporadic viral encephalitis .
Key Diagnostic Clues:
✅ Cold sores (Herpes labialis) → Suggests HSV infection. ✅ CSF Findings :
Lymphocytosis → Suggests viral (aseptic) encephalitis.
RBCs in CSF → HSV encephalitis causes hemorrhagic necrosis , leading to RBCs in the CSF.
Normal glucose → Rules out bacterial or fungal infections, which typically lower CSF glucose.
✅ Affected Brain Regions :
HSV encephalitis commonly affects the temporal lobes , leading to symptoms such as:
Fever
Altered mental status (confusion, memory loss)
Seizures
Personality changes
🔬 Definitive Diagnosis : PCR for HSV DNA in CSF
📌 Treatment : IV Acyclovir (ASAP!) to reduce mortality.
Why the Other Options Are Wrong:
Fungal Encephalitis (Incorrect )
Fungal infections (e.g., Cryptococcus, Candida, Aspergillus) are more common in immunocompromised patients .
CSF findings : Typically show low glucose , high protein, and lymphocytosis.
West Nile Fever (Incorrect )
West Nile virus can cause encephalitis but typically presents with flaccid paralysis, tremors, and extrapyramidal symptoms .
CSF does not usually show RBCs .
Bacterial Encephalitis (Incorrect )
Bacterial infections usually cause meningoencephalitis , with neutrophilic CSF response.
CSF glucose is typically low , and protein is elevated .
RBCs in CSF are not a common feature .
Protozoal Encephalitis (Incorrect )
Protozoal causes (e.g., Toxoplasmosis, Naegleria fowleri) usually affect immunocompromised individuals or occur due to contaminated water exposure .
Naegleria causes fulminant meningoencephalitis , with rapid deterioration and coma .
How often is CSF completely replaced in a day?
128 / 128
Category:
Neurosciences – Physiology
What is the rate of production of cerebrospinal fluid (CSF)?
The rate of cerebrospinal fluid (CSF) production is approximately 500 mL per day , with a normal total CSF volume of around 150 mL in the ventricular system and subarachnoid space .
CSF is primarily produced by the choroid plexus within the lateral, third, and fourth ventricles .
It is continuously produced, circulated, and reabsorbed into the dural venous sinuses via the arachnoid granulations .
The turnover rate of CSF ensures that it is replaced about 3-4 times per day .
CSF plays essential roles in: ✅ Cushioning the brain and spinal cord ✅ Removing metabolic waste ✅ Maintaining intracranial pressure ✅ Providing nutrients to nervous tissues
Why the Other Options Are Wrong:
250-300 mL/day (Incorrect )
This value is too low ; CSF production is nearly twice this amount .
350-400 mL/day (Incorrect )
While closer, this is still below the normal range .
150-200 mL/day (Incorrect )
This is far below the normal rate and does not reflect the actual CSF production rate.
50-150 mL/day (Incorrect )
This is extremely low and not physiologically normal.
Your score is
The average score is 82%
Restart quiz
Thank you for your feedback.